Você está na página 1de 204

Apresentao

Caros professores orientadores e alunos do Programa


de Iniciao Cientfica PIC da OBMEP
Contedo
Este um nmero especial da Revista do Professor de Matemtica
RPM, elaborado para utilizao no PIC da OBMEP a ser
realizado
Atividades em sala de aula . . . . a. .partir
. . . . .do
. . primeiro
. . . . . . . .semestre
. . . . . . . de
. . .2014.
05
Um jogo aritmtico . . .A. . RPM, . . . . . . .como
. . . . . .seu
. . . .nome
. . . . . .diz,. . . .. .uma . . . . .revista
12 dedicada aos
Seis problemas no triviais professores
equivalentes de Matemtica
. . . . . . . . . . . .da . . . educao
. . . . . . . . 16bsica, a alunos e
O menino . . . . . . . . . . professores
. . . . . . . . . . de
. . .cursos
. . . . . .de . . licenciatura
. . . . . . . . . . em . . . .Matemtica
22 e a todos
aqueles que se interessam pela Matemtica do nvel mdio. O
O problema dos cinco discos: sorte ou sabedoria? . . . . . . . . . . . . . . . 26
tratamento dado aos temas abordados procura ser acessvel e
Calculadora padro: umagradvel,
problema interessante
sem sacrificar . . .o. rigor.
. . . . . .A. .revista
. . . . . . 31
uma publicao da
Uma equao interessante . . . . . . . . . . . . . . . . . . . . . . . . .
Sociedade Brasileira de Matemtica SBM e tem sido editada e . . . . . . . . 37
Painis distribuda sem interrupes desde 1982.
Painel I
A revista publica crnicas, artigos e sees, como Problemas,
O nmero 12 . . . . . . . . . . . . . . . . . . . . . . . . . . . . . . . . . . . . . . . . . . . . . . 41
O leitor pergunta, Livros, Olhando mais de cima, etc. Nos
Painel II
artigos, temas interessantes de nvel elementar ou avanado
Sexta-feira 13 . . . . . . . . . . . . . . . . . . . . . . . . . . . . . . . . . . . . . . . . . . . . . . 43
so apresentados de modo acessvel ao professor e ao aluno
Painel III
O jogo de bilhar . . . . .do. . .ensino
. . . . . . .bsico
. . . . . . .ou
. . .de
. . .cursos
. . . . . . .de
. . .Licenciatura
. . . . . . . 45 em Matemtica.
Painel IV Uma experincia interessante em sala de aula, um problema que
Codicando e decifrando suscita
mensagensuma questo . . . . . .pouco
. . . . . .conhecida,
. . . . . . . . . . .uma
. . . . histria
. 47 que merea ser
Painel V contada ou at uma nova abordagem de um assunto conhecido. Nas
RPM OBMEP

Qual a relao entre os sees,


nmerosa102.564 revistaeconversa
410.256? . com . . . . .o. .leitor,
. . . . . . publicando
. 49 problemas
Painel VI e/ou solues propostas por eles, cartas, resenhas de livros, erros
Uma demonstrao visual encontrados
para a frmula em textos
do sen(A didticos,
+ B) . . . etc. . . . . . . . . . . 51
Painel VII
Valores irracionais de funes trigonomtricas . . . . . . . . . . . . . . . . . . . . 52
1
Painel VIII
Mgica com nmeros . . . . . . . . . . . . . . . . . . . . . . . . . . . . . . . . . . . . . . . . . 54
Painel IX
Destreza ou esperteza? . . . . . . . . . . . . . . . . . . . . . . . . . . . . . . . . . . . . . . . 56
Para este exemplar especial, o Comit Editorial da RPM escolheu
artigos que pretendem ampliar o conhecimento dos alunos em diferentes
tpicos, bem como temas que motivem discusses ou satisfaam
curiosidade terica e histrica de alunos interessados em Matemtica.
Alm disso, publicamos vrias propostas de atividades que podem
ser aplicadas nas salas de alunos de ensino fundamental e mdio. As
atividades tentam despertar a curiosidade dos alunos para tpicos
importantes da Matemtica que so explicitados nas justificativas dos
procedimentos propostos.
Apresentamos tambm uma seo Problemas com:
Problemas I: problemas interessantes com nmeros primos.
Problemas II: uma seleo de problemas extrados do PISA
(Programme for International Student Assessment Programa
Internacional de Avaliao de Alunos).
Problemas III: vinte problemas selecionados entre os publicados na
seo Problemas da RPM, que abrangem a maioria dos tpicos da
educao bsica.
As solues dos problemas propostos esto no final deste fascculo.
Os artigos aqui publicados no apresentam as referncias bibliogr-
ficas, citaes ou agradecimentos que constam nos artigos originais da
RPM.

Comit Editorial da RPM


RPM OBMEP

2
Contedo

Atividades em sala de aula . . . . . . . . . . . . . . . . . . . . . . . . . . . . . . . 05


Um jogo aritmtico . . . . . . . . . . . . . . . . . . . . . . . . . . . . . . . . . . . . . . . 12
Seis problemas no triviais equivalentes . . . . . . . . . . . . . . . . . . . . . . . 16
O menino . . . . . . . . . . . . . . . . . . . . . . . . . . . . . . . . . . . . . . . . . . . . . 22
O problema dos cinco discos: sorte ou sabedoria? . . . . . . . . . . . . . . . 26
Calculadora padro: um problema interessante . . . . . . . . . . . . . . . . . . 31
Uma equao interessante . . . . . . . . . . . . . . . . . . . . . . . . . . . . . . . . . 37
Painis
Painel I
O nmero 12 . . . . . . . . . . . . . . . . . . . . . . . . . . . . . . . . . . . . . . . . . . . . . . 41
Painel II
Sexta-feira 13 . . . . . . . . . . . . . . . . . . . . . . . . . . . . . . . . . . . . . . . . . . . . . . 43
Painel III
O jogo de bilhar . . . . . . . . . . . . . . . . . . . . . . . . . . . . . . . . . . . . . . . . . . . . . 45
Painel IV
Codificando e decifrando mensagens . . . . . . . . . . . . . . . . . . . . . . . . . . . . 47
Painel V
Qual a relao entre os nmeros 102.564 e 410.256? . . . . . . . . . . . . . . . 49
Painel VI
Uma demonstrao visual para a frmula do sen(A + B) . . . . . . . . . . . . . 51
Painel VII
Valores irracionais de funes trigonomtricas . . . . . . . . . . . . . . . . . . . . 52
Painel VIII
Mgica com nmeros . . . . . . . . . . . . . . . . . . . . . . . . . . . . . . . . . . . . . . . . . 54
Painel IX
Destreza ou esperteza? . . . . . . . . . . . . . . . . . . . . . . . . . . . . . . . . . . . . . . . 56
Painel X
Determinante para fatorar . . . . . . . . . . . . . . . . . . . . . . . . . . . . . . . . . . . . . 58
RPM OBMEP

Funes interessantes . . . . . . . . . . . . . . . . . . . . . . . . . . . . . . . . . . . 60
A formiga inteligente . . . . . . . . . . . . . . . . . . . . . . . . . . . . . . . . . . . 66
A demonstrao feita por Heron . . . . . . . . . . . . . . . . . . . . . . . . . . . . . 72

3
A Matemtica da folha de papel A4 . . . . . . . . . . . . . . . . . . . . . . . . . . 75
Retngulo ureo, diviso urea e sequncia de Fibonacci . . . . . . . . . 82
Usando Geometria para somar . . . . . . . . . . . . . . . . . . . . . . . . . . . . . 95
Mdias . . . . . . . . . . . . . . . . . . . . . . . . . . . . . . . . . . . . . . . . . . . . . . . 100
Problemas diversos resolvidos com Geometria Analtica . . . . . . . . 112
A sombra do meu abajur . . . . . . . . . . . . . . . . . . . . . . . . . . . . . . . . 119
Ilha do tesouro. Dois problemas e duas solues . . . . . . . . . . . . . . . 125
Qual mesmo a definio de polgono convexo? . . . . . . . . . . . . . . . 129
A soluo de Tartaglia para a equao do 3o grau
e a emergncia dos nmeros complexos . . . . . . . . . . . . . . . . . . . . . . . 135
Grandezas incomensurveis e nmeros irracionais . . . . . . . . . . . . . 153
A outra face da moeda honesta . . . . . . . . . . . . . . . . . . . . . . . . . . . . . . 162
Nmero de regies: um problema de contagem . . . . . . . . . . . . . . . . . 165
Intuio e Probabilidade . . . . . . . . . . . . . . . . . . . . . . . . . . . . . . . . . . . 170
Problemas I . . . . . . . . . . . . . . . . . . . . . . . . . . . . . . . . . . . . . . . . . . . . 172
Problemas II . . . . . . . . . . . . . . . . . . . . . . . . . . . . . . . . . . . . . . . . . . . . 175
Problemas III . . . . . . . . . . . . . . . . . . . . . . . . . . . . . . . . . . . . . . . . . . . . 181
Soluo dos Problemas . . . . . . . . . . . . . . . . . . . . . . . . . . . . . . . . . . . . 185
RPM OBMEP

4
Atividades em sala de aula

Aprende-se Matemtica fazendo Matemtica.


Apresentamos aqui dois grupos de atividades que
permitem a alunos do ensino fundamental fazer
Matemtica. As folhas contendo as atividades foram
copiadas de nmeros da revista The Mathematics
Teacher* (O professor de Matemtica).
No incio da primeira atividade, alguns exemplos podem
ser feitos coletivamente. Ao fim de cada atividade
interessante comparar os resultados obtidos pelos alunos
reforando o fato de que um mesmo problema pode ter
vrias solues.
O primeiro grupo de atividades trabalha as operaes
aritmticas com nmeros inteiros. O segundo grupo de
atividades trabalha com visualizao de figuras no espao,
permitindo aos alunos descobrir padres e fazer
conjecturas. As primeiras partes podem ser aplicadas em
salas do ensino fundamental e a parte final no ensino
mdio, explorando generalizaes e suas representaes
algbricas.
Na pgina 15 deste exemplar est uma soluo de cada
RPM OBMEP

problema proposto no Primeiro grupo.

* Publicao do National Council of Teachers of Mathematics


NCTM, Reston, Virginia, USA.

5
Primeiro grupo: atividades I, II e III.
I. Instrues
Em cada linha h 5 nmeros e um sexto nmero, chamado total.
Coloque os sinais +, , , e parntesis, colchetes, chaves, de
modo que o resultado das contas indicadas seja o total.
Os 5 nmeros devem ser usados, cada um deles uma s vez, em
qualquer ordem.
Exemplo: 7, 8, 1, 9, 9 total: 16.
Uma soluo: (9 9) (7 + 8 + 1) = 16.

1. 1, 5, 3, 6, 10 total: 5

2. 8, 11, 9, 1, 8 total: 2

3. 11, 10, 15, 20, 3 total: 6

4. 12, 18, 3, 11, 12 total: 8

5. 4, 16, 10, 24, 25 total: 1

6. 17, 14, 7, 17, 13 total: 7

7. 2, 9, 5, 9, 4 total: 22

8. 3, 6, 10, 5, 7 total: 2

9. 8, 6, 11, 5, 21 total: 7
RPM OBMEP

10. 6, 1, 2, 2, 17 total: 8

6
II. Instrues
Trabalhe com os nmeros 11, 14, 3, 19 e 9.
Coloque os sinais +, , , e parntesis, colchetes, chaves, para
obter todos os nmeros de 1 at 11.
Os nmeros 11, 14, 3, 19 e 9 devem ser usados, cada um deles uma
s vez, em qualquer ordem.

1. (11 + 14 19 + 3) 9 = 1

2.

3.

4.

5.

6. 11 [(19 + 9) 14 + 3] = 6

7.

8.

9.

10.
RPM OBMEP

11. [9 (19 14) 3] 11 = 11

7
III. Instrues
Trabalhe com os nmeros 2, 3, 5, 7 e 11. (Observe que so os cinco
primeiros nmeros primos.)
Coloque os sinais +, , , e parntesis, colchetes, chaves, para
obter os nmeros pedidos nos itens de 1 a 10 abaixo.
Os nmeros 2, 3, 5, 7 e 11 devem ser usados, cada um deles uma s
vez, em qualquer ordem.

1. Escreva (seguindo as instrues) o menor primo mpar.


3 = [(2 5) + (7 3)] 11
2. Escreva o menor nmero natural mpar.

3. Escreva o menor nmero natural primo.

4. Escreva o menor nmero natural composto.

5. Qual o maior nmero natural composto que voc consegue escrever?

6. Qual o maior nmero natural mpar que voc consegue escrever?

7. Escreva o menor nmero natural que voc consegue achar, usando


uma s vez cada uma das operaes.

8. Determine e escreva o maior nmero natural par possvel, usando


uma s vez cada uma das operaes.

9. Escreva um nmero natural usando apenas subtraes.


RPM OBMEP

10. Determine e escreva o maior nmero primo possvel obedecendo s


instrues.

8
Segundo grupo Descobertas com cubos: atividades I, II, III e IV
Descrio
Um cubo grande, decomposto em cubos pequenos, mergulhado
numa lata com tinta. Pergunta-se quantas faces dos cubos pequenos
ficaro pintadas.
Objetivos
Estudantes visualizaro figuras no espao, construiro uma tabela,
descobriro padres na tabela e, usando esses padres, faro conjeturas.
Diretrizes
Distribuir para cada aluno folhas com as atividades ou coloc-las
no quadro-negro. Sugere-se dividir a classe em grupos de dois alunos,
deixando-os trabalhar juntos.
Aps completar a atividade I, os estudantes devem registrar seus
resultados na tabela (atividade III). Certifique-se de que todos os
estudantes tm os valores corretos, pois conjeturas sero feitas a partir
dos dados da tabela. Poucos estudantes conseguiro completar a tabela
para um cubo 10 10 10, a menos que algum padro tenha sido
identificado (atividade IV). Pergunte: Existem constantes em uma
coluna? Existem mltiplos?. Sugerir aos alunos que procurem fatores
comuns vai ajud-los a reconhecer padres. Por exemplo, 0, 6, 24, 54 e
96 so as 5 primeiras entradas em uma das colunas. Um padro torna-se
mais visvel se esses nmeros forem escritos como 0, 6 1, 6 4,
6 9 e 6 16.

I. Responda s perguntas a seguir para cada um dos cubos das figuras 1,


2, 3 e 4.
a) Quantos cubos pequenos h no cubo grande?
Se esse cubo maior for jogado numa lata de tinta e totalmente
submerso:
b) Quantos cubos pequenos tero 3 faces pintadas?
RPM OBMEP

c) Quantos cubos pequenos tero 2 faces pintadas?


d) Quantos cubos pequenos tero 1 face pintada?
e) Quantos cubos pequenos tero 0 face pintada?

9
f) Qual a soma de suas respostas em b), c), d) e e)?

II. Complete a figura 5, desenhando um cubo 6 6 6. Responda


novamente s perguntas a), b), c), d), e), e f).
III. Agora registre as informaes na tabela abaixo. Considerando, em
cada caso, o lado dos cubos pequenos como unidade.

Comprimento do nmero de cubos pequenos nmero total


lado do cubo com faces pintadas de cubos
maior 0 1 2 3 4 pequenos

2
3
4
5
6
RPM OBMEP

10
IV. Voc observa padres na tabela? Em caso afirmativo, complete a
tabela para um cubo 7 7 7. Em caso negativo, desenhe o cubo e
ento complete a tabela.
Voc realmente pegou o jeito? Se voc acha que sim, complete a
tabela para um cubo 10 10 10.
Eis uma questo que pode ser usada para culminar essa atividade:
Seja n o comprimento de um lado do cubo. Quando voc completar
na tabela a linha correspondente a n, a soma dos valores dessa
linha ser n3?

Nota do tradutor
Para completar a tabela para um cubo de lado n, pode-se considerar
o cubo grande como sendo formado por n camadas horizontais. Cada
camada um quadrado n x n . Nos esboos abaixo, o nmero em cada
cubo pequeno indica quantas de suas faces ficam pintadas aps a imerso
do cubo na lata de tinta:

faces pintadas 0 1 2 3

camadas sup. e inf. 2(n 2)2 8(n 2) 8

camadas intermedirias (n 2)3 4(n 2)2 4(n 2)

total (n 2)3 6(n 2)2 12(n 2) 8


RPM OBMEP

Adaptado do artigo
Atividades em sala de aula
Renate Watanabe, RPM 61

11
Um jogo aritmtico

Introduo
Como seria bom se pudssemos fazer da Matemtica uma
fonte de prazer ainda maior do que ela j . Isso possvel
se tivermos como aliado um poderoso recurso ldico: o
jogo. Proponho aqui um jogo aritmtico, que muito fcil
de aprender, e pode ser jogado por duas ou mais pessoas.
A idia sortear um nmero que, em seguida, deve ser
obtido de outros, atravs das quatro operaes. Para
representar os inteiros usamos as cartas de um baralho
comum, com exceo dos coringas. O s (A), o valete
(J), a dama (Q) e o rei (K) representam os nmeros 1, 11,
12 e 13 respectivamente.

Formando nmeros
Estamos acostumados ao clculo de expresses aritm-
ticas, isto , dada uma expresso envolvendo nmeros e
operaes matemticas, encontrar o nmero que lhe
corresponde. Aqui se pede a soluo do problema rec-
proco: dado um nmero, encontrar uma expresso aritm-
tica que corresponde a esse nmero. No jogo s permi-
tido o uso das 4 operaes aritmticas bsicas (adio,
RPM OBMEP

subtrao, multiplicao e diviso) e de parnteses. Por


exemplo, com os nmeros 2, 5, 7, 8 e 11, alguns dos
nmeros que podemos formar so:

12
19 = 11 + 8
33 = (5 2) 11
64 = (8 2) (5 + 11)
81 = 2 5 7 + 11
80 = ((5 2) 7 11) 8
100 = (7 + 2 + 11) 5
Note que:
1. No necessrio usar todos os inteiros disponveis;
2. O uso de parnteses no tem restries. Podemos tambm usar
parnteses encaixados como na expresso do nmero 80;
3. S podemos usar cada inteiro disponvel uma nica vez;
4. No se pode formar nmeros por justaposio, isto , com o 5 e o 2
no podemos formar nem o 25 nem o 52.
Na prtica, no precisamos escrever a expresso usando parnteses.
Para formar o 80, declaramos: 5 menos 2 3; 3 vezes 7 21; 21 menos
11 10; 10 vezes 8 80. Para formar o 64, declaramos: 8 dividido por 2
4; 5 mais 11 16; 4 vezes 16 64.
O que necessrio
1. Um baralho (descartam-se os coringas);
2. Cada jogador pode, se julgar necessrio, ter caneta ou lpis e uma ou
mais folhas de papel.
Incio do jogo
Colocamos o baralho na mesa, com as cartas voltadas para baixo,
num monte, de modo que no se possa ver que nmeros representam.
Escolhe-se de comum acordo um participante para iniciar a rodada. Ento
os itens 1, 2 e 3 a seguir devem ser repetidos at que haja um vencedor.
O jogo
1. Escolhemos a carta de cima do monte e multiplicamos seu valor por
13: em seguida, somamos o resultado do produto ao valor de uma
RPM OBMEP

segunda carta retirada de cima do monte. Obteremos um nmero


entre 14 a 182 (13 1 + 1 = 14 e 13 13 + 13 = 182). Esse o
nmero que deve ser formado na rodada. As duas cartas tiradas vo
para baixo do monte;

13
2. O jogador da vez retira uma carta de cima do monte e a pe com o
nmero para cima, no centro da mesa, ou ao lado da ltima carta
retirada;
3. Ele ento faz suas anotaes e clculos, e ter duas opes:
a) Formar o nmero sorteado ganhando a rodada (1 ponto). Nesse
caso, o jogador da vez passa a ser aquele que est sua esquerda
e colocam-se as cartas retiradas debaixo do monte. A partir daqui,
precisa-se sortear um novo nmero, portanto retorna-se ao item 1
para o incio de outra rodada;
b) Passar a vez ao jogador da sua esquerda. Em seguida d-se
prosseguimento rodada retornando-se ao item 2.
Quem vencer um total de 3 rodadas primeiro vence o jogo. Enquanto
isso no ocorrer, repetem-se os itens 1, 2 e 3 sucessivamente.

Um exemplo
O primeiro jogador, A, tira a carta de cima do monte, digamos, 5 e a
carta seguinte, uma dama. Ento, o nmero a ser formado na rodada
ser 13 5 + 12 = 77. As duas cartas tiradas vo para baixo do monte.
O segundo jogador, B, tira a carta de cima do monte, digamos 8 e a
coloca aberta na mesa. No d para formar 77 com o nmero 8. Ele
passa a vez para A (se o jogo s tiver dois jogadores), que tira, digamos,
6. Com 8 e 6 e as quatro operaes ainda no d para obter 77. O 6 fica
aberto na mesa e A passa a vez para B que tira, digamos, um valete. Com
6, 8 e 11 no d para obter 77. a vez de A que tira, digamos, 3.
A dois casos podem ocorrer:
(1) A percebe que 6 11 + 8 + 3 = 77. Ento a rodada termina, A ganha
1 ponto, as cartas vo para baixo do monte e tudo comea de novo
com B tirando as duas cartas de cima do monte para obter um novo
nmero.
(2) A no percebeu que podia obter 77 com as cartas da mesa e passa a
vez para B. Se B obtiver o 77, ele que ganha um ponto e uma nova
rodada se inicia. Se B no obtiver o 77, ele tira mais uma carta do
RPM OBMEP

monte e assim, sucessivamente, at que um dos jogadores conseguir


formar o 77 com as cartas que esto abertas na mesa.

14
Concluso
Tenho jogado com amigos j h algum tempo. Estou convencido de
que esse um jogo intelectualmente estimulante e muito agradvel.
claro que existem muitos jogos com essas qualidades, mas esse tem a
vantagem de ser matematicamente educativo. Alm disso, uma forma
de viver a Matemtica, interagir com ela, senti-la, toc-la. Tambm estou
certo de que podemos criar jogos matemticos que trabalhem a
compreenso de teoremas e suas demonstraes, bem como suas
aplicaes na resoluo de problemas..., mas esse j um outro assunto...

Adaptado do artigo
Um jogo aritmtico
Eric Campos Bastos Guedes, RPM 55.

____________________________________
Respostas das Atividades Primeiro grupo
I II III
1. 10 (6 + 3 + 1) + 5 2. 9 {14 [(19 + 3) 11]} 2. 1 = 5 [( 11 + 3) 7 + 2]
2. 11 + 1 9 (8 8) 3. {11 [(19 + 9) 14]} 3 3. 2 = [(5 + 3) (11 7)] 2
3. 11 [(20 15) 3 10] 4. 19 9 + 11 14 3 4. 4 = [(5 + 3) (11 7)] 2
4. (11 + 3) 12 + 18 12 5. 9 {14 [(11 + 19) 3]} 5. 2 3 5 7 11 = 2310
5. (4 + 16) 10 (25 24) 7. 14 {[(3 + 19) 9] 11} 6. 11 7 5 (3 + 2) = 1925
6. 17 17 + (14 13) 7 8. 9 [(11 3) (14 + 19)] 7. 0 = [(11 + 3) 2 7] 5
7. (9 9) + ( 5 4) + 2 9. 11 3 (14 + 19) + 9 8. [(11 3) (5 + 7)] 2 = 48
8. (7 6) + [3 (10 5)] 10. 19 9 + 11 14 + 3 9. 5 {3 [(11 2) 7]} = 4
9. 5 [(11 + 21) 8] + 6 10. 11 7 5 3 2 = 1153
10. (17 1) [(6 2) 2]
RPM OBMEP

15
Seis problemas
no triviais equivalentes

O poder da Matemtica de relacionar o que


aparentemente no tem relao.

Neste artigo dois problemas sero chamados equivalentes


se sua resoluo fizer uso do mesmo tipo de Matemtica.
Problemas equivalentes evidenciam talvez a qualidade
mais importante da Matemtica: a possibilidade de um
conceito terico ser usado como modelo para muitas
idias diferentes. fcil produzir exemplos.
Se o conceito terico for combinaes, como em
Probabilidade, essa ideia tambm pode ser usada para
determinar as leis de Mendel em Biologia, para calcular
coeficientes binomiais, para calcular certas probabili-
dades em jogos de baralho, para achar o nmero de
polgonos de vrios tipos que tenham pontos arbitrrios,
como vrtices, e assim por diante, quase que indefini-
damente.
Mas difcil produzir bons exemplos quando se desejam
problemas equivalentes em uma escala muito menor, onde
mesmo tipo de Matemtica no significa Matemtica de
um mesmo campo, ou de um mesmo tpico dentro de
um campo ou assunto que usem as mesmas ideias.
Especificamente tentei encontrar problemas satisfazendo
RPM OBMEP

as seguintes condies:
1) Os problemas deveriam ser matematicamente
idnticos at nos nmeros usados na sua resoluo.

16
2) At que uma resoluo fosse examinada, nada no problema deveria
indicar que o mesmo tipo de Matemtica pudesse ser usado. Assim
os problemas deveriam, na medida do possvel, vir de tpicos
totalmente desvinculados dentro da Matemtica ou dentro de
aplicaes da Matemtica.
3) Os problemas deveriam estar no mbito da Matemtica do ensino
fundamental ou ensino mdio, quanto mais simples, melhor.

Problemas

1. Expresse 1 como soma de duas fraes de numerador 1 (fraes do


2
1
tipo , n um inteiro positivo).
n
2. Ache todos os retngulos cujos lados tenham por medida nmeros
inteiros e que tenham rea e permetro numericamente iguais.
3. Quais pares de inteiros positivos tm mdia harmnica igual a 4?
4. Ache os possveis pares de inteiros cujo produto seja positivo e igual
ao dobro de sua soma.
5. Dado um ponto P, ache todos os n tais que o espao em torno de P
possa ser coberto, sem superposio, por polgonos regulares,
congruentes, de n lados.
6. Para quais inteiros positivos n > 2, o nmero 2n divisvel por
n 2?
Para mostrar a equivalncia, verificaremos que os seis problemas se
reduzem resoluo de uma equao que a caracterizao do primeiro.

Reduo dos problemas a uma equao


Problema 1
1 1 1
Se 1 for a soma de duas fraes de numerador 1 ento = + ,
2 2 p q
onde p e q so inteiros positivos. (A equao ser resolvida mais
RPM OBMEP

adiante.)

17
Problema 2
Sejam a e b o comprimento e a largura do retngulo procurado. Como
a rea e o permetro so numericamente iguais, temos:
2a+ 2b = ab
2(a + b) = ab
a+b 1
=
ab 2
1 1 1
+ = .
a b 2
Como a e b devem ser inteiros e positivos essa ltima equao tem a
mesma forma que a equao do Problema 1.

Problema 3
2xy
A mdia harmnica de dois nmeros x e y .
x+ y
Sejam x e y inteiros positivos. Das condies dadas:

2 xy
=4
x+ y
xy
=2
x+ y
x+ y 1
= .
xy 2

A ltima equao tem a mesma forma que a equao na 3a linha do


Problema 2 e assim se reduz equao do Problema 1.

Problema 4
Sejam x e y dois inteiros, z o seu produto, z > 0. Os nmeros x e y
devem ser positivos pois a sua soma e produto so positivos. Das
z
RPM OBMEP

condies dadas obtm-se xy = z e x + y = . As condies juntas


2
xy x+ y 1
implicam: x + y = = .
2 xy 2
18
Essa ltima equao idntica equao da ltima linha do Problema 3
e assim reduz-se equao do Problema 1.

Problema 5
Este o problema mais difcil de caracterizar. Seja k o nmero de
polgonos com vrtice em P. Se os polgonos no se sobrepuserem,
forem regulares e congruentes, utilizando a notao da figura abaixo,
360
obter-se-: 1 = 2 = ... = k = em graus.
k

Mas os ai so medidas de ngulos de polgonos regulares de n lados,


portanto

(n 2)180
i = 1 < i < k.
n
Temos ento:
360 (n 2)180
=
k n
2 (n 2)
=
k n
2n = (n 2)k
2n + 2k = nk.
Das condies do problema segue-se que n e k devem ser inteiros
positivos e portanto essa equao tem a mesma forma que a da primeira
linha do Problema 2.

Problema 6
RPM OBMEP

Se 2n divisvel por n 2 ento 2n = (n 2)k, onde k um numero


inteiro. Essa equao idntica a uma das equaes do Problema 5 e
portanto se reduz do Problema 1.

19
Equao diofantina
Assim, os seis problemas podem ser resolvidos considerando-se a
equao do Problema 1. Devido s condies, essa equao uma
equao diofantina e sua soluo interessante.
1 1 1
1. Seja = + onde p e q so inteiros positivos.
2 p q
1 1 1 1
2. impossvel termos > e > (pois a soma no chegaria a
4 p 4 q

1 1
ser 1 ) e assim pelo menos uma das fraes ou deve ser maior
2 p q

1 1 1
do que ou igual a . Suponham os .
4 p 4

3. Ento p = 1, 2, 3 ou 4.

1 1
4. p = 1 = 1 + q = 2 o que no possvel pois q posi-
2 q

1 1 1 1
tivo; p = 2 = + = 0 , que no tem soluo;
2 2 q q
p = 3 q = 6;
p = 4 q = 4.
5. Por causa da simetria de p e q na equao original, obtemos resultados
1 1
correspondentes se .
q 4

6. Portanto temos 3 solues:


(p, q) = (3, 6); (p, q) = (4, 4); (p, q) = (6, 3).

Solues
RPM OBMEP

Todos os problemas esto agora resolvidos.

1 1 1 1 1 1 1
Problema 1 A resposta = + = + = + .
2 3 6 4 4 6 3

20
Problema 2 Existem dois retngulos satisfazendo as condies dadas:
um 4 4 e o outro, 3 6.
Problema 3 Duas respostas: 4 e 4 ou 3 e 6 so pares de inteiros cuja
mdia harmnica 4.
Problema 4 Os pares so idnticos ao do problema 3.
Problema 5 Os nicos polgonos regulares congruentes que, sem
superposio, cobrem o espao em torno de P (e assim cobrem o plano)
so os polgonos de 3 lados (seis tringulos eqilteros em torno de P),
os de 4 lados (quatro quadrados em torno de P) e os de 6 lados (trs
hexgonos regulares em torno de P), como se v na figura.

Seis tringulos, quatro quadrados, trs hexgonos.

Problema 6 A resposta : n 2 um divisor de 2n quando n = 3,


n = 4 ou n = 6. (A condio n > 2 no problema original garante ser
n 2 positivo. Sem essa condio existiriam as solues n = 1, n = 0
ou n = 2).
Resumo
Os seis problemas formam um grupo de problemas, no triviais
equivalentes que podem ser usados em classes de ensino fundamental e
mdio. fcil desenvolver outros grupos de problemas mais apropriados
para o uso em lgebra Elementar ou Geometria. Tais grupos de
problemas podem ser usados para demonstrar o poder de um pouco de
Matemtica abstrata na resoluo de exerccios que, primeira vista,
pareciam no relacionados.

Adaptado do artigo
RPM OBMEP

Seis problemas no triviais equivalentes


Zalman Usiskin, RPM 04.

21
O menino

No havia sada. Teria que esperar por trs horas o


prximo vo para Salvador. Arquiteto por formao e
profisso, tinha que apresentar um projeto na manh
seguinte, numa cidade prxima capital da Bahia.
Assentei-me como pude. Teria que olhar para aquele
relgio pendurado no teto por trs horas. Como se no
bastasse, o relgio registrava os segundos. Relgios que
registram segundos demoram mais que os que no o
fazem.
Alguns apelam para palavras cruzadas, outros giram os
polegares e eu, como o vcio do cachimbo entorta a boca,
trao em folhas de papel as formas que se me apresentam
no ambiente que alcanado pelas retinas. Lpis e papel
na mo, registrava dois lances de escada e uma escada
rolante que surgiram a minha frente. Mal traara as
primeiras linhas, deparei-me com uma questo que me
intrigou: quantos degraus deveria desenhar na escada
rolante? Em vo, tentei contar os degraus visveis. Se a
escada parasse, poderia cont-los. Tive mpetos de apertar
o boto vermelho prximo ao corrimo, onde se lia
RPM OBMEP

PARAR. Meu censurador no permitiu que o fizesse.


Fiquei ali, inerte, com o cachimbo na mo e sem poder
fumar.

22
Um menino sentou-se ao meu lado, brincando com uma bola. Sem
tirar os olhos da bola, ela disse em voz clara e pausada:
Pepino no parece inreal?
Olhei-o, ligeiramente, com o canto dos olhos e, sem nada dizer,
retornei ao meu cachimbo apagado. Alguns instantes depois, senti minha
camisa ser puxada e escutei novamente:
Pepino no parece inreal?
Dessa vez, com uma mo segurando a bola e com a outra puxando a
minha camisa, ele me olhava firmemente.
No inreal, irreal.
Pois , no parece?
Aquela insistncia irritou-me. Eu, diante do mais intrincado problema
da existncia humana quantos degraus ficam visveis quando a escada
rolante pra e aquele menino me questionando sobre a realidade de
um pepino! Tentando dissuadi-lo, resolvi apresentar-lhe a complexidade
do problema que me afligia.
Olha, menino, estou tentando desenhar aquelas escadas e no sei
como acabar o desenho da escada rolante. Quantos degraus devo
desenhar? Meu desenho est parado e a escada est subindo. Se a escada
parasse de repente, quantos degraus ficariam visveis?
Sem nada dizer, colocou a bola sobre a cadeira, subiu e desceu a
escada (que sobe). Apontando para o relgio, disse:
Eu deso a escada duas vezes mais rpido do que subo.
E repetiu sua viagem ao vo da escada, mostrando-me que, no mesmo
tempo em que dava um passo para subir, dava dois para descer.
Novamente sem nada dizer, comeou a subir a escada rolante, contando
os passos: um, dois, trs, ..., num total de vinte passos. Do alto da escada,
olhou-me como quem estivesse fazendo a mais bvia das coisas, e
RPM OBMEP

comeou a descer a mesma escada rolante, contando os passos: um,


dois, trs, ..., num total de trinta e cinco passos.
Em seguida tomou o lpis e o papel de minhas mos e completou,
com traos infantis, o meu desenho.

23
Nenhum censurador poderia me conter. Levantei-me bruscamente e
apertei o boto vermelho. Ansioso, comecei a contar os degraus. Para
meu espanto, correspondia ao desenho do menino. Com a maior seriedade
que j tive em minha vida voltei-me para o menino e perguntei-lhe:
Por que o pepino parece inreal?

Quantos degraus o menino desenhou?


Vamos resposta:
Vamos tomar como unidade de tempo o tempo no qual o menino d
um passo subindo a escada. Seja n o nmero de degraus da escada rolante
que desaparecem (ou surgem) na unidade de tempo. Como o menino
deu 20 passos para chegar ao topo da escada, ele demorou 20 unidades
de tempo.
Isso significa que desapareceram 20n degraus. Chamando de N o
nmero de degraus visveis, temos:
N 20
N = 20 + 20n ou n = . (1)
20
O menino deu 35 passos para descer a escada rolante (que sobe).
Lembremos que a frequncia de seus passos duas vezes maior na
descida que na subida. Ou seja, o tempo de dar dois passos descendo
1
igual ao de um passo subindo. Cada passo na descida demora da
2
unidade de tempo.
35
Ele demorou unidades de tempo para descer a escada. Isso
2

significa que surgiram 35n degraus novos. Assim,


2
35n 70 2 N
N = 35 ou n = . (2)
2 35
RPM OBMEP

Igualando (1) e (2):


N 20 70 2 N
=
20 35

24
35N 700 = 1400 40N
ou
75N = 2100, de onde
2100
N= = 28
75
O menino desenhou 28 degraus.

Adaptado do artigo
O menino
Ledo Vaccaro Machado, RPM 42.

Desafio
Distribuir os nmeros de 1 a 9 dentro dos pontos brancos (de interseco),
sem repetir, de forma que a soma dos nmeros pertencentes circun-
ferncia externa seja exatamente igual soma dos nmeros pertencentes
a cada uma das circunferncias internas.

RPM OBMEP

Resposta na pgina 171.

25
O problema dos cinco discos:
sorte ou sabedoria?

Neste artigo queremos mostrar uma curiosidade sobre o


antigo problema dos cinco discos. A mais bela
apresentao desse problema encontra-se em O homem
que calculava (Tahan, Malba 32a edio. Record, Rio
de Janeiro, 1986). Nele contada uma lenda onde trs
prncipes muito sbios e conhecedores da Matemtica
que pretendiam casar-se com a princesa Dahiz, filha do
rei Cassim.
A prova dos cinco discos foi proposta por um grande
sbio da corte para decidir qual dos trs pretendentes era
o mais inteligente.
Foram mostrados aos prncipes cinco discos, sendo dois
pretos e trs brancos, todos de mesmo peso e tamanho.
Em seguida vendaram-lhe os olhos e, ao acaso, foi
pendurado um desses discos s costas de cada um dos
trs. Disse o rei: Cada um de vs ser interrogado
particularmente e aquele que descobrir a cor do disco
que lhe coube por sorte, ser declarado o vencedor. O
primeiro a ser interrogado poder ver os discos dos
outros dois, ao segundo ser permitido ver o disco do
RPM OBMEP

terceiro, e o terceiro ter que formular a resposta sem


ver nada. Aquele que der a resposta certa ter que
justific-la.

26
Aconteceu ento que o prncipe Camoz quis ser o primeiro. Viu os
dois discos dos seus adversrios e errou. Em seguida, sabendo que
Camoz havia errado, o prncipe Benefir se prontificou em ser o segundo,
mas tambm errou. Aradim, o terceiro prncipe, acertou com absoluta
segurana. Qual foi a resposta do prncipe Aradim e como ele descobriu?
Esse o problema dos cinco discos. Malba Tahan d uma inteligente
soluo desse problema, onde conclui tambm que Aradim foi
considerado o mais inteligente entre os trs prncipes.
Eis a soluo de Malba Tahan: o prncipe Aradim afirmou que o seu
disco era branco e justificou da seguinte maneira: Se Camoz (o primeiro
a falar) tivesse visto dois discos pretos, ele obviamente teria acertado.
Como ele errou, conclui-se que viu dois discos brancos, ou um preto e
um branco. Na hiptese de Benefir ter visto em minhas costas um disco
preto, ele (usando o mesmo raciocnio que fiz com relao a Camoz)
teria acertado. Logo, ele s pode ter visto um disco branco e, portanto, o
meu disco branco.
A curiosidade que pretendemos apresentar que, sob o ponto de
vista matemtico e levando em conta somente o acerto da cor do disco,
a chance de erro dos dois anteriores era bem pequena, o que torna
discutvel a concluso de que Aradim fosse mais inteligente que Camoz
ou Benefir.
Com efeito, vamos calcular as probabilidades de acerto da cor do
disco de cada um dos trs prncipes, levando em conta que todos eles
so sbios.
As possveis distribuies dos discos
Sejam b = (disco branco) e p = (disco preto). Por simplicidade
escrevemos
A = Aradim, B =Benefir e C = Camoz.
Ento a ordem em que os prncipes se apresentaram para serem
interrogados pode ser representada por uma terna ordenada (C, B, A).
RPM OBMEP

A ttulo de exemplo, perguntamos quantas maneiras diferentes podem


C possuir disco branco, B possuir disco preto e A possuir disco preto?
Isto , de ocorrer (b, p, p).

27
Sabemos que existem trs discos brancos b1, b2 e b3 e dois discos
pretos p1 e p2. Por uma simples contagem, obtemos seis maneiras
diferentes de ocorrer (b, p, p), a saber:
(b1, p1, p2), (b1, p2, p1), (b2, p1, p2), (b2, p2, p1),
(b3, p1, p2) e (b3, p2, p1).
claro que o nmero total de maneiras em que podem ser distribudos
os discos aos prncipes A5, 3 = 60. Descrevendo esses casos, obtemos:

Eventos Frequncia
E1 = (b, b, b) 6
E2 = (p, b, b) 12
E3 = (b, p, b) 12
E4 = (b, b, p) 12
E5 = (p, p, b) 6
E6 = (p, b, p) 6
E7 = (b, p, p) 6

Lembretes
a) Se os conjuntos unitrios de um espao amostral finito U tm todos
a mesma probabilidade, ento a probabilidade de um evento A
qualquer de U ser dada por:
n( A)
P ( A) =
n(U )
onde n(A) o nmero de elementos do evento A e n(U) o
nmero total de elementos do espao amostral U.
b) Nas mesmas condies de a), se A1, A2, ..., An so eventos disjuntos
entre si,
n( A1 ) + n( A2 ) + ... + n( An )
P ( A1 A2 ... An ) = .
n(U )
RPM OBMEP

Como o problema afirma que a escolha dos discos feita ao acaso,


segue-se que o espao amostral associado ao problema satisfaz as
condies necessrias para a validade de a) e b). No difcil verificar
tambm que o acerto da cor do disco admite uma estratgia que maximiza
28
a probabilidade de vitria de cada concorrente e garante, com
probabilidade 1, a existncia de um vencedor, que certamente ser nico
uma vez que o processo terminaria no momento em que um dos
concorrentes acertasse a cor do seu disco. Como os concorrentes
supostamente so sbios, razovel admitir que eles seguiro a melhor
estratgia em cada situao e portanto teremos
P(C) + P(B) + P(A) = 1
onde P(C), P(B) e P(A) so, respectivamente, as probabilidades de
vitria de C, B e A.
A estratgia tima e a correspondente probabilidade de vitria de C.
Se C vir dois discos pretos nos seus adversrios, saber que restam
trs discos brancos. Responder ento com absoluta segurana que possui
um disco branco. Assim o evento E7 lhe favorvel. Caso C veja dois
discos brancos, saber que restam dois discos pretos e um disco branco.
Logo responder possuir disco preto, contando com a probabilidade
2/3 de acertar. Consequentemente, o evento E2 lhe favorvel e o
evento E1 lhe desfavorvel. Suponhamos agora que C tenha visto
um disco branco e um disco preto em seus concorrentes. Concluir que
restam dois discos brancos e um disco preto. Logo, dever responder
que possui um disco branco, contando com a probabilidade 2/3 de
acertar. Segue que os eventos E3 e E4 lhe so favorveis e o evento E6
lhe desfavorvel.
Em resumo, usando essa estratgia, C ir acertar na hiptese de ter
ocorrido qualquer um dos eventos disjuntos E2, E3, E4 ou E7 e ir
errar se houver E1, E5 ou E6. Segue-se, ento, que:

n( E2 ) + n( E3 ) + n( E4 ) + n( E7 ) 6 + 12 + 12 + 12 7 .
P (C ) = = =
n(U ) 60 10

Isso mostra que a probabilidade vitria do prncipe Camoz, o


primeiro candidato, de 70%, contando com a sua sabedoria, restando
RPM OBMEP

assim apenas 30% de probabilidade para que os outros dois prncipes


tivessem chance de serem apenas interrogados.

29
Considerando ainda que Aradim s seria interrogado caso Benefir (o
segundo interrogado) tambm errasse, pode-se mostrar que ele o que
teria a menor chance de ser escolhido como noivo de Dahiz.
No entanto, Aradim possuidor de muita sorte, pois os dois primeiros
concorrentes erraram.
Para completar, a probabilidade de Benefir acertar de 20% e a
probabilidade do prncipe Aradim acertar de apenas 10%.

A reabilitao de Aradim
Esse clculo, entretanto, diz respeito s ao fato de acertar, ou no, a
cor do seu disco. Acontece que o rei dissera que os prncipes deveriam,
tambm, justificar a resposta correta.
Fica a pergunta do que o rei entendia por justificar.
Seria aceitvel, em caso de dvida, uma adivinhao educada, isto ,
uma opo pela alternativa mais provvel? Ou seria necessria uma
explicao lgica de como se chegou nica alternativa correta possvel?
Neste caso, quais seriam as probabilidades de vitria de cada um dos
trs concorrentes?

Adaptado do artigo
O problema dos cinco discos: Sorte ou Sabedoria?
Ma-To Fu e Roberto Elias, RPM 11.
RPM OBMEP

30
Calculadora padro:
um problema interessante

Suponhamos que voc tem uma calculdadora de bolso,


padro, de 8 dgitos, que efetua as quatro operaes,
+, , , e extrai razes quadradas. Ser possvel, usando
essa calculadora, extrair a raiz n-sima de um nmero
qualquer?
Na verdade, dado um nmero real x, no negativo, usando
somente as teclas , +, , , possvel achar xp/q,
onde p e q so nmeros naturais. Vamos explicar como
isso possvel mostrando alguns exemplos.
3
Exemplo 1: Calcule 5.
3
Seja x = 5 . Ento x3 = 5. Multiplicando por x os dois
lados da igualdade, obtemos x4 = 5x ou x = 4 5 x .
Inicialmente criamos uma sequncia de nmeros reais
3
sendo x1 uma aproximao de 5 (por exemplo, x1 = 1)
e xn +1 = 4 5 xn , n = 1, 2, 3,". Vamos mostrar os valores
de alguns termos da sequncia obtidos na calculadora e,
em seguida vamos dar uma justificativa do por que a
RPM OBMEP

3
sequncia converge para 5.

Tomemos x1 = 1. Ento, x2 = 4 5 , x3 = 4 5 4 5 , e assim


por diante.

31
Vejamos os valores calculados sendo que a notao [*] significa
que apertamos a tecla *:

[5] [ ][ ] 1.4953487
Com o valor anterior mantido na tela, fazemos
[] [5] [=] [ ][ ] 1.653591
Repetindo sempre os comandos [] [5] [=] [ ][ ] obtemos
1.6957019
1.7063962
1.7090802
1.7097519
1.7099199
1.7099619
1.7099724
1.7099750
1.7099757
1.7099758
1.7099759
1.7099759,
o que indica que uma aproximao para 3
5 com erro menor que 107
1,7099759.
Uma idia do por que funciona: consideremos a expresso

4 4 4 4 com infinitos radicais. Podemos escrever


5 5 5 5"

1 1 1 1 1 1 1 1
4 4 4 4
5 5 5 5" = 5 4 516 5 64 5 256 " = 5 4 +16 + 64 + 256 " .

1
1 1 1 1 4 1
Sabemos que 4 + 16 + 64 + 256 + " = 1 1 = 3 , pela frmula da
4
soma dos termos de uma PG infinita, logo, temos

4 4 4 4 3
RPM OBMEP

5 5 5 5" = 51/3 = 5 = x. Ou seja, admitindo que existe o limite


3
de xn quando n tende ao infinito, ento, esse limite ser 5.

32
Um outro modo de verificar o valor x do limite da sequncia
(admitindo que esse limite exista) :

x = 4 54 54 54 5" , logo, x 4 = 54 54 54 5" = 5 x. Como x

3
diferente de zero, a igualdade implica x = 5.

Exemplo 2: Calcule 5 5 .

Seja x = 5 5 . Ento, x5 = 5 e vemos que x8 = x5x3 = 5x3 que implica


8
x = 5 x3 . Vamos construir uma sequncia x1, x2, x3, ..., xn, ..., com
x1=1 e os outros termos como na tabela, usando a calculadora para obter
os nmeros aps as flechas.
1
x2 = 8 5 x13 = 8 5 = 58 = 5 1.2228445
1+ 3
8 8
x3 = 8 5 x23 = 5 53 = 58 64 = 5 125 1.3186680

1+ 3 + 9
x4 = 8 5 x33 == 5 125 59 = 58 64 864 1.3565069
1 + 3 + 9 + 27
8 82 83 84
x5 = 5
1 + 3 + 9 + 27 + 81
8 82 83 84 85
x6 = 5
#
Aqui, a cada passo, utilizamos apenas as teclas [], [5], [=] e [ ].
Novamente aceitando que a sequncia escolhida converge para um
limite x diferente de zero, podemos fazer:
1
1 3 9 27 1
+ + + + " = 8 = , logo,
RPM OBMEP

8 8 2 3 4 3
1 8 5
8 8

1 3 9 27 1
+ + + +"
8 82 83 84
x=5 = 55 = 5 5.
33
Ou, como no exemplo anterior, elevando x a oitava potncia, obtemos
x = 5x3, e como x diferente de zero vem que x = 5 5 .
8

A argumentao dos exemplos apresentados foi baseada no fato de


que a sucesso x1 , x2 , x3 , " , x n , x n +1 , " converge. Isso realmente
acontece? Vamos responder a essa pergunta no caso do ltimo exemplo.

Os grficos das funes y = x8 e y = 5x3 esto mostrados na figura


abaixo e como o nmero procurado, x = 5 5 , satisfaz x8 = 5x3, vemos
que x ser a bscissa no nula do ponto de interseco dos dois grficos.
y=x 8






2 1  

5
y=5x 3
Partindo, por exemplo, de x1=1, examinemos a sucesso de pontos,
representados no grfico:
8
P1 = (1, 0); P2 = (1, 5); P3 = ( 8 5 , 5); P4 = ( 8 5 , 5 53 );
8 8 8 8 8 8 8
P5 = ( 5 53 , 5 53 ); P6 = ( 5 53 , 5 53 59 ); "

Observe que os pontos P2, P4, P6, ... pertencem ao grfico de


y = 5x3 e que os pontos P3, P5, P7, ... pertencem ao grfico de y = x8.
A sucesso de todos os pontos converge para o ponto de interseco

dos dois grficos que o ponto P = ( 5 5 , 5 58 ) , o que mostra que a

sucesso das abscissas x1, x2, x3, ..., xn, ..., converge para x = 5 5 .
RPM OBMEP

Isso mostrado rigorosamente usando-se tcnicas de Anlise Real.


Quanto a escolha x1=1, ela foi feita simplesmente para facilitar os

34
clculos e tambm por sabermos que 5 5 est prximo de 1. Se
tivssemos escolhido qualquer outro valor, por exemplo, x1=100, o limite
da sucesso x1, x2, x3, ..., xn, ... continuaria satisfazendo x8 = 5x3.

y = x 8 e y = 5x 3 esto mostrados na figura a seguir e como o

nmero procurado, x = 5 5 , satisfaz x8 = 5 x 3 , vemos que x ser


exatamente a abscissa, com x 0 , do ponto de interseco dos grficos
de y = x 8 e y = 5x 3 .
Ora, o fato algbrico de que a raiz x 8 = 5x 3 existe, equivalente ao
fato de que os grficos se cortam. Para 0 x 5 5 1,379, o grfico de
y = x 8 estar acima do grfico de y = 5x 3 .
Partindo de um valor arbitrrio, por exemplo, x1 = 1 , examinemos a
seguinte sucesso de pontos do plano cartesiano, representados no grfico

acima: P1 = (1, 0); P2 = (1, 5); P3 = (8 5 , 5); P4 = (8 5 , 5 5 );


8 3

P5 = ( 8 5 8 5 3 , 5 8 5 3 ); P6 = ( 8 5 8 5 3 , 5 8 5 3 8 5 9 ); "
Observe que os pontos P2 , P4 , P6 , " esto sobre o grfico de y = 5x 3

e que os pontos P3 , P5 , P7 ," esto sobre o grfico de y = x .


8

Os pontos P1 , P2 , P3 ,", Pn ," convergem para o ponto P = (51 5 , 58 5 ) ,


que exatamente o ponto de interseco dos dois grficos. ento bvio
que a sucesso x1 , x2 , x3 , " , x n , x n +1 , " converge exatamente para 5 5 .
Pode-se mostrar, rigorosamente, usando as tcnicas de Anlise Real, que
isso realmente acontece.
Quanto a escolha, x1 = 1, ela foi feita simplesmente para facilitar os
clculos e tambm por sabermos que 5 5 est prximo de 1. Se tivssemos
escolhido qualquer outro valor para x1 , por exemplo, x1 = 100, o limite da
RPM OBMEP

sucesso, que estamos supondo existir, continuaria satisfazendo x8 = 5x3 .

35
Voc certamente j percebeu o procedimento geral: se quisermos
calcular x = n a ou seja, achar x tal que xn = a, devemos transformar
essa igualdade de modo a obter, do lado esquerdo, um expoente que seja
uma potncia de 2. Isso feito multiplicando-se os dois membros por
uma potncia conveniente de x. Assim, por exemplo, se x = 11 a ento

x11 = a, que implica x16 = x11x5 = a x5 ou x = 16 ax 5 e utilizaremos a


16 5
sucesso x1, x2, x3, ..., xn, ..., na qual xn +1 = axn .

Adaptado dos artigos


Vamos usar a calculadora?
Hideo Kamayama e Eduardo Wagner, RPM 26.
Vamos continuar usando a calculadora
Joo Bosco Pitombeira de Carvalho, RPM 51.
RPM OBMEP

36
Uma equao interessante

H algum tempo, um amigo professor mostrou-me a


equao
3
2x 1 + 3 x 1 = 1
e fez a seguinte observao: apesar de, no decorrer da
resoluo, elevarmos as equaes somente a potncias
mpares (duas elevaes ao cubo), ainda assim,
surpreendentemente, aparece uma raiz falsa. Por qu?
Antes de mostrar como o professor resolveu a equao,
vejamos o porqu da sua surpresa.
Sabemos que x = y xn = yn, x, y R, n N,
mas a recproca desta afirmao s verdadeira se n for
mpar. Isto ,
xn = yn x = y, x, y R, se n for mpar.
fcil ver que a propriedade xn = yn x = y no vale se
n for par basta observar que 52=(5)2 e 5 5. O
que vale :
xn = yn | x | = | y |, x, y R, n N.
As falsas solues aparecem nitidamente quando,
RPM OBMEP

resolvemos equaes irracionais. Vejamos um exemplo:


Quais so as solues da equao
2x 3 = x 3 ?

37
1 2
2 x 3 = x 3 ( 2 x 3 ) 2 = ( x 3) 2 2 x 3 =
3 4
= x 2 6 x + 9 x 2 8 x + 12 = 0 x = 6 ou x = 2.
As passagens 2, 3 e 4 so equivalncias, mas a recproca da implicao
1 no verdadeira. por isso que, aps resolvermos a equao,
testamos as razes encontradas, para ver se elas, de fato, satisfazem a
equao inicial. No exemplo, 6 raiz de (2), mas 2 no .
Portanto, estamos acostumados com o aparecimento de falsas razes
na resoluo de equaes irracionais.
Mas, no exemplo que o professor apresentou, o fato de aparecer uma
raiz falsa era estranho, pois a resoluo da equao exigia apenas que
seus membros fossem elevados ao cubo e sabemos que, em R,
x3 = y3 x = y.
Vejamos como o professor resolveu a equao:
3
2x 1 + 3 x 1 = 1 . (1)
Elevando ao cubo, obtemos
2 x 1 + 3( 3 2 x 1) 2 .3 x 1 + 3 3 2 x 1.( 3 x 1) 2 + x 1 = 1 (2)
3 x 2 + 3 3 2 x 1.3 x 1( 3 2 x 1 + 3 x 1) = 1 (3)

o termo entre parnteses vale 1 ( a prpria equao (1)!)


3 x 2 + 3 3 (2 x 1)( x 1) = 1 (4)

3 x + 3 3 (2 x 1)( x 1) = 3 (5)
3
2 x 2 3x + 1 = 1 x (6)
2x2 3x + 1 = (1 x)3 (7)
2x 3x + 1 = 1 3x + 3x x
2 2 3
(8)
x x = 0.
3 2
(9)
RPM OBMEP

E, portanto, x = 0 ou x = 1.
Verifica-se, por substituio em (1), que 1 soluo, mas 0 no .

38
Onde e por que apareceu essa falsa raiz?
Sugiro que o leitor tente responder a essa pergunta antes de prosseguir.
Observe que x = 0 no soluo das equaes (1), (2) e (3), mas
soluo das equaes a partir de (4). Na verdade, (1), (2) e (3) so
equivalentes entre si (possuem o mesmo conjunto soluo), e as equaes
de (4) a (9) tambm so equivalentes entre si, mas (3) e (4) no so
equivalentes. Foi nessa passagem que fizemos algo ilcito.
O que fizemos para passar de (3) a (4)? Ora, usamos novamente a
equao (1) substituindo 3 2 x 1 + 3 x 1 por 1, e esse procedimento
no gera uma equao equivalente anterior. Tendo duas equaes
equivalentes, (1) e (3), se substituirmos uma na outra, obtemos uma
nova equao que consequncia das anteriores, mas no ,
necessariamente, equivalente a elas. Assim (3) (4), mas no vale a
recproca.
Vejamos um exemplo onde esse fato mais evidente:
x = 2 (o conjunto soluo {2}),
2 = x ( equivalente a de cima).
Substituindo uma na outra, obtemos
x = x, cujo conjunto soluo R!
Assim, o aparecimento de uma raiz falsa no est ligado ao fato de a
equao ser irracional nem s potncias que tomamos, e sim, ao
procedimento da resoluo.
Mais uma palavra sobre esse fato: o truque utilizado na passagem
de (3) para (4) til, pois limpou a equao, mas no uma
equivalncia no podemos perder de vista a equao original. Situaes
como essa so comuns, por exemplo, na trigonometria quando usamos,
numa equao, a identidade sen2 x + cos2 x = 1.
Vamos ilustrar o aparecimento de falsas razes atravs de mais dois
RPM OBMEP

exemplos:
x = 1 x (e, portanto, x = 1/2).
Se elevarmos ambos os membros ao cubo, teremos:

39
x = l x x3 = (l x)3 x3 = 1 3x + 3x2 x3
(substituindo x por 1 x)
x3 = 1 3(1 x) + 3x2 x3 2x3 3x2 3x + 2 = 0
x = 1/2; x = 1; x = 2.
Outro exemplo:
x = 1.
x = 1 (x l)2 = 0 x2 2x + 1 = 0
(substituindo x por 1)
x2 2.1 + 1 = 0 x2 = 1 x = l ou x = l.

Adaptado do artigo
Uma equao interessante
Cludio Possani, RPM 19.
RPM OBMEP

40
PAINIS

Painel I
O nmero 12

Ao nmero 12 so atribudos muitos significados, sobretudo


de ndole religiosa ou espiritual, cuja influncia provocou
alguns efeitos na organizao de nosso cotidiano.
Na historiografia judaico-crist, temos os 12 filhos de Jacob,
filho de Isaac e neto de Abrao, dos quais derivaram as 12
tribos de Israel. Refere-se ainda que Jacob usava um peitoral
sobre o qual haviam sido incrustadas 12 pedras preciosas
que so a revelao de 12 poderes csmicos. Tambm a
coroa usada na sagrao da monarquia inglesa tem 12 pedras
preciosas.
So 12 os deuses principais da mitologia grega, que vivem
no Monte Olimpo. O ano tem 12 meses. O zodaco divide a
esfera celeste em 12 casas. O relgio est dividido em 12
horas.
A bandeira da Unio Europia tem 12 estrelas douradas,
que, segundo a Comisso Europia, representam a
solidariedade e harmonia entre os povos da Europa, porque
o nmero 12 tradicionalmente um smbolo de perfeio,
de plenitude e de unio.
No consigo acreditar que os nmeros tenham algum
significado que os transcenda, porm creio que a natureza
abstrata dos nmeros propicia a sua utilizao como
representantes de significados que os transcendem.
Atribuies e interpretaes do significado do nmero 12,
RPM OBMEP

sobretudo relacionadas com questes religiosas e sociais,


decorrem do fato, do domnio aritmtico, de 12 ser o produto
de 3 por 4 e, alm disso, parece-me que o fato de 12 ter
muitos divisores, 1, 2, 3, 4, 6 e 12, pode ter ajudado na sua

41
projeo. que, entre as quatro operaes aritmticas elementares
efetuadas no conjunto dos nmeros inteiros, a diviso a nica que nem
sempre d resultados inteiros, sendo uma minoria os casos em que isso
acontece. Conjugando esse fato com a circunstncia de que a maioria
dos seres humanos se mostra mais disponvel para o clculo com inteiros
do que com outros tipos de nmeros, compreende-se a importncia dada
aos nmeros inteiros que se evidenciam por terem muitos divisores como
12, 24 e 60, por exemplo.
Mas o objetivo deste artigo mostar uma interveno do nmero 12
numa relao entre os domnios algbrico e geomtrico. Consideremos
uma funo quadrtica f(x) = ax2 + bx + c com a > 0 e = b2 4ac >
0, condio essa, como bem sabemos, que implica a existncia de duas
razes reais distintas da equao ax2 + bx + c = 0. Essas razes so as
abcissas dos pontos em que a parbola, grfico de f, intersecta o eixo x.
Consideramos o tringulo formado
pelos pontos A, B e pelo ponto V, vrtice
da parbola, como na figura. Esse tringulo
issceles, j que AV = VB. Vamos
verificar que o muito falado nmero 12
relaciona a funo quadrtica com a
possibilidade de o tringulo AVB ser
equiltero.

Como conhecido, as coordenadas dos pontos A, V e B so dadas


em funo dos coeficientes da funo quadrtica:
b b + b
A=( ; 0), B=( ; 0) e V =( ; ).
2a 2a 2a 4a
Podemos usar o teorema de Pitgoras para calcular AV:
2 2
2 b (b ( + 4)
AV = + ou AV = .
4a 2a 4a

b + b
RPM OBMEP

Por outro lado, AB = = .


2a 2a a

42
Para que AVB seja equiltero, devemos ter AB = AV, o que
equivalente a 12 = 2 ou, como > 0, equivalente a = 12.

Adaptado do artigo
O nmero 12
Carlos Grosso, RPM 67.

Painel II
Sexta-feira 13

As pessoas criaram um mito sobre as sextas-feiras 13, dizendo que


essas datas so propcias para ocorrer coisas macabras, horrveis... Parece
que nas ltimas dcadas esse mito foi bastante reforado e divulgado
pela srie de filmes Sexta-feira 13 que o cinema exibiu. Mas no so
apenas coisas ruins que esto ligadas sexta-feira 13, muito pelo
contrrio; temos um belo problema de Matemtica: todo ano h pelo
menos uma sexta-feira 13.
Para verificarmos o prometido, vamos inicialmente enumerar os dias
13 de um determinado ano. Para isso imaginemos um ano de 365 dias
(se o ano tiver 366 dias, o mesmo mtodo funciona!). Lembre que, num
ano de 365 dias, os meses de janeiro, maro, maio, julho, agosto, outubro
e dezembro tm 31 dias, enquanto abril, junho, setembro e novembro
tm 30 dias e fevereiro tem 28 dias. Assim, temos que:

1o de janeiro dia 1 13 de maro dia 72


2 de janeiro dia 2 ...
3 de janeiro dia 3 13 de abril dia 103
... ...
13 de janeiro dia 13 13 de maio dia 133
... ...
13 de fevereiro dia 44 13 de junho dia 164
RPM OBMEP

43
...
13 de julho dia 194
...
13 de agosto dia 225
...
13 de setembro dia 256
...
13 de outubro dia 286
...
13 de novembro dia 317
...
13 de dezembro dia 347

Assim, temos que os dias 13 de um determinado ano de 365 dias so


13, 44, 72, 103, 133, 164, 194, 225, 256, 286, 317 e 347, que, quando
divididos por 7 (uma semana tem sete dias), deixam restos 6, 2, 2, 5, 0, 3,
5, 1, 4, 6, 2 e 4, respectivamente. Perceba que todos os restos possveis
de uma diviso por 7 apareceram, isto , 0, 1, 2, 3, 4, 5 e 6. Assim,
perceba que:
se a primeira sexta-feira do ano for dia x (x no mximo
7), as sextas-feiras seguintes sero os dias x + 7, x + 14, x
+ 21, ... Se, por exemplo, x for 7, ento todas as sextas-
feiras do ano cairo nos dias 7, 14, 21, 28, 35, 42, ... do
referido ano. Como entre os dias 13 h um que mltiplo
de 7 (o dia 133), segue que esse dia ser uma sexta-feira 13
(isso ocorreu em 2005; veja que 13 de maio de 2005 foi
uma sexta-feira 13). Seguindo o mesmo raciocnio, se a
primeira sexta-feira do ano fosse dia 6 de janeiro, ento as
sextas-feiras seriam os dias 6, 13, 20, 27, 34, 41, ... Como
entre os dias 13 h um cujo resto da diviso por 7 6 (o
dia 13), segue que, nesse ano, 13 de janeiro seria uma sexta-
feira 13.
Esse raciocnio mostra que em qualquer ano existe pelo menos uma
sexta-feira 13. Perceba que pode haver mais de uma sexta-feira 13. Se,
por exemplo, o dia 2 de janeiro for uma sexta-feira, ento as demais
sextas-feiras desse ano sero os dias 2, 9, 16, 23, 30, 37,44, ..., ou seja,
RPM OBMEP

os dias que deixam resto 2 quando divididos por 7.


Assim, em um ano de 365 dias em que 2 de janeiro uma sexta-feira,

44
os dias 44, 72 e 317 (que divididos por 7 deixam resto 2) seriam sextas-
feiras 13. Noutras palavras, 13 de fevereiro, 13 de maro e 13 de novembro
seriam sextas-feiras 13 (que ano azarado, hein??? Prepare-se, 2009 ser
assim!).
Adaptado do artigo
Sexta-feira 13
Carlos A. Gomes, RPM 59.

Painel III
O jogo de bilhar

Estava numa pousada, no salo de jogos, observando uma partida de


bilhar. Em dado momento, apresentou-se a situao ilustrada na figura,
sendo que o jogador precisava acertar a bola cinza, mas no podia bater
na bola preta.
Para ajudar, um amigo do jogador
adotou a estratgia:
mediu, com um outro taco,
colocado apoiado na direo
perpendicular borda da mesa,
como na figura, a distncia d da
bola cinza at o ponto B, na borda
da mesa.
marcou nesse taco o ponto A tal
que a distncia BA vale d.
disse ao jogador para mirar no ponto
A e bater na bola branca.
Ao bater no ponto C, na borda da
mesa, a bola branca, no movimento
refletido, acertou a bola cinza.
RPM OBMEP

A pergunta que me ocorreu foi: Por


que deu certo? A resposta fundamenta-
se na lei fsica que afirma que, na

45
situao descrita, a medida do ngulo
de incidncia da bola, ao bater na mesa,
igual ao ngulo de reflexo. O ngulo BCA
e o ngulo de incidncia so opostos pelo
vrtice, logo tm a mesma medida ,
mostrando que a reta r a bissetriz do
ngulo DCA, sendo D um ponto da
trajetria de reflexo. Em consequncia, o simtrico de A, em relao a
r, que o ponto no qual est a bola cinza, pertence reta CD. Logo, a
trajetria de reflexo da bola branca passa pela bola cinza.
Alm disso, a estratgia adotada fornece a trajetria de menor percurso
para a bola branca atingir a bola cinza nas condies do problema. Isso
garantido pelo teorema a seguir, atribudo a Heron, matemtico de
Alexandria que viveu no primeiro sculo depois de Cristo.

Teorema de Heron
Dada uma reta r e dois pontos P e Q, no mesmo lado da reta r, o
ponto R sobre a reta r tal que a distncia PR + RQ a menor possvel
aquele em que os ngulos que os segmentos PR e RQ fazem com a
reta r so iguais.
Demonstrao do teorema
Seja Q o simtrico de Q em relao reta r. Por hiptese, a reta
r bissecciona o ngulo QRQ. Segue, por congruncia de tringulos, a
igualdade QR = QR. Seja R qualquer ponto sobre a reta r, diferente
de R.
Ento,
QR + RP = QR + RP = QP.
Mas
QP < QR + RP= QR + RP.
Logo, QR + RP < QR + RP.
RPM OBMEP

Adaptado do artigo
O jogo de bilhar
Jos Carlos Magossi, RPM 69.

46
Painel IV
Codificando e decifrando mensagens

Operaes de servios disponveis na Internet, movimentaes bancrias


e outras transaes eletrnicas necessitam da criptografia para
comunicao confidencial de dados.
A palavra criptografia tem origem grega (kripto = escondido, oculto;
grapho = grafia) e define a arte ou cincia de escrever mensagens em
cdigos, de forma que somente pessoas autorizadas possam decifr-las.
A criptografia to antiga quanto a prpria escrita; j estava presente no
sistema de escrita hieroglfica dos egpcios e os romanos utilizavam
cdigos secretos para comunicar planos de batalha. Contudo, desde
aquele tempo, seu princpio bsico continua o mesmo: encontrar uma
transformao (funo) injetiva f entre um conjunto de mensagens
escritas em um determinado alfabeto (de letras, nmeros ou outros
smbolos) para um conjunto de mensagens codificadas. O fato de f ser
inversvel a garantia de o processo ser reversvel e as mensagens
poderem ser reveladas pelos receptores. O grande desafio de um processo
criptogrfico, portanto, est em ocultar eficientemente os mecanismos
(chaves) para a inverso de f, de modo que estranhos no possam faz-
lo.

Descreveremos aqui dois exemplos elementares de processos cripto-


grficos, sendo o primeiro acessvel inclusive para alunos do ensino
fundamental.
Inicialmente, relacionamos nmeros ao alfabeto (o smbolo #
representa um espao em branco) que vamos utilizar nos modelos. Assim:
RPM OBMEP

Portanto, cifrar uma mensagem recai no problema de permutar


nmeros por meio de uma regra f. Pode-se fazer isso, de forma muito

47
prtica, por exemplo, atravs das funes afins f(x) = ax + b com a, b
inteiros, a 0, definidas no conjunto {0, 1, ..., 26}. Suponhamos que
Ana e Ivo desejem trocar mensagens sigilosas utilizando o alfabeto
escolhido. O primeiro passo a tomarem definirem a funo cifradora,
digamos f(x) = 2x 3. Assim, por exemplo,

mensagem R E V I S T A R P M
Ana associa a sequncia numrica 18 5 22 9 19 20 1 0 18 16 13

mas transmite a Ivo a seqncia numrica obtida pelas imagens de f,


isto ,
33 7 41 15 35 37 1 3 33 29 23.
1 x+3
Ao receb-la, Ivo, calculando a imagem de f ( x) = nessa
2
sequncia e utilizando a correspondncia alfabeto-numrica, obtm a
mensagem original.
Depois de os alunos dominarem o processo, seria oportuno que o
professor propusesse situaes em que um intruso tente decifrar
mensagens apoderando-se das sequncias numricas codificadas. Como
estamos utilizando funes afins, para tanto suficiente apenas duas
associaes corretas entre nmeros das sequncias original e codificada.
Admitindo conhecidas essas associaes, um exerccio interessante
para os alunos determinarem f.
O segundo mtodo criptogrfico que apresentaremos utiliza matrizes
invertveis como chaves, o que dificulta um pouco mais sua violao.
Suponhamos que Ana e Ivo combinem previamente utilizar a matriz
3 2 1 1 2
A= e sua inversa A = como chaves. Para trans-
1 1 1 3
mitir a mesma mensagem acima, Ana inicialmente monta uma matriz
mensagem M dispondo a sequncia numrica associada em coluna e
completa a posio restante com 0, ou seja, obtm
RPM OBMEP

18 22 19 1 18 13
M = .
5 9 20 0 16 0
Em seguida, codifica-a calculando,

48
3 2 18 22 19 1 18 13 64 84 97 3 86 39 .
AM = =
1 1 5 9 20 0 16 0 23 31 39 1 34 4 13
e transmite a seqncia 64 23 84 31 97 39 3 1 86 34 39 13. Para ler a
mensagem recebida, Ivo, da mesma forma, restaura a forma matricial
AM, e em seguida, com sua chave A1, pode recuperar M atravs da
identidade matricial, M = A1(AM).
Os mtodos tratados neste trabalho tem apenas carter instrutivo. Na
prtica atual so pouco utilizados pela inconvenincia de exigirem trocas
prvias de chaves entre os usurios. So, portanto, inviveis na descrio
de transaes eletrnicas nas quais um nico receptor recebe dados de
milhares de emissores, como ocorre em vendas pela Internet, transaes
bancrias e outras. Mesmo nesses casos mais complexos, a Matemtica
resolveu a trama, e desta vez, quem diria, o ramo da Teoria dos Nmeros.
O leitor interessado neste envolvente tema poder consultar a apostila
Criptografia, IC-OBMEP 2007.

Adaptado do artigo
Codificando e decifrando mensagens
Antonio Carlos Tamarozzi, RPM 45.

Painel V
Qual a relao entre os
nmeros 102.564 e 410.256?

Facilmente observamos que o segundo nmero do ttulo, 410 256,


o qudruplo do primeiro, 102 564. O que nos chama a ateno que os
algarismos desses nmeros so os mesmos, tendo sido o bastante
trasladarmos o algarismo 4 das unidades do primeiro nmero para a
esquerda, a fim de obtermos o segundo nmero.
RPM OBMEP

Admitamos agora a questo sendo apresentada sob o seguinte aspecto:


determinar um nmero inteiro positivo N, formado de n algarismos e
terminando pelo algarismo 4, tal que ao trasladarmos esse 4 (algarismo
das unidades) para a primeira posio, obtemos outro nmero que o
qudruplo desse nmero N.
49
Resoluo
Seja N = a1a2a3...an1 4 um nmero de n algarismos, n natural no
nulo. Retirando o algarismo 4 desse nmero, obtemos:
N 4
N' = = a1a2 a3 ...an 1 . [No exemplo: 10256 = (102 564 4)/10.]
10
Colocando o algarismo 4 esquerda do primeiro algarismo de N,
obtemos
N 4
N " = 4a1a2 a3 ...an 1 = 4 10n 1 + a1a2 a3 ...an 1 = 4 10n 1 + .
10
Para que N = 4N precisamos ter
N 4 4(10n 1)
4 N = 4.10n 1 + 40 N = 4.10n + N 4 N = .
10 39
Para N ser inteiro, devemos ter 39 como divisor de 10n1. O menor
valor de n que satisfaz essa condio n = 6:
4(106 1) 4(999999)
N= = = 4 25641 = 102564. N = 410256 = 4N.
39 39
Podemos mostrar que, fazendo n = 6k, k = 1, 2, 3, ..., k N*,
obtemos todos os nmeros N terminados em 4 e que satisfazem a
4(106 k 1)
condio procurada; logo, N = , k = 1,2.3,.........., k N*.
39
Exemplos
4(1012 1)
Para k = 2, obtemos N = , donde N = 4 25641025641 =
39
102564102564.
4(1018 1)
Para k = 3, obtemos N = , donde
39
N = 4 2564102564125641= 102564102564102564.
RPM OBMEP

Podemos propor problemas semelhantes ao anterior, como, por


exemplo, obter um nmero inteiro positivo N, formado por n algarismos
e terminando com o algarismo a, tal que ao trasladarmos esse a
(algarismo das unidades) para a primeira posio, temos como resultado

50
outro nmero que igual a aN. Procure obter N fazendo a = 1, 2, 3,
..., 9, verificando os resultados curiosos que sero obtidos, podendo,
em alguns casos, no haver soluo.
Adaptado do artigo
Qual a relao que existe entre os nmeros 102.564 e 410.256?
Augusto Manoel de Albuquerque Barros, RPM 63.

Painel VI
Uma demonstrao visual para
a frmula do sen(A + B)

Cada um tem a sua demonstrao favorita das importantes frmulas


de sen(A B) e cos (A B). De qualquer forma, sabido que, deduzida
uma delas, as outras podem ser obtidas por complemento, suplemento,
etc. Uma das mais simples e rpidas uma demonstrao visual, que
se baseia, na igualdade
a = b cosC + c cosB
onde a, b, c, A, B, C so os lados e
ngulos respectivos de um tringulo. A
igualdade pode ser obtida facilmente e diz
apenas que o lado a a soma (ou a
diferena, se B ou C for obtuso) das
projees ortogonais dos lados b e c sobre
o prprio a, como se v na figura ao lado.
Por outro lado, tambm bastante conhecida a lei dos senos em um
tringulo, segundo a qual:
a b c
= = = 2R
senA senB senC
onde R o raio do crculo circunscrito. Isso decorre imediatamente da
RPM OBMEP

figura da pgina seguinte.


Ento num crculo de dimetro 1, tem-se: a = senA, b = senB e
c = senC.

51
Para um tringulo inscrito nesse crculo, a
igualdade inicial fica:
senA = senB cosC + senC cosB .
E como, finalmente, o ngulo A o
suplemento de B + C, ou seja, tm o mesmo
seno, obtm-se a clebre frmula:
sen(B + C) = senB cosC + senC cosB .
a 2 a
Essa deduo vlida para B + C < 180o, senA = = 2R
R senA
o que suficiente para deduzir o caso geral.
A demonstrao anterior baseia-se numa
ideia de S.H. Kung, encontrada na revista Mathematics Magazine, vol.
64, no 2, abril de 1991.
Adaptado do artigo
Demonstraes visuais
Jos Paulo Q. Carneiro, RPM 27.

Painel VII
Valores irracionais de funes
trigonomtricas

RPM: O que segue uma transcrio adaptada de alguns resultados


encontrados no livro Nmeros: racionais e irracionais, de I. Niven, SBM,
RJ, 1984, que decidimos publicar por julgar do interesse de nossos
leitores.
So conhecidas as identidades trigonomtricas
cos2 = cos2 sen2, sen2 = 2sencos,
sen(a + b) = sena cosb + senb cosa e
cos(a + b) = cosa cosb + sena senb,
as quais, juntamente com a relao fundamental, cos2 + sen2 = 1,
RPM OBMEP

implicam
cos3 = 4cos3 3cos.
Fazendo = 20 na ltima igualdade, obtemos:

52
1
= cos 60D = 4 cos3 20D 3 cos 20D .
2
Se escrevemos x no lugar de cos20, obtemos a equao
8x3 6x 1 = 0,
que por construo tem cos20 como raiz.
Aplicando a essa equao o conhecido resultado sobre razes racionais
de equaes polinomiais:
Se p/q, frao irredutvel, raiz de uma equao com
coeficientes inteiros anxn + an1xn1 + ... + a1x + a0 = 0,
ento p divisor de a0 e q divisor de an,
temos que as nicas possveis razes racionais da equao so
1 1 1
1, , e . Mas, substituindo-se na equao, um clculo
2 4 8
simples mostra que nenhum desses nmeros raiz; logo, a equao no
tem razes racionais e, portanto, cos20 um nmero irracional.
Tambm temos cos20 = cos210o sen210o = 1 2sen210o.
Logo, se sen10 fosse racional, ento 1 2sen210o seria racional, o
que implicaria cos20 racional, o que uma contradio.
Portanto, sen10 irracional.
Usando cos20 = cos210o sen210o = 2cos210o 1, conclui-se, de
modo anlogo, que cos10 tambm irracional.
Generalizando, temos o resultado:
Se for um ngulo tal que cos2 irracional, ento cos, sen
e tg so tambm irracionais.
A verificao de que cos e sen so irracionais se faz de modo
anlogo ao utilizado para = 10, usando as igualdades
cos2 = cos2 sen2 = 1 2sen2 = 2cos2 1.
Finalmente, se tg fosse racional, ento tg2 seria racional e de
RPM OBMEP

1
1 + tg 2 = sec 2 = ,
cos 2

53
teramos cos racional e, novamente, concluiramos que cos2
racional, uma contradio. Portanto, tg irracional.
Com repetidas aplicaes do resultado anterior mostra-se que cos,
sen e tg so irracionais, para, por exemplo, os valores de :
5; 2 30; 1 15; 3730", etc.

Adaptado do artigo
Valores irracionais de funes trigonomtricas
Paulo A. da Mata Machado e Aldo Trajano Lourdo, RPM 46.

Painel VIII
Mgica com nmeros

Truques de adivinhaes aritmticas tm sido apresentados a pessoas


e alunos de vrios nveis de escolaridade e sempre causam surpresa e
fazem muito sucesso. Vamos apresentar o truque da adivinhao egpcia
com a subseqente explorao das propriedades aritmticas subjacentes
a ele.
Nesse truque o apresentador pede a um espectador que pense em um
nmero de 10 a 100. O apresentador segue ento os seguintes passos:
1. Pergunta ao espectador se o nmero par ou mpar. Ouvida a resposta,
se for par, pede ao espectador que divida o nmero por 2. Se for
mpar, pede a ele que subtraia 1 e que ento divida o resultado por
dois.
2. Pergunta se o resultado obtido par ou mpar e, ouvida a resposta,
pede ao espectador para repetir o procedimento descrito no item 1.
3. O procedimento continua com cada novo resultado at o resultado
(quociente de uma diviso por 2) tornar-se igual a 1, quando ento os
clculos do espectador terminaro.
RPM OBMEP

Quando o apresentador informado de que o resultado igual a 1,


ele revela imediatamente ao espectador o nmero pensado por ele.

54
Como funciona o truque da adivinhao egpcia
Suponhamos que o nmero pensado pelo espectador seja 52. Nas
sucessivas etapas, ele efetuar as contas da coluna abaixo esquerda,
enquanto simultaneamente o apresentador ir fazendo, secretamente, as
anotaes da coluna direita.
Aluno Professor
1
52 (nmero pensado)
2
26
4 9
13
8
6
16 9
3
32 9
1
Para cada nmero mpar informado pelo espectador, o apresentador
anota 9. Nos sucessivos estgios da brincadeira, o apresentador marca
as potncias de 2, iniciando em 20 = 1. Em seguida, o apresentador
soma as potncias de 2 correspondentes s marcas 9,
4 + 16 + 32 = 52,
e resgata o nmero que foi pensado pelo espectador!
O truque foi concebido observando o mtodo das divises sucessivas
por 2, usado para representar um inteiro positivo no sistema binrio,
isto , como soma de potncias (distintas) de 2, a partir de sua
representao no sistema decimal. Nesse mtodo, tomando como
exemplo o nmero 52, fazemos a seguinte escada de divises
sucessivas por 2, at atingirmos quociente igual a 1, quando o algoritmo
termina.
Lendo da direita para a
esquerda os 0s e 1s, que
so o ltimo quociente e os
restos das divises, obte-
mos a representao do
nmero 52 (aqui represen-
RPM OBMEP

tado no sistema decimal) no


sistema de numerao de
base 2:

55
52 = (110100)2 = 1 25 + 1 24 + 0 23 + 1 22 + 0 21 + 0 20
= 22 + 24 + 25.
Na seqncia das divises, um resto ser 0 quando o dividendo for
par, e 1 quando o dividendo for mpar, da a importncia de tomar nota
apenas das potncias de 2 correspondentes aos restos mpares.
O ttulo adivinhao egpcia inspirado nos algoritmos de
multiplicao dos antigos egpcios, baseados na decomposio de inteiros
positivos como somas de potncias distintas de 2.

Adaptado do artigo
Mgicas com nmeros
Joo C. V. Sampaio, RPM 60.

Painel IX
Destreza ou esperteza?

Certa vez, quando eu tinha 15 anos, um amigo da minha famlia


afirmou que sabia fazer contas mentalmente e com muita rapidez. Para
provar isso, props a seguinte brincadeira:
Vou escrever um nmero com sete algarismos. Em seguida, voc
escreve, abaixo do meu nmero, outro nmero com sete algarismos.
Repetimos isso mais uma vez, eu escrevo meu terceiro nmero e, ento,
eu direi a voc, sem fazer clculos, qual o valor da soma dos cinco
nmeros.
Eu, um tanto desconfiado, aceitei a proposta, ocorrendo o seguinte:
1o nmero escrito por ele: 3 574 186
1o nmero escrito por mim: 1 247 064
2o nmero escrito por ele: 8 752 935
2o nmero escrito por mim: 4 955 231
RPM OBMEP

3o nmero escrito por ele: 5 044 768


Soma fornecida por ele: 23 574 184

56
Conferi a soma manualmente e constatei que estava correta. Fiquei
atnito observando aqueles nmeros por alguns instantes, mas nada
consegui concluir. Ele props outra conta e novamente acertou o resultado
em poucos segundos. Claro que eu sabia (ou desconfiava) que existia
algum truque por trs daquilo, mas fiquei por alguns anos sem saber
qual era. Vamos agora mostrar que, na realidade, tudo no passa de um
pouquinho de lgebra: observe que o segundo e o terceiro nmeros
escritos por ele so construdos a partir do anterior, de modo que a soma
com o anterior seja igual a 9 999 999. Veja:
1o nmero escrito por mim + 2o nmero escrito por ele
1 247 064 + 8 752 935 = 9 999 999
2o nmero escrito por mim + 3o nmero escrito por ele
4 955 231 + 5 044 768 = 9 999 999
Observe agora que, como 9 999 999 = 10 000 000 1, a soma total
igual a: primeiro nmero somado + 2 (10 000 000 1) = 20 000 000
2, ou seja, (3 574 186 + 20 000 000) 2 . Para efetuar a soma entre
parnteses, observando que o nmero de zeros em 20 000 000 igual
ao nmero de dgitos do nmero inicial, basta acrescentar o dgito 2 na
frente do nmero original, o que resulta em 23 574 186. Subtraindo 2,
obtemos a soma.
Note que, para realizar a ltima operao, no caso em que o algarismo
das unidades do primeiro nmero maior do que ou igual a 2, basta
subtrair 2 do algarismo das unidades, mantendo os outros dgitos
inalterados. Se ele for 0 ou 1, ento a subtrao um pouco mais
complicada, sendo necessrio emprestar 1 do algarismo das dezenas
para depois subtrair 2. Como 10 2 = 8, isso equivalente a subtrair
1 do algarismo das dezenas e somar 8 ao algarismo das unidades, se
esse no for nulo. Se o algarismo das dezenas for nulo, ento preciso
emprestar 1 do algarismo das centenas e assim por diante.
Observe que, no caso do desafio proposto pelo amigo de minha
RPM OBMEP

famlia, o nmero inicial 3 574 186. Colocando 2 no incio, obtemos


23 574 186. Subtraindo 2 do algarismo das unidades, obtemos
23 574 184, que a soma procurada.

57
Se algum o desafiar, voc pode tentar dificultar o trabalho para o
desafiante dizendo: Quero ver se voc acerta o resultado no caso do
primeiro nmero escrito ter o algarismo das unidades menor que 2, ou
seja, igual a 0 ou 1, e o das dezenas nulo. Isso testar se ele entendeu
realmente como funciona o truque, que pode ser adaptado facilmente
para o caso de mais dgitos ou para um nmero maior de somandos.
Deixamos para o leitor esse trabalho.

Adaptado do artigo
Destreza ou esperteza?
Vanderlei Nemitz, RPM 64.

Painel X
Determinante para fatorar

H alguns anos, quando ainda existia a Unio Sovitica, submeteu-


se aos participantes de uma olimpada juvenil de Matemtica a seguinte
questo, aparentemente simples:
Fatorar a expresso a3 + b3 + c3 3abc
Mesmo bons professores de Matemtica, se no conhecerem algum
truque, tero dificuldade em resolver esse problema pelo mtodo direto.
Quem duvidar, que o tente.
Entretanto, a teoria dos determinantes d uma soluo fulminante ao
problema. Vejamos: seja o determinante
a b c
c a b = a3 + b3 + c3 abc abc abc = a3 + b3 + c3 3abc ,
b c a
exatamente a expresso que desejamos fatorar.
RPM OBMEP

O determinante no se altera se substitumos, por exemplo, a primeira


linha da matriz por sua soma com as duas outras, ou seja

58
a b c (a + b + c) (a + b + c) (a + b + c)
c a b = c a b =
b c a b c a

1 1 1
(a + b + c) c a b = (a + b + c)(a 2 + b 2 + c 2 ab ac bc) .
b c a

e o problema foi resolvido.


Muitos realmente so os caminhos da Matemtica e precisamos ter a
mente aberta e desbloqueada para encontr-los.

Adaptado do artigo
Usando determinantes para fatorar
Gilberto Garbi, RPM 41.

RPM OBMEP

59
Funes interessantes

A aplicao de situaes do cotidiano na motivao,


estudo e ensino de tpicos de contedos programticos
aumenta, na maioria da vezes, o interesse e compreenso
dos alunos da educao bsica, alm de evidenciar que a
Matemtica faz realmente parte da vida de todos ns. No
ensino de funes, que pode ser iniciado j no nvel
fundamental, as aplicaes so muito indicadas para fugir
do formalismo terico. Nessa direo, vou apresentar e
estudar alguns aspectos de funes bastante simples que
modelam situaes reais e comuns.
I. Em uma capital brasileira, os preos das corridas de
txi tiveram o seguinte aumento:
bandeirada: passou de R$ 3,20 para R$ 3,50, tendo,
portanto, um aumento de aproximadamente 9,3%;
quilmetro rodado: passou de R$ 1,80 para R$2,20,
tendo, portanto, um aumento de aproximadamente
22,2%.
A determinao da funo que fornece o preo de uma
corrida j suscita uma discusso interessante. Vrios textos
didticos apresentam funes que modelam situaes
RPM OBMEP

desse tipo como polinomiais de primeiro grau, cujo grfico


uma reta. No nosso caso seria:
P1(x) = 3,20 + x 1,80 P2(x) = 3,50 + x 2,20,

60
onde P1(x) e P2(x) denotam o preo da corrida de x km antes e depois
do aumento, respectivamente.
Essa interpretao pressupe uma variao contnua no preo da
corrida em funo dos quilmetros rodados. Mas a realidade no assim.
O taxmetro varia em fraes no valor de R$ 0,30, ou seja, supondo que
o carro no pare durante a corrida:
os valores de P1(x) variam 1,80/0,30 = 6 vezes durante cada km
rodado, o que significa a cada intervalo de rodagem de
aproximadamente 166,66 m;
os valores de P2(x) variam 2,20/0,30 = 7, 333... vezes durante cada
km rodado, o que significa a cada intervalo de rodagem de
aproximadamente 136,36 m.
Conversas com taxistas nos fizeram concluir que eles no tm em
mente o valor exato do comprimento do trecho percorrido antes de cada
mudana no preo, apenas deduzem valores aproximados (recebemos
respostas de 200 m, 150 m, etc.); dizem que quem determina o valor
exato o INMETRO ao ajustar os aparelhos dos txis.
Aqui cabe uma observao interessante: no caso da P1, o taxmetro
muda um nmero inteiro de vezes, 6, em cada km rodado, o que no
acontece na P2, uma vez que 2,20 no mltiplo de 0,30. Nesse caso,
para que o taxmetro mude um nmero inteiro de vezes, necessrio que
2, 20
x seja inteiro, isto , que x seja mltiplo de 0,30, sendo x o
0, 30
nmero de km rodados. Isso significa que a expresso afim da funo
P1 ou P2 fornece o preo exato de uma corrida de x km, se x ,
respectivamente, inteiro ou inteiro mltiplo de 3.
Voltemos ento s funes, P1 e P2 reais, que mudam de valor
aos saltos, a cada intervalo de 166,66 m ou de 136,36 m. Seus grficos
tm a forma de escada, um exemplo no usual de funo . Esboamos,
tambm, os grficos das P1 e P2 afins.
RPM OBMEP

61
Perguntas
1. Quais os preos, antigo e depois do aumento, de uma corrida de
3,5 km = 3500 m?
Como 3500/166,6 um valor entre 21 e 22, vemos que o preo
antigo dado pelo 22o degrau do grfico da funo P1(x), ento
P1(3500) = 3,20 + 21 0,30 = 9,50, ou seja, o preo R$ 9,50. Um
clculo anlogo mostra que o preo novo dessa corrida seria R$ 11,00.
Vamos responder s perguntas a seguir, considerando as aproximaes
de P1(x) e P2(x) pelas funes afins anteriormente consideradas.
Isso permite estabelecer expresses algbricas simples para as funes
envolvidas, alm do que os grficos acima mostram que a funo
afim uma aproximao razoavelmente boa.
2. Qual ser o aumento percentual no preo de uma corrida de 10 km?
Considerando P1(10) = 3,20 + 10 1,80 = 21,20 e
P2(10) = 3,50 + 10 2,20 = 25,50, vemos que o aumento percentual
de 20,28%.
3. Qual a funo que fornece o aumento percentual numa corrida de
RPM OBMEP

x km?
Considerando as funes afins, queremos, em funo de x, o valor de
p
p tal que P2 ( x) = P1 ( x) + P1 ( x) , sendo P1(x) = 3,20 + x 1,80 e
100
62
P2(x) = 3,50 + x 2,20. Substituindo os valores e fazendo os clculos,
30 + 40 x
obtemos p ( x) = . Como x > 0, temos que o domnio
3, 20 + 1, 80 x
dessa funo o intervalo [0, +]. interessante observar que, para
x = 0, o aumento igual a 30/3,20, que aproximadamente 9,3%,
aumento da bandeirada. Para valores de x muito grandes, observando
30
+ 40
que p ( x) = x , vemos que p tende para p = 40/1,80 =
3, 20
+ 1, 80
x
22,22222... que o aumento percentual do km rodado, isto , para
corridas muito grandes, o aumento da bandeirada no conta, valendo
apenas o aumento do km rodado. O grfico da funo p(x), a seguir,
ilustra esse resultado e tambm evidencia uma peculiaridade dos
taxistas: eles no tm como receber aumentos de um percentual fixo.

II. O governo de um Estado brasileiro mudou a contribuio


previdenciria de seus contribuintes: de 6% sobre qualquer salrio
passou para 11% sobre o que excede R$ 1200,00 nos salrios. Por
exemplo, sobre um salrio de R$ 1700,00, a contribuio anterior
era
0,06 R$1700,00 = R$ 102,00 e a atual
RPM OBMEP

0,11 (R$ 1700,00 R$ 1200,00) = R$ 55,00.


Provavelmente os alunos no tero dificuldades em determinar as
funes que fornecem o valor das contribuies em funo do valor x

63
do salrio. Sendo C1(x) a contribuio anterior e C2(x) a atual, temos:

6
C1 ( x) = x = 0, 06 x
100
0 se 0 x < 1200

C2 ( x) = 11 .
( x 1200) se x 1200
100

Os grficos dessas funes esto esboados a seguir e uma anlise


deles permite tirar vrias concluses, por exemplo:
1. Para um salrio de, aproximadamente, R$ 2700,00, o valor da
contribuio permanece o mesmo, por volta de R$ 160,00. Para obter
o valor exato do salrio que mantm a contribuio, basta resolver a
equao 0,06x = 0,11(x 1200), chegando a x = 2640 e C1(2640) =
C2(2640) = 158,40.
2. Para salrios abaixo de R$ 2640,00, a contribuio previdenciria
diminuiu, pois nesse caso temos, para um mesmo x, C2(x) menor do
que C1(x). Fica interessante fazer simulaes com salrios e populao
para calcular os valores das arrecadaes antes e depois da mudana
da lei, verificando que em determinadas situaes, bastante provveis,
a arrecadao estadual diminui consideravelmente.
RPM OBMEP

64
3. C2(x) maior que C1(x) para salrios maiores que R$ 2640,00, logo a
nova lei aumenta a contribuio dos salrios maiores que esse valor.
4. A inclinao da reta do grfico de C2(x), x > 1200, maior que a da
reta de C1(x); logo, a contribuio, com a nova lei, aumenta mais
rapidamente do que antes, medida que o salrio aumenta.

Adaptado do artigo
Funes interessantes
Ana Catarina P. Hellmeister, RPM 63.

RPM OBMEP

65
A formiga inteligente

Um problema
Imagine dois postes verticais AA e BB de tamanhos
diferentes no plano horizontal . Para que posies uma
formiga P, no plano, v os dois postes do mesmo
tamanho?

Em primeiro lugar, devemos pensar o que ocorre quando


vemos dois objetos com o mesmo tamanho. Por exemplo,
uma moedinha de 1 centavo segura entre os dedos com o
brao esticado tem, aparentemente, o mesmo tamanho
da lua cheia. A concluso a seguinte: dois objetos
aparentam ter o mesmo tamanho para certo observador,
quando os ngulos de visada so iguais. Portanto,
observando a figura acima, a formiga v os postes AA e
RPM OBMEP

BB do mesmo tamanho se os ngulos de visada APA


e BPB forem iguais.
Mesmo sem pensar ainda como resolver o problema,
a formiga inteligente pode verificar que existem dois
66
lugares onde isso acontece, ambos na reta AB. Andando na reta AB,
de A para B, certamente encontraremos um ponto interior ao segmento
AB, onde APA = BPB, como mostra a figura.

Se o poste A for maior que o poste B, esse ponto P estar obviamente


mais prximo de B do que de A e, para obter exatamente a posio de
P, basta ligar o ponto A ao simtrico de B em relao reta AB.
Por outro lado, uma segunda posio para P a interseo da reta
AB com a reta AB. A figura a seguir mostra uma outra posio de P,
onde APA = BPB.

Alm dessas duas posies determinadas intuitivamente, existe


certamente uma infinidade de outras, no plano, mas fora da reta AB. J
sabemos que os ngulos de visada APA e BPB so iguais para que a
formiga veja os dois postes do mesmo tamanho. Porm, como os postes
so verticais, isso significa que os tringulos APA e BPB so
PA AA
semelhantes. Portanto, = que constante, pois a razo entre
PB BB
os comprimentos dos dois postes (veja novamente a primeira figura).
Temos ento dois pontos fixos A e B no plano e buscamos o lugar
RPM OBMEP

geomtrico dos pontos cuja razo das distncias a esses pontos constante
e igual razo entre os comprimentos dos postes.

67
Antes de resolver o problema, interessante recordar o teorema das
bissetrizes:
Uma bissetriz de um ngulo de um tringulo divide o lado
oposto na mesma razo dos lados adjacentes.

Veja a seguir uma demonstrao desse importante teorema.

a) Na figura abaixo, AD bissetriz interna do ngulo A do ABC.


Traando por D as perpendiculares
DM e DN aos lados AB e AC, temos
que DM = DN, pois o ponto D est na
bissetriz do ngulo A. Por outro lado,
como os tringulos ADB e ADC tm
mesma altura a partir de A, ento a razo
entre suas reas, A, igual razo entre
suas bases, ou seja,
DB A( ADB ) ( AB DM ) / 2 AB
= = = .
DC A( ADC ) ( AC DN ) / 2 AC

b) Na figura a seguir, AE bissetriz externa do ngulo A do ABC.

Traando por E as perpendiculares EP e EQ s retas AB e AC,


temos que EP = EQ, pois o ponto D est na bissetriz do ngulo
externo A. Por outro lado, como os tringulos AEB e AEC tm mesma
altura a partir de A, ento a razo entre suas reas, A, igual razo
RPM OBMEP

entre suas bases, ou seja,


EB A( AEB ) ( AB EP ) / 2 AB
= = = .
EC A( AEC ) ( AC EQ) / 2 AC

68
Demonstrado o teorema das bissetrizes, importante lembrar que
vale a sua recproca, ou seja, se D um ponto da base BC do ABC
DB AB
e = , ento AD bissetriz interna do ngulo A e, se E um
DC AC
EB AB
ponto do prolongamento de BC e = , ento AE bissetriz
EC AC
externa.

A circunferncia de Apolnio
Passamos agora a analisar o problema seguinte:
Dados dois pontos A e B no plano e um nmero k > 0, determinar
PA
o lugar geomtrico do ponto P tal que = k.
PB
Em primeiro lugar, se k = 1, temos PA = PB e o lugar geomtrico de
P naturalmente a mediatriz de AB.
Em seguida, vamos imaginar k > 1 (o caso 0 < k < 1 inteiramente
anlogo). Como j vimos antes, podemos encontrar com alguma
facilidade dois pontos da reta AB que possuem a propriedade desejada.
MA NA
Sejam portanto M e N pontos da reta AB tais que = = k.
MB NB
Observe ento a figura a seguir em que as retas r e s so paralelas. RPM OBMEP

Sabemos que, dado um segmento unitrio, para cada real positivo k


existe um segmento de comprimento k e a semelhana de tringulos

69
MA NA
permite concluir que, na figura acima, temos = = k . Ateno:
MB NB
a figura acima mostra que os pontos M e N existem, mas a construo
com rgua e compasso s possvel se k construtvel (racionais, por
exemplo, so construtveis). Na situao acima, dizemos que os pontos
M e N dividem harmonicamente o segmento AB.

Vamos agora considerar um ponto P fora da reta AB tal que PA = k


PB
e investigar seu lugar geomtrico.
Observe a figura a seguir.

Como MA = PA , ento PM bissetriz interna do ngulo APB e


MB PB
NA PA
como = , ento PN bissetriz externa. Mas essas bissetrizes
NB PB
so perpendiculares (verifique!) e, como M e N so fixos, o lugar
geomtrico de P a circunferncia de dimetro MN.
RPM OBMEP

Essa circunferncia chama-se circunferncia de Apolnio do segmento


AB na razo k. Ela o lugar geomtrico dos pontos cuja razo das
distncias a dois pontos fixos igual a uma constante dada.

70
O problema da formiga est resolvido. O lugar geomtrico dos pontos
de onde a formiga v os postes de mesmo tamanho a circunferncia de
Apolnio do segmento AB na razo AA/BB.

Nota
Apolnio de Perga viveu no sculo 3 a.C. Foi clebre gemetra e
astrnomo, mas a maior parte de sua vasta obra desapareceu. Felizmente,
a sua obra-prima As Cnicas foi quase toda preservada. Entretanto,
so conhecidos os ttulos e contedos dos muitos tratados que escreveu
devido a relatos de matemticos posteriores. Sabemos por isso que
Apolnio escreveu um livro chamado Lugares Planos dedicado anlise
de diversos lugares geomtricos e que um deles era justamente o lugar
geomtrico dos pontos cuja razo das distncias a dois pontos fixos
constante. Esse lugar geomtrico ficou conhecido at hoje como
Circunferncia de Apolnio, um tanto injustamente, pois Aristteles j
o tinha descoberto anos antes.

Adaptado do artigo
A formiga inteligente
Eduardo Wagner, RPM 61.
RPM OBMEP

71
A demonstrao feita
por Heron

Quando pequeno, li sobre Heron de Alexandria em uma


enciclopdia biogrfica que havia em casa. Fiquei
sabendo que ele viveu no sculo II d.C. na cidade de
Alexandria (obviamente), que foi engenheiro e
matemtico. No me lembro que outras coisas mais havia
sobre Heron, mas ficou gravada em minha memria a
frmula que l estava para calcular a rea de um tringulo:
A= p ( p a )( p b)( p c) ,
sendo p a metade do permetro do tringulo.
O que me encantou nessa frmula? No sei. Talvez por
ter uma raiz quadrada, que naqueles dias escolares lhe
dava um ar de Matemtica superior; ou pelo fato de s
usar os lados do tringulo, e no a altura, como na
formulinha usada na escola.
Anos mais tarde, aps ter encontrado vrias vezes a
frmula e at depois de ter visto sua demonstrao como
mero corolrio de um clculo de medianas, continuava
intrigado: como Heron a havia demonstrado?
Este ano comprei o livro Introduo Histria da
RPM OBMEP

Matemtica, de Howard Eves, e qual no foi minha


surpresa ao encontrar no livro a meno de que, a
demonstrao feita por Heron (que est em seu livro A

72
mtrica) estava esquematizada num dos exerccios do livro. Com algumas
pequenas modificaes aqui vai ela:

BC = a
AC = b
AB = c
a+b+c
p=
2

1. rea ABC = rea ABI + rea IBC + rea AIC =


r
( AB + BC + CA) = rp , sendo r o raio da circunferncia inscrita.
2
2. Como ADI AIF, DBI IBE e FIC IEC, temos
AD = AF, DB = BE e CE = CF.
3. Seja J o ponto da semi-reta AB tal que BJ = CE.
AD + AF BD + BE CE + CF AB + BC + CA
AJ = + + = = p.
2 2 2 2
Ento
p c = AJ AB = BJ; p b = AJ AC = DB e
p a = AJ BC = AD.
4.
RPM OBMEP

i) Seja K o ponto construdo como indicado na figura. O quadriltero


AKBI inscritvel numa circunferncia de dimetro AK; logo
AIB + AKB = 180 o e, como + + = 180, temos
AIB + CIE = 180o, de onde AKB = CIE = .
73
Ento temos CIE AKB, o que implica AB = CE = BJ .
BK r r
ii) No tringulo retngulo ALI temos r2 = DL.AD e de DLI
BK r
BLK (verifique) temos = .
LB DL

iii) De i) e ii) temos AB = LB , o que implica AB + BJ = LB + DL


BJ DL BJ DL
AJ AJ DB AD , que juntamente com r2 = DL.AD leva a
ou . = .
BJ AJ DL AD
AJ2.r2 = BJ.AJ.BD.AD.
Usando-se as igualdades apresentadas em 3, obtemos
p2r2 = (p c)p(p b)(p a),
que, pela igualdade exibida em 1, demonstra a frmula.

Adaptado do artigo
A demonstrao feita por Heron
Mrio Dalcin, RPM 36.
RPM OBMEP

74
A Matemtica da folha
de papel A4

Introduo
O formato do papel que usamos rotineiramente nos
servios de impresso ou fotocpia possui uma histria
fascinante e repleta de Matemtica. Neste artigo,
compartilho com o leitor algumas idias que esto por
trs dessa histria.

A intrigante folha de papel A4


O formato de papel mais usado para impresses e
fotocpias, que recebe a denominao A4, tem
210 milmetros de altura por 297 milmetros de largura.
Diferentemente do que se possa imaginar, a razo
297/210 no a razo urea (ver artigo Retngulo ureo,
diviso urea e sequnci de Fibonacci). Analisaremos, a
seguir, de onde vem essas estranhas medidas.
Inicialmente padronizaremos neste artigo que as palavras
largura e altura sempre sero usadas como referncia
ao maior e ao menor lado de um retngulo, respecti-
vamente.
Imagine-se tendo que resolver o seguinte problema: qual
RPM OBMEP

deve ser a largura e a altura de uma folha retangular de


modo que, quando ela for dividida ao meio, os dois novos
retngulos obtidos mantenham a razo entre altura e
largura da folha original?

75
O problema de soluo simples, como se v a seguir:

L A
= L2 = 2 A2 L = 2 A
A L
2

Portanto, a folha retangular com razo L/A igual a 2 a nica


que, quando dividida ao meio, conforme processo descrito, resultar em
retngulos semelhantes ao da folha original. Lembramos que de forma
diferente dos tringulos, onde bastam ngulos congruentes para que sejam
figuras semelhantes, no caso dos quadrilteros a semelhana s se garante
se os ngulos forem congruentes e se a razo entre os lados das figuras
for preservada. No caso das medidas de uma folha A4, note que
297/210 uma tima aproximao racional para 2 , com erro muito
pequeno, da ordem de centsimo de milsimo.
A classificao de papis da qual A4 faz parte chama-se srie A,
que comea com o A0 e vai at o A10. Essas folhas tm em comum a
razo 2 entre largura e altura. A srie comea com uma folha retangular
de rea 1 m, definida como A0. A partir dela obtemos a folha do
formato seguinte, A1, dividindo-se A0 ao meio. As dimenses da
folha A0, em metros, podem ser obtidas a partir da soluo do seguinte
sistema de equaes:
1 1
L = 2 A 4
8
A = e L = 4 2 ou A = 2 4 e L = 2 4 .
L A = 1 2

Passando essas medidas para milmetros, e aproximando para o


milmetro mais prximo, encontramos as dimenses da folha A0, que
so 841 mm de altura por 1189 mm de largura.
Faamos agora os clculos da folha A1, que obtida a partir da
RPM OBMEP

diviso ao meio da folha A0:

76
L = 2A 3 1

1 A=2 e L=2 4.
4

L A =
2
Adota-se, nesse caso, a aproximao 594 mm por 841 mm.
Dividindo-se A1 ao meio, obtemos A2, que dividida ao meio
resultar A3, e assim por diante at A10. Pode-se verificar de maneira
simples que a altura e a largura de uma folha A(k), em metros, sero
1+ 2 k 1 2 k

dadas, respectivamente, por 2 4 e 2 4 . Para o caso da folha
A4, aplicando k = 4 na frmula, obtemos os misteriosos valores
padronizados do formato, que so 210 mm por 297 mm.

Qual a vantagem da proporo 1: 2 ?


A literatura sobre artes grficas cita dois aspectos importantes sobre
a convenincia do uso de uma folha retangular de razo 1 : 2 . As p-
ginas de um livro so impressas em uma folha de mquina de grande
formato. Nela so feitas dobras e cortes e, a partir disso, so montados
os cadernos que, juntos, compem o livro. Normalmente as dobras so
feitas ao meio, fazendo com que o nmero de pginas seja uma potncia
de 2. Se o papel for dobrado ao meio por uma dobra, resultar em 2
folhas (chamado in-flio) que, quando impressas frente e verso,
constituiro 4 pginas do livro. Se essa ltima folha for novamente
dobrada ao meio, agora com dobras cruzadas, resultar em 4 folhas
(in-quarto), ou seja, 8 pginas de livro. Com uma nova dobra teremos
o in-oitavo: 3 dobras, 8 folhas e 16 pginas de livro; e assim
sucessivamente.
Uma vez que cada formato deriva do seu precedente fazendo uma
dobra sobre o maior lado do retngulo, a razo inicial 1 : 2 sempre
ser mantida em todas as pginas do livro, seja qual for o nmero de
dobras feitas na composio. Outros formatos no permitiriam isso como,
RPM OBMEP

por exemplo, um retngulo de razo 3:4 (tambm usado na confeco


de livros) que obedece a um padro de alternncia no decorrer das
sucessivas dobras. A primeira dobra gera retngulos de razo 2:3; a

77
segunda gera retngulos 3:4, a terceira retngulos 2:3, e assim suces-
sivamente. Deixo por conta do leitor a demonstrao do resultado: dada
y
a razo x:y, se x , ento as razes se alternam entre x:y e y:2x no
2
decorrer das sucessivas dobras que dividem o lado maior do retngulo
ao meio (obs.: o nico caso em que no h alternncia ser quando
x y
= , que justamente o caso em que temos a razo 1 : 2 ).
y 2x
Vale citar que nem todos os estudiosos de composio em artes
grficas esto de acordo sobre a relevncia da vantagem que acabamos
de descrever da razo 1 : 2 sobre outras razes. Para um bom acaba-
mento final das dobras de um livro recomenda-se que as dobras sejam
feitas paralelamente s fibras do papel. Com isso, folhas de papel que,
em virtude da direo das fibras, so adequadas ao in-quarto no
poderiam ser usadas para livros in-oitavo porque a fibra correria em
direo errada. Portanto, a vantagem da razo preservada em 1 : 2
aps as dobras fica comprometida quando levamos em considerao a
direo das fibras [1].
Outra vantagem que os papis de razo 1 : 2 da srie A apresentam
e essa aceita por todos os especialistas a de que evitam o desperdcio
de papel nos trabalhos de fotocpias.
Imagine que voc queira copiar duas folhas quadradas, juntas, em
uma nova folha quadrada. Essa tarefa no pode ser realizada sem o
desperdcio de papel. Se os quadrados tm lado 10 cm, lado a lado
formaro um retngulo de 10 por 20 cm, o que exigir uma folha
quadrada de 20 por 20 cm para que o servio seja feito. Nesse caso,
haver desperdcio de metade da folha. O mesmo no ocorre, por
exemplo, com duas folhas A4 lado a lado, que podem ser copiadas,
sem desperdcio de papel, em uma folha A3.
Se voc observar com ateno, as fotocopiadoras que fazem
RPM OBMEP

ampliao e reduo a partir das folhas da srie A possuem alguns


comandos pr-definidos, como, por exemplo, os de reduo de 71%,
50%, 35%, 25%, 18% e 12,5%.

78
Voc j se perguntou de onde vm essas estranhas porcentagens?
Responderemos essa pergunta calculando qual deve ser o fator de reduo
usado na altura e na largura de uma folha A(k) para que ela seja reduzida
a uma folha A(k +1):

2
Como 0, 71 , uma reduo de 71% far o servio desejado. As
2
demais redues indicadas referem-se, respectivamente, s redues de
A(k) para A(k + 2), A(k + 3), A(k + 4), A(k + 5) e A(k + 6).

Outros formatos de papel: as sries B e C


H registros do uso da razo 1 : 2 durante a Alta Idade Mdia,
quando muitos livros eram escritos em duas colunas. Gutenberg
(1398-1468), porm, preferia para suas pginas a razo 2:3, e, durante a
Renascena, raramente se produziu livro na razo 1 : 2 .
A idia de se padronizar um formato de papel surge no sculo XX, e
tem a ver com aspectos relacionados praticidade e economia. Com o
uso generalizado de um formato padro de papel o que se reflete
diretamente na padronizao dos formatos de livros, revistas, jornais,
envelopes , as bibliotecas podem planejar de forma mais eficiente as
alturas de suas prateleiras, as grficas podem trabalhar com ajustes de
mquina pr-definidos, as fotocopiadoras e impressoras podem
padronizar programas para reduo e ampliao, etc.
O padro internacional para o tamanho de papis o ISO 216
RPM OBMEP

(International Organization for Standartization, norma 216), que


adotado por todos os pases industrializados do mundo, exceto EUA,
Canad e partes do Mxico. Essa norma regulamenta o formato de
algumas sries bsicas de papel, como as sries A, B e C. As sries B

79
e C destinam-se, entre outras aplicaes, aos formatos de envelopes
que podem ser usados para conter folhas da srie A. O formato de uma
folha B(k) definido como a mdia geomtrica entre A(k) e A(k 1),
e o da folha C(k) como a mdia geomtrica entre A(k) e B(k). Usando
a frmula que vimos anteriormente para altura de uma folha A(k), as
frmulas de clculo da altura das folhas B(k) e C(k) sero:

1+ 2 k 1+ 2 ( k 1) k

k
B (k ) = A(k ) A(k 1) = 2 4 4 = 2 = 22

1+ 2 k k 1+ 4 k 1+ 4 k

C (k ) = A(k ) B (k ) = 2 4 2 2 = 2 4 =2 8

Ficam a cargo do leitor a formulao de B(k) e C(k) para a largura


das folhas dessas duas sries, bem como a demonstrao de que tambm
nas sries B e C a razo 1 : 2 se preserva.
Seja qual for o nmero k da srie, sempre teremos, tanto para a
altura quanto para a largura, a relao A(k) < C(k) < B(k). Verificaremos
tal fato para a altura, cujos dados j foram calculados anteriormente:
1+ 2 k 1+ 4 k k
2 + 4k 1 + 4k 4k
2 4 <2 8 <2 2 < < 2 < 1 < 0 ,
8 8 8
para qualquer k.
Demonstrao anloga pode ser feita entre as larguras das trs sries.
Os formatos das sries B e C so maiores que os da srie A e, por
esse motivo, so usados nos envelopes que devero conter folhas da
srie A. Como A(k) < C(k) < B(k), se queremos enviar pelo correio um
documento com poucas folhas A4, devemos usar um envelope C4, porm,
se a quantidade de folhas for muito grande, provvel que elas fiquem
melhor acomodadas em um envelope B4. Se voc quiser enviar uma
folha A4 dobrada uma nica vez, recomenda-se um envelope C5. Para
RPM OBMEP

uma folha A4 com duas dobras cruzadas, o envelope ideal o C6 e, se as


duas dobras forem paralelas, o envelope ideal o DL (ilustrado na figura
a seguir).

80
Adaptado do artigo
A Matemtica da folha de papel A4
Jos Luiz Pastore Mello, RPM 66.

RPM OBMEP

81
Retngulo ureo, diviso urea
e sequncia de Fibonacci

O retngulo ureo
Chama-se retngulo ureo qualquer retngulo ABCD
(figura 1) com a seguinte propriedade: se dele
suprimirmos um quadrado, como ABFE, o retngulo
restante, CDEF, ser semelhante ao retngulo original.
B a F b C
Se a + b e a so os compri-
mentos dos lados do retngulo
a
original a definio acima se
a b
traduz na relao = . (1) A E D
a+b a figura 1

Como veremos logo adiante, esse tipo de retngulo tem


muitas propriedades interessantes que justificam o
qualificativo ureo. Ele tem sido considerado por
RPM OBMEP

figura 2
arquitetos e artistas como o retngulo mais bem
proporcionado e de grande valor esttico. A figura 2
82
reproduz a foto de uma residncia suburbana de Paris, projetada pelo
famoso arquiteto Le Corbusier, na qual ele utiliza o retngulo ureo. H
a dois retngulos ureos, um deles representado pelo corpo inteiro da
casa e o outro, disposto verticalmente, representado pela parte da casa
esquerda da escada.

figura 3
O Partenon (figura 3), ou templo da deusa Atena, uma das mais
admiradas obras da arquitetura universal, revela, em seu frontispcio
(figura 4) um quase exato retngulo ureo. Todavia no h evidencia
histrica de que, ao construir o templo no 5o sculo a.C., os arquiteto de
Pricles tenham conscientemente usado o retngulo ureo.

figura 4
Voltemos relao (1). Dela decorre, por uma propriedade bem
conhecida das propores, que:
a b a b b a b
= = ou seja = .
a + b a ( a + b) a a b
a b
Isto significa que se o retngulo de
RPM OBMEP

lados a + b e a ureo, ento tambm a 2b  a


o o retngulo de lados a e b. ab

figura 5

83
Evidentemente o mesmo raciocnio se aplica para mostrar que tambm
so ureos os retngulos de lados b e a b, a b e 2b a, etc.
(figura 5). Em outras palavras, dados os nmeros positivos a e b,
satisfazendo a relao (1), formemos a seqncia a + b, a, b, a b,
2b a, 2a 3b, 5b 3a, 5a 8b, 13b 8a, ... (2)
Sendo, a partir do terceiro, an = an2 an1.
O raciocnio anterior estabelece que quaisquer dois elementos
consecutivos dessa sequncia so os lados de um retngulo ureo.
Portanto, o processo anterior de retirar quadrados de retngulos ureos
conduz a uma sequncia infinita de retngulos ureos, com dimenses
cada vez menores.
A figura tambm sugere que a seqncia (2) tende a zero, e isso
verdade porque as dimenses de cada retngulo da seqncia podem ser
obtidas multiplicando as dimenses correspondentes do retngulo
anterior por b/a, que menor que 1; ora, o termo geral de uma progresso
geomtrica de razo menor do que 1 tende a zero quando o nmero de
termos tende a infinito. Deve ser notado que o smbolo da Sociedade
Brasileira de Matemtica utiliza a mencionada sucesso de retngulos
ureos, unidas por quadrantes de circunferncias.

Os lados de um retngulo ureo so grandezas incomensurveis. (Veja


a definio desse e de outros conceitos correlatos no artigo Grandezas
Incomensurveis, desta apostila.) De fato, se fossem comensurveis,
teriam um submltiplo comum s, e, com referencia figura 1,
AD = (a + b)s e AB = as,
onde a e b seriam ento nmeros inteiros. Em consequncia, todos os
nmeros da sequncia (2) seriam inteiros e positivos. Isso um absurdo,
RPM OBMEP

pois no existe sequncia infinita e decrescente de nmeros inteiros


positivos (Princpio da Descida Infinita de Fermat). Conclumos, ento,
que os lados de um retngulo ureo so incomensurveis.

84
A diviso urea
O retngulo ureo est intimamente ligado com a chamada diviso
urea de um segmento, ou diviso em mdia e extrema razo, que
introduziremos a seguir.
Diz-se que um ponto C de um segmento AB (figura 6) divide esse
AC CB
segmento em mdia e extrema razo se = . (3)
AB AC

A a C b B

figura 6
A relao (3) precisamente a relao (1) se pusermos AC = a e
CB = b, de sorte que os segmentos AC e CB da diviso urea (ou
AB = a + b e AC = a) so os lados de um retngulo ureo.
interessante notar que se C1 divide AB em mdia e extrema
razo, e se marcarmos no segmento AB os pontos C2, C3, C4,... de tal
maneira que AC2 = C1B, AC3 = C2C1, AC4 = C3C2, ... (figura 7), ento
Cn divide ACn1 em mdia e extrema razo n = 2, 3, 4,... . Esse
resultado segue facilmente do que j provamos antes sobre a sequncia
infinita de retngulos ureos, donde segue tambm que os segmentos
AC1 e C1B da diviso urea de AB so incomensurveis. Sugerimos
que o leitor faa uma demonstrao completa destes resultados.

A C4 C3 C2 C1 B

figura 7
Como j observamos h pouco, as relaes (1) e (3) so idnticas
quando pomos AC = a e CB = b. Delas segue-se que
b2 + ab = a2. (4)
O nmero m = b/a conhecido como a razo urea. Dividindo a
equao anterior por a2 obtemos: m2 + m = 1. (5)
RPM OBMEP

A raiz positiva dessa equao do segundo grau :

5 1
M= 0, 618 . (6)
2
85
A razo urea, como foi definida, a razo entre o menor e o maior
lados de um retngulo ureo. Deve ser observado que muitos autores
usam como razo urea (ou nmero ureo) o seu inverso u, que a
razo entre o maior e o menor lados de um retngulo ureo:
1 5 +1
u= = 1, 618 .
m 2
O nmero u a raiz positiva da equao u2 = u + 1.
Entre estes dois nmeros, existem as relaes: u = 1 + m e mu = 1.

Construes geomtricas
Vamos construir um retngulo ureo a partir de seu menor lado
AE = a (figura 8). Para isso construmos EF = AE perpendicularmente
a AE. Com centro em G, ponto mdio do segmento AE, traamos o

B F C

a
A H E D
a/2 a/2 b b ab b
A G E D
figura 8 figura 9

p , com D na reta AE e E interno ao segmento AD. Como


arco FD
GF = GD = b + a/2, o teorema de Pitgoras aplicado ao triangulo retngulo
GEF nos d:
a a
(b + ) 2 = a 2 + ( ) 2 .
2 2
Simplificando, obtemos daqui a relao (4) que, como vimos, equivale
relao (1). Logo ABCD um retngulo ureo.
Se o problema fosse dividir o segmento AE = EF em mdia e extrema
razo, bastaria completar a construo anterior marcando, no segmento
RPM OBMEP

AE, o ponto H tal que AH = b (figura 9).

86
A sequncia de Fibonacci e a razo urea
surpreendente que a razo urea esteja intimamente relacionada
com a chamada sequncia de Fibonacci, como veremos a seguir, pois
aparentemente uma coisa nada tem a ver com a outra.
Leonardo de Pisa, muito conhecido como Fibonacci (filho de
Bonaccio), viveu no perodo de aproximadamente 1170 a 1250. Ele foi
educado na frica e viajou muito pela Europa e sia Menor. Tornou-se
famoso por conhecer muito bem toda a Matemtica ento acumulada.
Em 1202 ele publicou o Liber Abaci, ou Livro do Clculo, que teve
importncia decisiva na tarefa de tornar conhecida na Europa a
Matemtica dos rabes e hindus. Foi esse livro que popularizou no
Ocidente o uso dos algarismos arbicos e os mtodos hindus de clculo
com nmeros inteiros, fraes e razes.
A seqncia de Fibonacci aparece num dos problemas tratados no
Liber Abaci e que consiste no seguinte:
Um casal de coelhos torna-se produtivo aps dois meses de vida e, a
partir de ento, produz um novo casal a cada ms. Comeando com um
nico casal de coelhos recm-nascidos, quantos casais existiro ao final
de um ano?
Vamos designar com fn o nmero de casais de coelhos existentes
aps n meses. Evidentemente, f0 = f1 = 1. Por outro lado, o nmero de
casais existentes no n-simo ms, fn, igual ao numero existente um
ms antes, fn1, mais o numero de nascimentos novos. Ora, esse nmero
precisamente o nmero de casais existentes h dois meses, fn2, que
tm pelo menos dois meses de vida, portanto em condies de reproduzir.
Ento, cada elemento da sequncia de Fibonacci a soma dos dois
precedentes. Como j sabemos que f0 = f1 = 1, podemos construir toda
a sequncia:
f0 = 1, f1 = 1, f2 = f0 + f1 = 2,
f3 = f1 + f2 = 3, f4 = f2 + f3 = 5,
RPM OBMEP

f5 = f3 + f4 = 8, f6 = f4 + f5 = 13, ...
ou seja,
1, 1, 2, 3, 5, 8, 13, 21, 34, 55, 89, 144,...

87
Que relao pode existir entre esta sequncia e a razo urea?
Aparentemente nenhuma. No entanto, vamos demonstrar agora para
surpresa de todos ns! ... que a razo urea m o limite, com n tende
a infinito, da razo fn1/fn, isto ,
5 1 f
m= = lim n 1 . (7)
2 n f
n

De fato, pode-se observar (e demonstra-se por induo) que os


coeficientes de a e tambm os de b na sequncia (2), a partir do quarto
termo, a menos de sinal, so os nmeros de Fibonacci, isto , se
a2 = a b, a3 = a + 2b, a4 = 2a 3b, ... ento
an = (1)n(fn2a fn1b). (8)
Lembrando que a sequncia (2) tende a zero quando n , tem-se:
fn2a fn1b 0.

b f
Dividindo por afn1, v-se que = lim n 2 .
a n f n 1

A ttulo de curiosidade, vamos calcular fn1/fn, com trs casas


decimais, para n = 1, 2, ..., 8:
1 1 2 3 5
= 1, = 0, 5, 0, 666, = 0, 6, = 0, 625,
1 2 3 5 8
8 13 21
0, 615, 0, 619, 0, 618.
13 21 34
Para n > 8, a razo fn1/fn sempre 0,618, com arredondamento na
a
3 casa decimal.

O pentagrama e observaes finais


A diviso urea conhecida desde os pitagricos de cinco sculos
a.C. Ao que tudo indica, essa diviso foi descoberta no pentgono regular,
que exibe uma surpreendente profuso de segmentos na razo urea.
RPM OBMEP

Talvez este tenha sido o motivo que levou os pitagricos a adotarem o


pentagrama (pentgono regular estrelado) como smbolo de sua seita
(figura 10).

88
Na figura, os vrtices da estrela dividem o D
crculo em cinco partes iguais. Portanto, cada
arco mede 72. Os tringulos NBE e ABE so A M N B
semelhantes porque seus ngulos internos
medem 36, 72 e 72. Logo,
NB BE C E
= .
BE AB figura 10
n = NEA
Mas AN = NE (porque NAE n = 36D ) e NE = BE (porque
n = NBE
BNE n = 72D ). Temos portanto,

NB BE
= .
AN AB
e consequentemente essas razes so ureas. Pela semelhana dos
tringulos DMN e AND, prova-se, da mesma forma, que MN/AM
tambm urea.
muito improvvel que Pitgoras ou seus primeiros discpulos
soubessem que os segmentos da diviso urea fossem incomensurveis,
embora haja fundadas razes para se acreditar que a descoberta dos
incomensurveis tenha ocorrido com o pentgono regular no fim do 5o
sculo A.C. Certamente, Pitgoras e seus discpulos sabiam como
construir geometricamente a soluo (6) da equao (5). As construes
correspondentes s figuras 8 e 9 acima se encontram nos Elementos
de Euclides, de cerca de 300 anos A.C.
Na antiguidade, a diviso de um segmento em mdia e extrema razo
tornou-se to familiar que era conhecida simplesmente como a seo,
em qualquer qualificativo. O nome diviso urea lhe foi dado por
Kepler (1571-1630), que escreveu:
A Geometria possui dois grandes tesouros: um o Teorema de
Pitgoras; o outro, a diviso de um segmento em mdia e extrema
razo. Podemos comparar o primeiro a uma poro de ouro e o
RPM OBMEP

segundo a uma jia preciosa.


Tanto a razo urea, como os nmeros de Fibonacci, aparecem numa
variedade enorme de situaes inesperadas. Hoje a literatura sobre os
nmeros de Fibonacci enorme. Existe at uma revista The Fibonacci

89
Quartely fundada em 1963, dedicada pesquisa em torno desses
nmeros!

O Teorema de Lam
Vamos mostrar agora uma aplicao surpreendente dos nmeros de
Fibonacci.
Para encontrar o o mximo divisor comum de dois inteiros, bastante
conhecido o processo que exemplificamos no caso do mdc(243, 37):
6 1 1 3 5
243 37 21 16 5 1
21 16 5 1 0
As operaes efetuadas acima foram as seguintes:
243 = 6 37 + 21
37 = 21 1 + 16
21 = l 16 + 5
16 = 5 3+1
5 = 1 5.
O algoritmo termina quando o resto da diviso nulo; o mximo
divisor comum o ltimo divisor obtido.
O algoritmo do processo apresentado acima o clebre algoritmo de
Euclides, conhecido desde a antiguidade. Ele encontra-se exposto, para
nmeros, na Proposio 1 do livro VII dos Elementos de Euclides,
escritos em torno de 300 a.C.
Os historiadores da Matemtica acreditam que esse algoritmo era
conhecido j em 400 a.C. Ele de importncia fundamental em teoria
dos nmeros.
O que fizemos no exemplo acima pode ser generalizado: Sejam a e
b inteiros positivos, com a > b. Usando sucessivamente o algoritmo da
diviso, escreva
a = bq1 + b1, 0 < b1 < b,
RPM OBMEP

b = b1q2 + b2, 0 < b2 < b1,


b1 = b2q3 + b3, 0 < b3 < b2,
...

90
bn2 = bnl qn + bn, 0 < bn< bn1,
bnl = bnqn+l.
Ento mdc(a, b) = bn.
Com efeito, em primeiro lugar, o processo acima realmente chega
ao fim. De fato, como 0 < bn < bn1 < ... < b1 < b, vemos que esse
processo no pode repetir-se indefinidamente, pois temos uma seqncia
estritamente decrescente de inteiros positivos e h um nmero finito de
inteiros entre 0 e b.
Alm disso, usando estas equaes de baixo para cima, constatamos
que bn divide bn1, bn2, etc, at concluir que bn um divisor comum
de a e b. Em seguida, usando estas mesmas equaes de cima para
baixo, constatamos que todo divisor comum d de a e b tambm um
divisor comum de b1 e b2, de b2 e b3, etc, at concluir que d
tambm um divisor de bn, ou seja, sendo bn um mltiplo de d, bn
maior que ou igual a d, o que mostra que bn o mximo divisor
comum de a e b.
Usando o algoritmo de Euclides, so necessrias n + 1 divises
para vermos que mdc(a, b) = bn, pois s chegamos a uma concluso
quando verificarmos que bn1 = bnqn+1 + bn+1 = bnqn+1 + 0 = bnqn+1.
Chamaremos de comprimento do algoritmo de Euclides o nmero
de divises necessrias para calcular o mdc(a, b). Usando a notao do
teorema, o comprimento do algoritmo de Euclides n + 1.
O algoritmo de Euclides bem eficiente. Por exemplo, se
quisermos verificar que mdc(97, 24) = 1, sero necessrios apenas dois
passos:
97 = 4 24 + 1
24 = 24 1.
Agora, se queremos calcular mdc(21479, 24), temos
21479 = 894 24 + 23,
24 = 1 23 + 1,
RPM OBMEP

23 = 1 23.
Ou seja, em 3 passos vemos que mdc(21479, 24) = 1. Por fim, como
ltimo exemplo, para calcular mdc(49745692, 24), temos

91
49745692 = 2072737 24 + 4,
24 = 6 4;
isto , em apenas 2 passos chegamos ao resultado desejado.
Dados dois nmeros inteiros e positivos a e b, uma pergunta natural
: qual o comprimento do algoritmo de Euclides aplicado a eles? Em
outras palavras, quantas divises so necessrias para calcular o mximo
divisor comum de a e b.
imediato verificar que, se mantivermos b fixo, mesmo que a seja
muito grande em relao a b, o nmero de divises no algoritmo de
Euclides no pode crescer. Em verdade, esse nmero depende apenas de
b. Com efeito, usando mais uma vez a notao acima, sabemos que, no
algoritmo, mdc(a, b) = bn e que 0 < bn < bn1 < ... < b1 < b. Como h no
mximo b 1 inteiros distintos no negativos entre 0 e b, vemos que
n < b 1, donde n + 1 < b. Ora, como j vimos, so necessrias n + 1
divises para determinar o mximo divisor comum. Assim, so
necessrias no mximo b divises para achar mdc(a, b).
No entanto, esse resultado no muito bom. Por exemplo, se b =
99, devemos ter que n + 1 < 99 e chegamos concluso de que talvez
tenhamos que efetuar 99 divises para calcular o mximo divisor
comum!
O Teorema de Lam melhora muito essa situao:
Teorema (Lam). Sejam a e b inteiros positivos. Ento, o comprimento
do algoritmo de Euclides aplicado aos nmeros a e b menor que ou
igual a cinco vezes o nmero de dgitos na representao decimal de b.
Segundo o teorema, se b igual a 99, que tem dois algarismos,
ento o nmero de divises no algoritmo de Euclides no mximo 10,
no sendo influenciado por a. Isso representa um progresso notvel em
relao estimativa anterior.
Esse teorema devido a Gabriel Lam (1795-1870) engenheiro e
matemtico francs, conhecido por seus trabalhos sobre a equao do
RPM OBMEP

calor e criador das coordenadas curvilneas. Embora no tenha se


dedicado sistematicamente teoria dos nmeros, ele deixou algumas
jias sobre o assunto, uma das quais o teorema anterior.

92
A demonstrao do Teorema de Lam um exemplo de utilizao
inteligente dos nmeros de Fibonacci. Em verdade, essa foi, em 1844, a
primeira aplicao significativa desses nmeros.
Para efetuarmos a demonstrao, voltemos ao algoritmo de Euclides.
Em primeiro lugar, b n > 1, pois bn um nmero inteiro. De
bn1 = bnqn+1, vemos que bn1 > 2, pois bn1 > bn. Assim, bn > f1 e
bn1 > f2. Ento,
bn2 = bn1qn + bn > f2 + f1 = f3, pois qn > 1.
Analogamente,
bn3 = bn2 qn1 + bn1 > f3 + f2 = f4 , pois qn1 > 1.
Continuando dessa maneira, vemos, de maneira geral, que
bn-k > fk+1 para k = 0, 1, 2, ..., n 1,
e, enfim,
b = b1q2 + b2 > fn + fn1 = fn+1,
ou seja, fazendo b0 = b, temos:
bnk > fk+1, para k = 0, 1, 2, ..., n.
Ilustrando: ...
bn bn 1 bn  b1 b
> > > ... > >
f1 f2 f3 ... fn fn + 1
Esse resultado nos mostra que o comprimento do algoritmo de
Euclides menor ou igual ao nmero de ordem do maior nmero de
Fibonacci menor ou igual a b.
Podemos ver que esse resultado o melhor possvel achando o
mximo divisor comum entre dois nmeros de Fibonacci consecutivos.
Calculemos, por exemplo, mdc(21,13) = mdc(f7, f6):
21 = 13 + 8
13 = 8 + 5
8=5+3
RPM OBMEP

5=3+2
3=2+1
2 = 1 2 + 0.
Nesse exemplo, f7 e f6 no desempenham nenhum papel essencial,

93
pois o mesmo acontece no caso geral, para achar mdc(fn1, fn).

5 +1
Consideremos agora o nmero ureo u =  1, 618... .
2
Temos, como visto anteriormente, u2 = 1 + u. Logo,
u2 = u + 1 < 2 +1 < f2 + f1 = f3.
u3 = u2 + u < f3 + 2 < f3 + f2 = f4,
u4 = u3 + u2 < f4 + f3 = f5,
e assim sucessivamente, chegando enfim a uj < fj+1, j = 2, 3, 4, ... .
Em particular, un < fn +1 < b.
Como a funo log10x estritamente crescente, temos que
log10 b
nlog10u < log10b, ou, equivalente, n < .
log10 u
Ora, calcula-se facilmente, usando uma tbua de logaritmos ou uma
mquina de calcular, que
1+ 5 1 1
log10 u = log10 = 0, 20898 > 0, 20 = , ou seja, < 5.
2 5 log10 u
log10 b
Assim, n < < 5 log10 b
log10 u
Se o nmero de algarismos na representao decimal de b s,
ento
b = ts110s1 + ts210s2 + ... + t110 + t0,
e, portanto, b < 10s, donde log10b < s, e vemos que n < 5s. Como n
um inteiro estritamente menor do que 5s, temos que n + 1 < 5s, o
resultado procurado.
Adaptado dos artigos
Retngulo ureo, diviso urea e seqncia de Fibonacci.
Geraldo vila, RPM 06.
RPM OBMEP

O smbolo da SBM
Eduardo Wagner, RPM 20.
Euclides, Fibonacci e Lam
Joo Bosco Pitombeira de Carvalho, RPM 24.

94
Usando Geometria
para somar

Introduo
Um dos maiores prazeres da Matemtica o da descoberta
(ou da redescoberta) de resultados matemticos mesmo
que j conhecidos. Iniciamos este artigo com o problema
clssico de calcular a soma dos n primeiros nmeros
naturais pelo mtodo que Gauss teria utilizado aos dez
anos para somar de 1 a 100 de cabea, para surpresa do
seu professor. Mostraremos ainda como calcular a soma
dos quadrados dos n primeiros naturais e outros
resultados interessantes.
Seja S = 1 + 2 + 3 + ... a soma dos n primeiros nmeros
naturais. O mtodo utilizado pelo jovem Gauss para
calcular essa soma bastante simples, embora engenhoso.
Ele escreveu a soma pedida e, embaixo, escreveu a mesma
soma ao contrrio
S = 1 + 2 + (n 1) + n
S = n + (n 1) + ... + 2 + 1
Somando, obtemos 2S = (n + 1) + (n + 1) + ... + (n + 1)
n(n + 1)
com n parcelas, ou seja, S =
RPM OBMEP

.
2
A demonstrao a seguir usa essencialmente a mesma
ideia.

95
A soma dos n primeiros nmeros naturais
I. A soma dos n primeiros nmeros naturais pode ser visualizada
geometricamente atravs da figura abaixo. Nela v-se um retngulo
formado por bolinhas. A base do retngulo possui n + 1 bolinhas e a
altura tem n bolinhas. No total, temos ento n(n + 1) bolinhas. Observe
agora que elas esto divididas em duas partes iguais pela linha poligonal
e em cada uma delas aparece a soma S = 1 + 2 + 3 + ... + n. Obtm-se,
ento, a frmula da soma dos n primeiros nmeros naturais.

1
1 + 2 + ... + n = n(n + 1)
2

Nota da RPM: A figura acima apareceu na coluna de Martin Gardner


Mathematical Games, da Scientific American de outubro de 1973, junto
com vrias outras chamadas look-see diagrams (diagramas olhe-veja) e
o autor cita que ela j era conhecida pelos gregos antigos. Posteriormente,
figuras demonstrando resultados matemticos conhecidos apareceram
em vrias revistas. Em 1993, as melhores demonstraes dessas revistas
foram reunidas por Roger Nelsen e publicadas no livro Proofs without
words Exercices in visual thinking pela MAA (Mathematical
Association of America). Esse livro ser a nossa principal referncia
neste artigo, doravante designado por [Pww] (Pww - Proofs without
words Demonstraes sem palavras)
II. Uma outra forma de obter a soma dos n primeiros nmeros naturais
utiliza a figura a seguir e o conceito de rea. Observe que a soma
1 + 2 + 3 + ... + n igual rea do tringulo grande (metade de um
Quadrado de lado n) mais a metade de n quadrados.
RPM OBMEP

n 2 n n(n + 1)
1 + 2 + ... + n = + =
2 2 2

96
A soma dos nmeros mpares
A soma dos n primeiros nmeros mpares pode ser visualizada atravs
da figura a seguir. O fato de que essa soma igual a n2 j era do
conhecimento dos antigos pitagricos, mas a figura da autoria de
Nicmaco de Gerasa (um pitagrico tardio), que viveu em torno do ano
100 d.C. [Pww].

1 + 3 + 5 + ... + (2n 1) = n 2

A soma dos quadrados dos n primeiros nmeros naturais


I. Considere inicialmente trs castelos iguais como os da figura A.
Cada um deles formado por 1 + 4 + 9 + ... + n2 cubos unitrios. Na
figura B os trs castelos foram reunidos e nota-se que o ltimo andar
possui apenas a metade dos cubos necessrios para completar um
paraleleppedo.

RPM OBMEP

figura A figura B
figuras de [Pww]

97
A figura C mostra os cubos do ltimo andar cortados horizontalmente
pela metade, sendo a parte de cima (mais escura) utilizada para completar
o paraleleppedo. Pela figura D vemos que a soma 1 + 22 + ...+ n2, que

o volume dos castelos iniciais, um tero do volume de um

1
paraleleppedo de base n por n + 1 e altura n + . Portanto,
2
1 1 n(n + 1)(2n + 1)
1 + 22 + ... + n 2 = n(n + 1)(n + ) = .
3 2 6

figura C figura D
figuras de [Pww]

A soma de uma srie geomtrica


Sendo r um nmero positivo menor que 1, quanto vale a soma
infinita 1 + r + r2 + r3 + ...? A resposta pode ser obtida atravs de um
desenho bastante engenhoso.
Na figura seguinte, ASPQ um quadrado de lado 1 e AR = r. A reta
PR forma o tringulo PTS no qual ST = 1 + r + r2 + r3 + ...
(Essa ltima igualdade pode ser verificada construindo-se um novo
quadrado de lado r obtendo o segmento de medida r2 uma vez que a
1 r
razo entre as medidas dos segmentos = . Continuando o
r r2
RPM OBMEP

processo, construindo-se quadrados de lados r2, r3, ... obtemos a medida


indicada para o segmento ST.)

98
ST QP
Da semelhana entre os tringulos PTS e RPQ temos = ,
SP QR
ou seja, 1 + r + r 2 + r 3 + ... = 1 .
1 r
Para voc pensar: O que cada umas das figuras seguintes pode mostrar?

Sugesto: figura da esquerda: 4(1 + 3 + ...)


figura da direita: 1 + 2.22 + 3.32 + 4.42 + 5.52 = ...

Observao
certo que, as demonstraes visuais podem parecer, em um primeiro
momento, ter algo de magia. importante perceber que a mgica
apenas aparente: a descoberta de um resultado matemtico fruto de
experimentao, dedicao e compreenso dos conceitos envolvidos.
Deve ficar clara tambm a necessidade, no caso de nmeros naturais,
por exemplo, do axioma da induo, para provar definitivamente certos
resultados.
RPM OBMEP

Adaptado do artigo
Usando Geometria para somar
Eduardo de Campos Valadares e Eduardo Wagner, RPM 39.

99
Mdias

As mdias mais conhecidas pelos estudantes e professores


de Matemtica so a mdia aritmtica, a mdia
geomtrica e a mdia harmnica.
Para dois nmeros a e b, estas mdias so,
respectivamente:
a+b 2ab
A= G = ab H= .
2 a+b
Para calcular a mdia geomtrica G, costuma-se exigir
que a e b sejam positivos e, para calcular a mdia
harmnica H, exige-se que a e b sejam no nulos. De
agora por diante, consideraremos apenas nmeros
positivos.
A mdia geomtrica tambm pode ser escrita como:

G2 = ab ou a = G . A mdia harmnica tambm pode


G b
ser vista como o inverso da mdia aritmtica dos inversos,

isto : 1 = 1 a + 1 b .
H 2
RPM OBMEP

Decorre imediatamente das definies que AH = G2, o


que mostra que a mdia geomtrica de dois nmeros
tambm a mdia geomtrica entre a mdia aritmtica e a
mdia harmnica destes nmeros.

100
Para ter uma viso unificada destas trs mdias, considere as relaes
seguintes envolvendo os nmeros reais a, b e c, positivos e distintos:
ac a
= (1)
cb a
ac a
= (2)
cb b
ac a
= (3)
cb c
Estas equaes diferem apenas nos segundos membros: na equao
(1) o denominador do quociente a, na (2) b, e na (3) c.
Isolando c na equao (1), obtemos c = (a + b)/2, ou seja, c a
mdia aritmtica de a e b; isolando c em (2), obtemos
c = 2ab/(a + b), ou seja, c a mdia harmnica de a e b; isolando c
em (3), obtemos c = ab , ou seja, c a mdia geomtrica de a e b.
Por exemplo, para os nmeros 2 e 18, temos:
2 + 18 2.2.18
A= = 10 G = 2.18 = 6 H= = 3, 6 .
2 2 + 18
Neste exemplo, observa-se que qualquer destas mdias est entre o
menor e o maior dos nmeros. Mais ainda, no exemplo:
2 < 3,6 < 6 < 10, ou seja a < H < G < A < b.
Vamos mostrar agora que este fato geral, isto :
Dados os nmeros positivos a e b, com a < b,
tem-se: a < H < G < A < b.
Alm disso, se a < b, ento: a < H < G < A < b.
a+a 2aa
De fato, se a = b, ento A = = a; G = a.a = a; H = = a,
2 a+a
isto , a = H = G = A = b.
RPM OBMEP

Por outro lado, se a < b, temos, sucessivamente:


2ab
a + b < b + b = 2b; a(a + b) < 2ab; a < , isto : a < H.
a+b

101
2 4a 2 b 2
(a b)2 = a2 2ab + b2 > 0; a2 + 2ab + b2 > 4ab; (a + b) > ;
ab

4a 2 b 2 2ab
ab > 2
; ab > , ou seja: H < G.
( a + b) a+b

a+b
0 < ( a b )2 = a + b 2 a b ; ab < , ou seja: G < A.
2
a+b
a + b < b + b; a + b < 2b; < b , isto : A < b.
2
possvel tambm visualizar geometricamente essas desigualdades.
Para isso, como na figura 1, colocamos consecutivamente numa mesma
reta os segmentos PQ = a e QS = b, com Q entre P e S. Com centro
no ponto mdio M de PS, construmos uma semicircunferncia K, e
os segmentos MT e QD, perpendiculares a PS, com T e D em K.
T
D
K A
H
E G

P M Q S
a
b
figura 1
Construimos tambm o segmento DM e o ponto E, projeo
ortogonal de Q sobre DM. Como a + b dimetro e MT o raio de K,
ento MT a mdia aritmtica de a e b, isto , MT = A.
Alm disso, o tringulo PDS retngulo em D, por estar inscrito
em uma semicircunferencia de dimetro PS; logo, a altura QD a
mdia geomtrica de PQ = a e QS = b, ou seja: QD = G.
Finalmente, no tringulo retngulo DQM, DE a projeo ortogonal
RPM OBMEP

do cateto DQ sobre a hipotenusa DM = A como DE; logo: DQ2 =


DM.DE, isto : DE = DQ2/DM = G2/A = H. Logo: DE = H a mdia
harmnica de a e b.

102
Na figura 1, pode-se verificar que H < G < A. De fato, H < G
porque H cateto e G hipotenusa no tringulo DEQ, enquanto
G < A porque G cateto e A hipotenusa no tringulo DQM.
Note que quando a = b, o ponto Q coincide com M. Neste caso,
H = G = A, como era de esperar.
Outra maneira de visualizar as desigualdades entre as mdias
considerar um trapzio com bases a e b.
a

b
figura 2
Observamos que conforme um segmento paralelo s bases a e b,
com extremidades nos dois lados transversos, caminha se afastando
de a e se aproximando de b, sua medida assume todos os valores entre
a e b, e, consequentemente, todas as mdias consideradas.
Veremos a seguir que os valores A, G e H aparecem conforme
o segmento paralelo s bases assume alguma posio notvel no trapzio.
1) Suponhamos o segmento de medida m equidistando das bases.
a

h
m

b
figura 3
RPM OBMEP

A soma das reas dos dois trapzios menores a rea do trapzio


inicial:
(a + m)h (m + b)h (a + b)2h o que implica m = a + b = A .
+ =
2 2 2 2
103
2) Consideremos agora que o segmento de medida m divida o trapzio
inicial em dois trapzios semelhantes.
a
h1
m

A
h2

b
figura 4
a m
Temos = , implicando m = ab = G .
m b
a h
Note que, sendo = 1 , pois os trapzios so semelhantes,
ab h2

temos h1 < h2, pois a < ab , e, por isso, h1 < h. Logo, m = G < A.

3) Finalmente, consideremos o segmento de medida m passando pelo


encontro das diagonais.
a
h3
x y
h4

b
figura 5
Por semelhana de tringulos temos:
x h4 x h3 y h4
= ; = ; e = ,
a h4 + h3 b h4 + h3 a h4 + h3

x y
implicando x = y e + = 1.
a b
RPM OBMEP

104
ab
Substituindo x = y na ltima igualdade, encontramos x = e,
a+b
2ab
portanto, m = 2 x = =H .
a+b
a h3
Ainda, usando semelhana de tringulos, temos = . Mas
b h4
a h a a h h
= 1 e < , ou seja, 3 < 1 e como h3 + h4 = h1 + h2
ab h2 b ab h4 h2

tem-se h3 < h1 e, portanto, H < G.

Caso permitssemos que as bases do trapzio se igualassem, ou seja,


ter a = b, o trapzio se transformaria num paralelogramo e assim, obvia-
mente, as trs mdias se igualariam a a e b.

Onde aparece a mdia harmnica


So inevitveis as perguntas pragmticas que alunos e professores
costumam fazer: Para que serve o estudo da mdia harmnica? Onde se
aplica a mdia harmnica?
Sem a pretenso de responder cabalmente a essas perguntas, vou
apenas salientar a importncia da mdia harmnica, assinalando a sua
presena em alguns problemas da vida prtica.
O problema das velocidades
O sr. Mrio, um imprudente vendedor de filtros de gua, costuma
acordar cedo e viajar de carro, da cidade A at a cidade B, com a
velocidade mdia de 120 km/h. Depois de visitar seus clientes e tomar
com eles algumas garrafas de cerveja, ele volta de B para A, com a
velocidade mdia de 60 km/h. Qual a velocidade mdia que o sr.
Mrio desenvolve no percurso todo?
A resposta mais imediata que surge em nosso crebro que a
velocidade mdia no percurso todo a mdia aritmtica das velocidades
na ida e na volta, o que daria 90 km/h. Essa resposta, embora intuitiva,
RPM OBMEP

est errada! Temos que estar sempre alertas, maneira dos escoteiros,
para no deixar a razo matemtica ser desgovernada por falsas
intuies.

105
A resoluo correta do problema a seguinte. Sejam:
d: a distncia entre as cidades A e B
v1: a velocidade mdia na ida t1: o tempo de viagem na ida
v2: a velocidade mdia na volta t2: o tempo de viagem na volta
Temos ento que d = v1t1 = v2t2. Se v a velocidade mdia no per-
curso todo, temos:
2d = v(t1 + t2). Logo, 2d = v(d/v1 + d/v2).
Simplificando: v = 2v1v2/(v1 + v2).
Substituindo os valores v1 = 120 km/h e v2 = 60 km/h, obtemos
v = 80 km/h.
Moral da histria: a velocidade mdia no percurso todo a mdia
harmnica das velocidades na ida e na volta.
A mdia harmnica geralmente aparece em problemas que envolvem
velocidades, vazes, frequncias e taxas. O exemplo seguinte uma
verso simples de um problema de vazo bastante conhecido.
O problema das torneiras
Se uma torneira enche um tanque em 60 minutos e uma outra torneira
enche o mesmo tanque em 30 minutos, em quanto tempo as duas
torneiras juntas enchem o tanque?
Os leitores esto convidados a resolver mais esse problema, e para
isso damos uma pequena dica: a resposta no a mdia harmnica de
60 min e 30 min, mas est relacionada a ela.
Problemas de torneiras so antiqussimos. Uma de suas verses
aparece por exemplo na Antologia grega organizada por Metrodoro, um
matemtico grego que vivia por volta do ano 500 depois de Cristo. A
traduo para o portugus seria mais ou menos a seguinte:
Eu sou um leo de bronze; de meus olhos, boca e p direito jorra
gua. Meu olho direito enche uma jarra em dois dias, meu olho
RPM OBMEP

esquerdo em trs dias, e meu p direito em quatro dias. Minha


boca capaz de ench-la em seis horas, diga-me quanto tempo
os quatro juntos levaro para ench-la?
Para finalizar esta seo, mais um problema.
106
O problema do usque
Durante 4 meses consecutivos, o sr. Mrio comprou usque para o
bar de sua casa aos preos, respectivamente, de 16, 18, 21 e 25 reais
por garrafa. Qual foi o custo mdio do usque para o sr. Mrio nesse
perodo todo?
Esse um daqueles problemas que nos deixam frustrados, pois s
depois de muita batalha notamos que faltam dados; temos
necessariamente que introduzir alguma hiptese para poder resolver o
problema.
(i) Uma hiptese plausvel que, talvez por ser um bebedor regular, o
sr. Mrio tenha comprado a mesma quantidade x de usque a cada
ms.
Logo, ele despendeu 16x + 18x + 21x + 25x = 80x reais para comprar
usque no perodo. Da, o custo mdio no perodo de 4 meses foi de
80x/4x = 20 reais por garrafa. Portanto, caso essa hiptese seja verdadeira,
o custo mdio no perodo a mdia aritmtica dos custos mensais.
(ii) Uma outra hiptese plausvel que, talvez por no ter tido aumento
de salrio nesse perodo, o sr. Mrio tenha gasto a mesma quantia y
de reais a cada ms.
Logo, ele consumiu y/16 + y/18 + y/21 + y/25 garrafas no perodo.
Assim, o custo mdio nesse perodo foi, aproximadamente:
4y/(y/16 + y/18 + y/21 + y/25) = 19,5 reais por garrafa.
Portanto, neste caso, o custo mdio no perodo a mdia harmnica
dos custos mensais.

Mdias para mais de dois nmeros


As mdias que vimos para dois nmeros podem ser generalizadas
para mais nmeros. Vamos fazer isto aqui somente para as mdias
aritmtica e geomtrica.
Dados n nmeros positivos x1, x2, ... , xn, definimos.
RPM OBMEP

x1 + x2 + ... + xn e
A= G = n x1 x2 ... xn .
n

107
Deixamos para o leitor verificar que, se todos os nmeros forem iguais
a um valor v, ento A = G = v. Porm, se eles no forem todos iguais,
a mdia geomtrica sempre menor que a mdia aritmtica, mas a
demonstrao, nesse caso, no to fcil como no caso n = 2. Existem
demonstraes de vrios tipos, de diversos graus de sofisticao e
baseadas em diferentes teorias. A mais conhecida a demonstrao de
Cauchy (1789-1857), que pode ser encontrada no livro Meu Professor
de Matemtica e outras histrias de Elon Lages Lima, p. 153, publicado
pela SBM. Vamos dar aqui a demonstrao concebida por Polya, que se
baseia na desigualdade ex > 1 + x, justificada a seguir:
Observemos que se pode definir o logaritmo (natural) de um nmero
positivo a como sendo a rea limitada pelo
eixo das abscissas, pela curva y = l/x e pelas y = 1x
retas verticais x = 1 e x = a. Como essa
regio est contida no retngulo de altura 1 e
base a 1, temos, claro, ln a < a 1. 1
Fazendo a = 1 + x, obtemos ln(1+ x) < x ou lna
ex > 1 + x, valendo a igualdade apenas se 1 a
a = 1, ou seja, x = 0.
Resolvida essa parte, podemos ento demonstrar a desigualdade das
mdias para n nmeros.
xi
Na desigualdade ex > 1 + x vamos substituir x por 1 , com
A
i = 1, 2, ..., n, obtendo as relaes
x1
1 x1
eA
A
x2
1 x2
eA
A
xn
1 xn
RPM OBMEP

eA
A
x1 + x2 +...+ xn
n x1 x2 ... xn
Multiplicando, obtemos e A .
An
108
Gn
Mas, x1 + x2 + ... + xn = nA, logo 1 ou A > G.
An
como queramos demonstrar. claro que a igualdade vale se, e somente
xi
se, 1 = 0 ou seja, xi = A para todo i = 1, 2, ..., n.
A

Aplicaes das desigualdades das mdias


1
Exemplo 1: Mostre que se x > 0, ento: x + 2.
x
Soluo: Aplicando a desigualdade das mdias aos nmeros x e l/x,
1
x+
obtemos x x. 1 = 1 ou seja, x + 1 2 , ocorrendo a igualdade
2 x x
se e somente se, x = 1.

Exemplo 2
Para todos os valores das variveis x, y, z, w, reais positivas, qual o
menor valor da expresso
x y z w
E= + + +
y z w x ?
Soluo:
x y z w
+ + +
y z w x x y z w
4 . . . =1
4 y z w x

Logo, E > 4, ocorrendo a igualdade se x = y = z = w.

Exemplo 3
2 1
Para x > 0, qual o valor mnimo de y = x + ?
x
RPM OBMEP

1 1
Soluo: Escrevemos y = x 2 + + . Desta forma temos
2x 2x

109
1 1
x2 + +
2 x 2 x 3 x2 . 1 . 1 = 3 1 .
3 2x 2x 4
3 1
Portanto, y 3
, ocorrendo a igualdade quando x 2 = , ou seja,
4 2x
1
x= 3 .
2
Exemplo 4
Se x, y e z so positivos, qual o valor mnimo de
1 1 1
( x + y + z )( + + ) ?
x y z
Este exemplo ser deixado como exerccio para o leitor. A resposta 9.
A desigualdade entre as mdias aritmtica e geomtrica tem como
consequncia as seguintes afirmaes:
I) Se a soma de n nmeros positivos for constante, ento o produto
ser mximo quando todos os nmeros forem iguais.
II) Se o produto de n nmeros positivos for constante, ento a soma
ser mnima quando todos os nmeros forem iguais.
Daremos mais dois exemplos para mostrar como funciona a afir-
mao I).
Exemplo 5
Sendo x e y nmeros reais positivos, determinar o mximo de
E = xy(1 x y).
Soluo: Consideremos apenas os valores de x e y tais que x + y < 1
(se x + y > 1, teremos 1 x y < 0 e o mximo que estamos procurando
obviamente positivo).
Ento, os nmeros x, y e 1 x y so positivos e possuem soma igual
a 1. Logo, o produto ser mximo quando todos forem iguais ou seja,
RPM OBMEP

1 1 1 1
Emax = . . = .
3 3 3 27

110
Exemplo 6
Provar que, de todos os tringulos de mesmo permetro, o equiltero
possui a maior rea.
Soluo: Consideremos um tringulo de lados a, b e c com
a + b + c = 2p. A rea S desse tringulo dada pela frmula de Heron,
S= p.( p a )( p b)( p c)

Para poder aplicar a afirmativa I, devemos escrever

S= p . ( p a )( p b)( p c)

Ora, o semipermetro p constante. Ento S ser mximo quando


(p a)(p b)(p c) for mximo. Mas
p a + p b + p c = 3p 2p = p (constante)
logo o produto ser mximo quando
p
p a = p b = p c = , ou seja
3
2p
a=b=c= como queramos demonstrar.
3

Adaptado dos artigos


Duas mdias
Eduardo Wagner, RPM 18.

Mdia harmnica
Seiji Hariki, RPM 32.

Uma aula sobre mdias


Chico Nery, RPM 68.
RPM OBMEP

111
Problemas diversos resolvidos
com Geometria Analtica

A Geometria Analtica, ou melhor o mtodo das


coordenadas, uma ferramente til para resolver
problemas diversos, mesmo aqueles que no contm
equaes ou coordenadas. interessante observar
problemas que permitem a introduo de um sistema
adequado de coordenadas e conseguir assim, solues
simples e convincentes.
A seguir, mostraremos alguns problemas desse tipo. Para
cada um deles outras formas de resoluo so possveis,
mas o mtodo das coordenadas uma boa opo.
Problema 1
Um pesado caminho parte ao meio dia da cidade A para
a cidade B viajando com velocidade constante de
40 km/h, e s 6 horas da tarde chega cidade B. Um
automvel parte da cidade B s 2 horas da tarde desse
dia e, viajando com velocidade constante pela mesma
estrada, chega cidade A tambm s 6 da tarde. Pergunta-
se em que momento o caminho e o automvel se
cruzaram na estrada.
Soluo
RPM OBMEP

A distncia entre as cidades A e B, ao longo da estrada


de 6 40 = 240 km. Vamos introduzir o seguinte sistema
de coordenadas: para um objeto qualquer que se mova

112
ao longo da estrada, seja x o tempo (em horas) decorrido aps o meio
dia e seja y (em quilmetros) a sua distncia cidade A. Os grficos
correspondentes aos movimentos do caminho e do automvel so retas,
uma vez que eles viajam com velocidades constantes. De acordo com os
dados do problema, o grfico que mostra o movimento do caminho
um segmento de reta cujos extremos so os pontos (0, 0) e (6, 240), e
o grfico que mostra o movimento do automvel um segmento de reta
cujos extremos so os pontos (2, 240) e (6, 0). As equaes das retas
do grfico abaixo so y = 40x e y = 60x + 360.
y
Resolvendo o sistema encontra-
mos x = 3,6 e y = 144. Conclu- 240
mos ento que o encontro se deu
3,6 horas aps o meio dia, ou
seja s 3 horas e 36 minutos da
tarde e, nesse momento, ambos
estavam a 144 km da cidade A. x
Esse problema nada tem de original, mas serve para ilustrar que
diversos problemas de cinemtica escalar podem ser resolvidos com o
mtodo analtico.

Problema 2
Considere todos os nmeros reais x e y tais que x + 2y = 10. Para que
valores de x e y a expresso E = x2 + y2 assume menor valor?
Soluo
Este um problema de lgebra. Sua soluo, uma vez que o enunciado
esteja bem entendido, no difcil. Entretanto, a soluo analtica
interessante.
Estabelecendo um sistema de y
coordenadas, todos os pontos s
P(x, y) tais que x + 2y = 10 P
pertencem a uma reta r e o valor
RPM OBMEP

de E o quadrado da distncia r
de P origem do sistema de
coordenadas. O

113
Precisamos ento encontrar o ponto de r cuja distncia ao ponto (0, 0)
mnima. A reta s, perpendicular a r e passando pela origem tem equao
2x y = 0 e a interseo dessas retas o ponto que procuramos.
Resolvendo o sistema formado pelas duas equaes encontramos x = 2
e y = 4 que a soluo do problema. Conclumos ainda que o valor
mnimo de E 22 + 42 = 20.
Problema 3
Na molcula do metano (CH4) o tomo de carbono ocupa o centro de
um tetraedro regular em cujos vrtices esto os tomos de hidrognio.
Determine o ngulo entre duas das valncias do carbono.
Soluo
O resultado deste problema est presente em todos os cursos de qumica
orgnica. O estranho nmero fornecido aceito por todos, mas, em geral,
no se tem a menor idia de como esse resultado foi obtido. Para calcular
esse ngulo, a Geometria Analtica um mtodo imbatvel, aliada
claro, com alguma inventividade.
Vamos utilizar um sistema de coordenadas no espao e usar a frmula
G G
que fornece o cosseno do ngulo entre dois vetores u e v :
G G
u v
cos = G G . Consideremos inicialmente um cubo de aresta 2 (para
| u || v |
facilitar) com um vrtice na origem, outro no eixo X, outro no eixo Y
e outro no eixo Z. No difcil escolher quatro vrtices deste cubo que
formem um tetraedro regular.
Os pontos A = (0, 0, 0), B = (2, 2, 0), C = (0, 2, 2) e D = (2, 0, 2)
formam um tetraedro regular (uma vez que as distncias entre dois
quaisquer deles so diagonais de faces do cubo) e so ocupados pelos
hidrognios.
O ponto P = (1, 1, 1), centro do cubo e tambm centro do tetraedro,
est ocupado pelo carbono.
RPM OBMEP

O resto fcil. Para calcular, por exemplo o ngulo APB,


G JJJG G JJJG
consideremos os vetores u = PA = (1, 1, 1) e v = PB = (1,1, 1) .

114
O cosseno do ngulo entre eles : C
1 1 + 1 1 D
cos = = .
3 3 3
P
Com uma calculadora, determinamos um
valor muito aproximado para esse ngulo:
= 1092816,395". B

Vamos, nos prximos exemplos, resolver vrios problemas de


Geometria Plana, usando Geometria Analtica. Naturalmente, todos os
problemas apresentados podem ser resolvidos utilizando-se geometria
sinttica. Sugerimos que os leitores tentem obter essas solues.

Problema 4
O quadrado ABCD tem lado 10. Sendo M o ponto mdio de BC, trace
DP perpendicular a AM. Qual o comprimento do segmento DP?
Soluo
A soluo pela geometria sinttica passa pela descoberta que os tringulos
ABM e DPA so semelhantes. Com isso, e mais uma aplicao do
teorema de Pitgoras se resolve o problema. Entretanto, com os recursos
da Geometria Analtica, a soluo no depende da descoberta dessa
semelhana. Podemos escolher um sistema de coordenadas com o eixo
X passando por AB e com o eixo Y passando por AD.
Na figura ao lado temos:
A = (0, 0), B = (10, 0), C = (10, 10), D C
D = (0, 10) e M = (10, 5). A equao da
reta AM x 2y = 0 e o comprimento do M
segmento DP a distncia do ponto D
reta AM. A distncia do ponto (x0, y0) P
reta ax + by + c = 0 dada por: A B
| ax0 + by0 + c | .
d=
RPM OBMEP

a 2 + b2
| 0 2.10 | 20
Temos, ento, DP = = =4 5.
12 + (2) 2 5

115
Problema 5 A B
Na figura, o quadrado ABCD tem lado 9 e os
pontos P e Q dividem o lado CD em trs
segmentos congruentes. Calcule a distncia do
vrtice A ao baricentro G do tringulo BPQ. G

Soluo D P Q C
Encaixemos o quadrado ABCD no primeiro quadrante do plano
cartesiano, com o vrtice D coincidindo com a origem. Sendo
A= (0, 9), B = (9, 9), P = (3, 0) e Q = (6, 0), conhecido que as coor-
denadas do baricentro G so:
y
xB + xP + xQ 9+3+ 6 A B
xG = = = 6,
3 3
yB + yP + yQ 9 + 0 + 0
yG = = = 3,
3 3 G
portanto, G = (6, 3).
D P Q C x
A distncia procurada :

d AG = ( xG x A ) 2 + ( yG y A ) 2 = (6 0) 2 + (3 9) 2 = 72 = 6 2 .

Problema 6 A
As medianas AM e BN de um
tringulo ABC so perpendiculares N
e medem, respectivamente, 9 cm e G
12 cm. Calcule o comprimento da
terceira mediana desse tringulo. B M C
Soluo
y
6 A
O encontro das medianas o
baricentro G do tringulo ABC.
Usando o fato de que as medianas
RPM OBMEP

AM e BN se cortam perpendi- B G N
cularmente em G, coloquemos 4 x
8
esse tringulo no plano cartesiano M

C
116
com origem em G. Usando a conhecida proporo em que G divide as
medianas, temos:
AM = 9 AG = 6 e GM = 3
, ou seja, A = (0, 6), M = (0, 3),
BN = 12 BG = 8 e GN = 4
B = (8, 0) e N = (4, 0). As equaes segmentrias das retas AN e BM
so, respectivamente,
x y x y
+ =1 e + = 1.
4 6 8 3
Resolvendo o sistema anterior, encontramos o ponto C = (8, 6). O
comprimento da terceira mediana 3/2 da distncia entre C e G:
3 3
( xC xG ) 2 + ( yC yG ) 2 = (8 0) 2 + (6 0) 2 = 15 cm .
2 2

Problema 7 C
Calcule a rea do tringulo ADE, E
retngulo em E, inscrito num trapzio B
retngulo ABCD, com AB = 10 cm,
AD = 30 cm e CD = 20 cm (figura).
A D
Soluo
Encaixemos o trapzio ABCD no y
primeiro quadrante do plano cartesiano, C
fazendo os lados AD e AB ficarem
contidos, respectivamente, nos eixos E
B
20
x e y. Como a reta BC tem coeficiente 10
M
yC yB 20 10 1 A 15 D x
angular m = = = e
xC xB 30 0 3

1
coeficiente linear 10, sua equao reduzida y = x + 10 . A
3
RPM OBMEP

circunferncia de dimetro AD, com centro M = (15, 0), passa pelo


ponto E e tem equao:
(x 15)2 + y2 = 152.

117
1
y = x + 10
O ponto E dado pela soluo do sistema: 3 ,
( x 15) 2 + y 2 = 225

ou seja, E = (6, 12) ou E = (15, 15). Portanto, a rea do tringulo ADE

0 0 1 0 0 1
1 2 1
: S ADE = 30 0 1 = 180 cm ou 30 0 1 = 225 cm 2 .
2 2
6 12 1 15 15 1

Adaptado dos artigos


Sobre o ensino de Geometria Analtica
Eduardo Wagner, RPM 41
A Geometria Analtica no ensino mdio
Chico Nery, RPM 67
RPM OBMEP

118
A sombra do meu abajur

Introduo
A fotografia abaixo reproduz o abajur do meu quarto e a
sombra que ele projeta na parede. Que curvas so essas?

A cpula do abajur um tronco de cone, com altura h e


raios das bases R e r. A lmpada centralizada, de modo
a ficar no eixo do tronco de cone. Para simplificar,
podemos imaginar a lmpada concentrada em um ponto,
situado a uma distncia b da base inferior e a uma
distncia c da base superior do tronco, sendo b + c = h
(ver figura 1).
RPM OBMEP

figura 1
119
A funo da cpula barrar uma parte dos raios de luz, evitando que
a luz atinja diretamente a vista. Os raios de luz que escapam dessa
barragem formam um par de cones, ambos com vrtice na lmpada (ver
figura 2). Devemos imaginar esses cones prolongados para alm das
bases da cpula, um para cima e outro para baixo.

figura 2

Equacionamento e resoluo do problema


Para descobrir a natureza da sombra do abajur, vamos cuidar primeiro
da parte superior da sombra, que a interseo da parede com o cone de
luz superior. Esse cone fica caracterizado pelo ngulo da figura 3, que
define sua abertura e tal que: m = tg = r/c.

figura 3
Criemos um sistema de coordenadas em trs dimenses OXYZ, de
modo que a origem O do sistema esteja sobre a lmpada (concebida
RPM OBMEP

como um ponto) e o eixo do cone coincida com o semi-eixo positivo


OZ, como na figura 4.

120
figura 4
Nessa figura, vemos que um ponto genrico P = (x, y, z) pertence
superfcie do cone se e s se, enquanto ele distar z = AP do plano XOY,

sua distncia PB ao eixo permanecer igual a OA = x 2 + y 2 . Porm,


a figura 5, vista no plano AOB, mostra que: PB/PA = tg = m, ou seja:

PB = mPA = mz. Logo, a equao do cone : mz = x 2 + y 2 .

RPM OBMEP

figura 5
A parede um plano paralelo ao eixo do cone. Podemos ajustar os
eixos OX e OY de modo que o plano XOZ fique paralelo parede. Nesse

121
caso, a parede tem equao y = d, onde d a distncia da lmpada
parede.
Finalmente, a curva que procuramos a interseo do cone de equao

mz = x 2 + y 2 com o plano de equao y = d, isto , o conjunto dos

2 2
pontos (x, y, z) que so solues do sistema: mz = x + y .
y = d

2 2
Esse sistema obviamente equivalente ao sistema: mz = x + d ,
y = d
e, se olharmos esses pontos no plano y = d, poderemos ficar somente

com a equao mz = x 2 + d 2 , que a equao dessa curva plana


nesse plano.
c 2
Lembrando que m = r/c, essa equao fica: z = x + d2 .
r
Para a parte inferior da sombra, um raciocnio inteiramente anlogo
b
concluiria que a equao dessa parte da sombra : z = x2 + d 2 .
R
Para o abajur do meu quarto, essas dimenses so, aproximadamente:
r =10 cm, R = 25 cm, b = 10 cm, c = 20 cm e
(quando o abajur est no seu lugar mais usual) d = 40 cm.
Nesse caso, as equaes da
sombra so, em centmetros:

z = 2 x 2 + 1600

e z = 0, 4 x 2 + 1600 .
RPM OBMEP

A figura 6 mostra esses


grficos obtidos por um programa
de computador.
figura 6

122
Identificao das curvas
Essas curvas que obtivemos so uma novidade ou ser que j as vimos
por a no ensino mdio? Na verdade, elas so velhas conhecidas. A sombra
c 2
superior tem equao z = x + d 2 . Elevando essa equao ao
r

z2 x2
quadrado e manipulando, obtemos: = 1 , que a equao de
( )
2
cd d2
r

uma hiprbole com centro na origem do plano XZ, eixo transverso sobre
2cd
o eixo Z, de comprimento , e eixo no transverso sobre o eixo X, de
r
comprimento 2d.

c 2
Como a equao z = x + d 2 equivalente ao sistema,
r

z2 x2
=1
( )
2
cdr d2 , v-se que a sombra superior o ramo positivo dessa

z > 0
hiprbole. Analogamente, a sombra inferior o ramo negativo da
z2 x2
hiprbole de equao: = 1.
( bd
R
)2 d2

interessante observar que, quando a lmpada se situa exatamente


c b
no encontro das diagonais do trapzio da figura 1, ento =
r R
(verifique!), de modo que a sombra superior e a sombra inferior so os
dois ramos de uma mesma hiprbole.
O leitor pode verificar tambm que, se o abajur for cilndrico, as
RPM OBMEP

duas partes da sombra sero tambm os dois ramos de uma mesma


hiprbole. Mais ainda: se o cilindro for eqiltero (altura igual ao
dimetro da base) e a lmpada estiver centralizada, a resultante hiprbole

123
ser equiltera (eixos transverso e no transverso de mesmo com-
primento).

Comentrios
1. No deve ser novidade para muitos leitores que a interseo de um
cone e um plano possa ser, em certos casos, uma hiprbole. Na
verdade, o nome cnicas vem justamente do fato de que a seo de
um cone de duas folhas por um plano , em geral, uma elipse, uma
parbola ou uma hiprbole, conforme o plano seccionador forme com
o eixo do cone um ngulo maior, igual ou menor do que o ngulo que
a geratriz do cone forma com o seu eixo. No caso em questo, o
plano da parede paralelo geratriz do cone, formando portanto um
ngulo nulo, menor do que o ngulo que a geratriz do cone forma
com o seu eixo.
Esse teorema j era conhecido por Apolnio de Perga (sc. III a.C.) e
j foi algumas vezes citado na RPM.

2. Um tema-chave no ensino mdio o ensino de funes (reais de uma


varivel real) e seus grficos. Dentre essas, ningum pode negar a
grande importncia das funes polinomiais de 1o e 2o graus, das
funes logaritmo e exponencial e das funes trigonomtricas.
Muitas vezes, porm, queremos sair um pouco da rotina e apresentar
outras funes que tenham uma definio simples e estejam ligadas
a aplicaes prticas. Est a um interessante exemplo: as funes da

forma f ( x) = k x 2 + d 2 , cujo grfico um ramo de hiprbole. Alis,


uma boa ocasio para pensar tambm nos grficos de funes da

forma f ( x) = k x 2 d 2 ou f ( x) = k d 2 x 2 , cujos grficos so


tambm partes de cnicas (desafio: quais?).

Adaptado do artigo
A sombra do meu abajur
RPM OBMEP

Jos Paulo Q. Carneiro, RPM 59.

124
A ilha do tesouro
Dois problemas e duas solues

Problema 1
O problema a seguir foi inspirado numa histria do livro
Um, dois, trs, ..., infinito de George Gamow.
Era uma vez dois irmos aventureiros que encontraram,
no ba das lembranas de seu bisav, o mapa de um
tesouro, juntamente com as instrues para localiz-lo.
O tesouro estava numa ilha, cuja localizao estava
descrita de forma clara; encontrada a ilha, deveriam
procurar um campo aberto com um grande espao
arenoso, perfeitamente circular. No exterior do dito
crculo encontrariam numerosas palmeiras alinhadas ao
longo de uma reta. Deveriam, ento, procurar a palmeira
com um desenho geomtrico no seu tronco e, partindo
de sua base, traar as tangentes pista circular, chamando
de T1 e T2 os pontos de tangncia. A seguir, deveriam
traar tambm o dimetro, AM, da circunferncia fron-
teira da clareira, perpendicular reta das palmeiras.
Encontrariam o tesouro enterrado exatamente no ponto
de interseco de AM com T1T2.
RPM OBMEP

Os jovens viajaram muito contentes at a ilha, levando


cordas e outras ferramentas necessrias. L estavam a
formosa plancie, a grande clareira circular e a comprida
fila de belas palmeiras. Mas todas as palmeiras

125
apresentavam figuras geomtricas nos seus grossos troncos!
Esse inesperado fato derrubou todos os planos. No sabiam qual era
o ponto inicial e, sem ele, imaginaram que o trabalho seria gigantesco
ou impossvel. Dessa forma tiveram de voltar com as mos vazias
Entretanto, se aqueles aventureiros soubessem um pouco de
Geometria, teriam escolhido uma palmeira qualquer da fila, como ponto
inicial, e teriam encontrado o tesouro. Vejamos por qu.

Na figura:
O e r so, respectivamente, o centro e o raio da circunferncia fronteira
da clareira circular; H o ponto de interseco da reta determinada por
AM com a reta das palmeiras; P o ponto que representa a palmeira
escolhida, eleita para iniciar a procura do tesouro; B o ponto de
interseco de OP com T1T2; T interseco de T1T2 com AM, ponto
onde deveriam cavar para encontrar o tesouro.
Temos ento:
Os tringulos retngulos OBT1 e OT1P so semelhantes.
OT1 OP
Logo, = , ou seja, r 2 = OB.OP .
OB OT1
Analogamente, os tringulos retngulos OBT e OHP so
semelhantes, o que implica:
RPM OBMEP

OT OB
= , ou seja, OT .OH = OB.OP .
OP OH

126
r2
Assim, OT .OH = r 2 ou OT = , o que mostra que a posio
OH
do ponto T independe do ponto P, ou seja, independe da palmeira
escolhida inicialmente.

Problema 2
O problema a seguir foi inspirado em um exerccio do livro
Polynomials, de E. J. Barbeau, e foi apresentado a professores do ensino
mdio, alunos de um curso, de formao continuada, sobre nmeros
complexos.
Dois piratas decidem enterrar um tesouro em uma ilha. Escolhem,
como pontos de referncia, uma rvore e duas pedras. Comeando na
rvore, medem o nmero de passos at a primeira pedra. Em seguida,
dobram, segundo um ngulo reto, direita e caminham o mesmo nmero
de passos at alcanar um ponto, onde fazem uma marca. Voltam rvore,
medem o nmero de passos desde a rvore at a segunda pedra, dobram
esquerda, segundo um ngulo reto, e caminham o mesmo nmero de
passos at alcanar um ponto, onde fazem outra marca. Finalmente,
enterram o tesouro exatamente no ponto mdio entre as duas marcas.
Anos mais tarde, os dois piratas voltam ilha e decidem desenterrar
o tesouro, mas, para sua decepo, constatam que a rvore no existe
mais (o vento, a chuva e os depredadores a haviam arrancado). Ento
um dos piratas decide arriscar. Escolhe ao acaso um ponto da ilha e diz:
Vamos imaginar que a rvore estivesse aqui. Repete ento os mesmos
procedimentos de quando havia enterrado o tesouro: conta os passos at
a primeira pedra, dobra direita, etc., e encontra o tesouro.
A pergunta : esse pirata era sortudo ou um matemtico?
Mesmo tendo sido apresentado em um curso sobre nmeros
complexos, e para alunos que tinham bastante experincia eram
professores de Matemtica , o problema da ilha do tesouro causou uma
comoo. Na verdade, todos admitiram que, se o curso no fosse sobre
RPM OBMEP

nmeros complexos, a nenhum dos presentes teria ocorrido a ideia de


resolver esse problema usando a lgebra dos nmeros complexos. E,
mesmo depois da sugesto para faz-lo, quase ningum conseguiu.

127
Qual a relao entre o problema e os nmeros complexos? Bem,
tudo se baseia em dois fatos fundamentais:
1) no plano complexo, a diferena entre dois complexos traduz o vetor
com origem no primeiro ponto e extremidade no segundo; o que se
JJJG
costuma formular por: AB = B A ;
2) multiplicar um complexo pelo nmero i (a unidade imaginria)
equivale a gir-lo de um ngulo reto positivo.
A figura ilustra a situao do
problema. Sendo A a rvore, e P e
Q as pedras, o tesouro est no ponto
T mdio dos pontos P e Q.
Considerando os pontos pertencentes
ao plano complexo, no importando
onde esteja a origem, tem-se:

P + Q P i ( P A) + Q + i (Q A) P + Q Q P
T= = = +i .
2 2 2 2

Observando que P + Q o ponto mdio


2
JJJG
de PQ e que Q P = PQ , esse resultado
no s demonstra que a localizao do tesouro
independe da posio da rvore (o pirata era
um matemtico...), como tambm permite
localiz-lo como o terceiro vrtice de um dos
tringulos retngulos issceles com
hipotenusa PQ.

Adaptado do artigo
A ilha do tesouro. Dois problemas e duas solues
Jess A. P. Snchez e Jos Paulo Q. Carneiro, RPM 47.
RPM OBMEP

128
Qual o mesmo a definio
de polgono convexo?

Quando pensamos num polgono convexo, imaginamos


seus vrtices todos apontando para fora, ou seja, que ele
no possui vrtices reentrantes. Como os dois polgonos
da esquerda na figura 1.

figura 1: Dois polgonos convexos e dois no convexos.


Essa idia intuitiva necessita, entretanto, uma formulao
mais precisa, para poder ser usada com segurana e
generalidade. Alm disso, h outras maneiras de pensar
num polgono convexo. Conforme o contexto, uma dessas
definies pode ser mais adequada do que as outras. Por
isso conveniente conhecer as principais alternativas e
saber mostrar que elas so equivalentes.
A seguir, daremos trs definies diferentes de polgono
convexo e provaremos a equivalncia entre elas.
Chamamos polgono a uma linha poligonal fechada sem
auto-intersees, isto , cada lado tem apenas um ponto
RPM OBMEP

comum com o lado anterior e com o seguinte, mas no


com os demais.
s vezes, a palavra polgono tambm designa a
regio do plano limitada por essa linha poligonal fechada
129
sem auto-intersees. Por exemplo, quando falamos da rea de um
polgono, claro que nos referimos regio poligonal, no linha que
a limita.
Um subconjunto F do plano chama-se uma figura plana convexa
quando, para quaisquer dois pontos X e Y em F, o segmento de reta
XY est inteiramente contido em F.

figura 2: Duas figuras planas convexas e duas no convexas.

Primeira definio
Um polgono diz-se convexo quando a regio por
ele limitada uma figura plana convexa.
Segue-se desta definio que toda diagonal de um polgono convexo
est inteiramente contida na regio por ele limitada.
Para a segunda definio, lembremos que toda reta r decompe o
plano em duas regies que tm r como fronteira comum. Chamaremos
essas regies as margens de r.
As margens de uma reta so figuras planas convexas. Se os pontos X
e Y esto em margens opostas da reta r, o segmento de reta XY
corta r.
Diz-se que r uma reta de apoio do polgono P quando P tem
pelo menos um ponto em comum com r e situa-se inteiramente numa
das margens de r.
RPM OBMEP

figura 3: r a reta de apoio dos polgonos P1 e P2 mas no de P3 e P4.

130
Segunda definio
Um polgono chama-se convexo quando a reta que
contm qualquer dos seus lados uma reta de apoio.
Por exemplo, dos polgonos na figura 3, apenas P4 convexo.
Para formular a terceira definio de polgono convexo, definimos
um ziguezague ABCD como uma poligonal com trs lados, AB, BC e
CD, dispostos de modo que AB e CD se situem em margens opostas
da reta (que contm o segmento) BC.

figura 4: A poligonal ABCD um ziguezague mas ABCD no .

Terceira definio
Um polgono diz-se convexo quando no contm ziguezagues.
Notemos que se ABCD um ziguezague contido no polgono P,
ento um dos vrtices B, C saliente e o outro reentrante.

figura 5: No ziguezague ABCD, o vrtice C


saliente para o polgono P e reentrante
para Q. O contrrio ocorre com o vrtice B.

Para demonstrar a equivalncia entre estas trs definies de polgono


convexo, usaremos a noo de ponta de um polgono.
Sejam A, B, C vrtices consecutivos do polgono P. Diz-se que B
uma ponta de P quando o segmento AC uma diagonal interna desse
polgono.
RPM OBMEP

figura 6: Os vrtices B e D (mas no A e


C) so pontas de ABCD.

131
Lema
Todo polgono tem pelo menos uma ponta.
Demonstrao
Um polgono P, de n lados, decompe-se, mediante diagonais
internas, em n 2 tringulos justapostos (RPM 18, p. 36). Cada um
dos n lados de P pertence a pelo menos um desses n 2 tringulos.
Pelo princpio da casa dos pombos (RPM 8, p. 21) h 2 lados de P no
mesmo tringulo. O vrtice comum a esses dois lados uma ponta de P.
O teorema seguinte estabelece a equivalncia entre as trs definies
de polgono convexo dadas acima.

Teorema 1
Cada uma das seguintes afirmaes a respeito de um polgono P
implica, as demais:
1) A regio limitada por P uma figura plana convexa;
2) A reta que contm qualquer lado de P uma reta de apoio;
3) P no possui ziguezagues.
Demonstrao
Provaremos as implicaes 1) 2) 3) 1).
1) 2). Admitindo 1), suponhamos,
por absurdo, que 2) seja falsa, isto , que
exista um lado AB do polgono P e pontos
X, Y da regio F limitada por P situados
em margens opostas da reta AB, como na
figura 7.
Sendo F convexa, todos os pontos do segmento XY, e da todos os
pontos do tringulo AXY, obtidos ligando A aos pontos de XY, esto
contidos em F. Ento AB no lado de P. Contradio.
2) 3). Se ABCD um ziguezague, AB e CD esto em margens
RPM OBMEP

opostas da reta BC. Portanto, um polgono onde a reta que contm


qualquer dos seus lados de apoio no pode conter ziguezagues.
3) 1). Para provar esta ltima implicao suponhamos, por
absurdo, que exista um polgono P, com n lados, que no contm

132
ziguezagues mas a regio F, por ela limitada, no uma figura plana
convexa. Tomemos P de modo que n seja o menor possvel. Ento
3) 1) para polgonos com menos de n lados. Pelo lema, existem
vrtices consecutivos L, A, B, C, D de P tais que B uma ponta. A
diagonal AC decompe P em dois polgonos justapostos: o tringulo
ABC e um polgono Q, de n 1 lados, que no contm ziguezagues,
logo limita uma figura plana convexa G.

figura 8: O polgono P decomposto no tringulo ABC e


no polgono Q, de n 1 lados.
Assim, para provar que F uma figura plana convexa, basta tomar
um ponto X no tringulo ABC, um ponto Y na regio G e mostrar que
o segmento de reta XY est contido em F. Como j vimos que 1) 2),
sabemos que AC uma reta de apoio para Q, logo X e Y esto em
margens opostas de AC. Alm disso, como LABC e ABCD no so
ziguezagues, X e Y esto na mesma margem em relao s retas AB e
BC. Tudo isto significa que o segmento XY corta a reta AC mas no as
retas AB ou BC. Noutras palavras, o segmento XY sai do tringulo
ABC por um ponto Z do segmento AC. Ento XZ est contido no
tringulo ABC e ZY est contido na regio G, logo XY est contido
na regio F, como queramos demonstrar.
Para finalizar, breves observaes sobre as definies acima propostas:
1. A primeira definio a que melhor se adapta aos padres atuais da
Matemtica, tanto Pura, como Aplicada. Ela se aplica literalmente a
figuras slidas com um nmero qualquer de dimenses. Dela resulta
facilmente que a interseo de duas ou mais figuras convexas uma
RPM OBMEP

figura convexa. Por isso simples deduzir dela que um polgono


convexo se, e somente se, tem exatamente dois pontos em comum
com qualquer reta que passa pelo seu interior. (Isto seria uma quarta
definio de polgono convexo.)

133
2. A segunda definio tambm se estende a poliedros em espaos com
um nmero qualquer de dimenses. Ela permite caracterizar um
polgono convexo como o conjunto das solues (x, y) de um sistema
de desigualdades lineares do tipo ax + by < c. Por isso desempenha
papel fundamental em Programao Linear.
3. As duas primeiras definies tm carter global enquanto a terceira
nitidamente local. Para verificar se um dado polgono convexo no
sentido das duas primeiras definies necessrio examinar (vrias
vezes) todos os seus lados ao mesmo tempo. J na terceira definio,
para cada lado, olha-se apenas para o lado sua esquerda e para o
lado sua direita. Do ponto de vista computacional, isto bem mais
simples. Por outro lado, a no existncia de ziguezagues s faz sentido
no plano. Alm disso, trata-se de uma hiptese da qual, em que pese
seu grande apelo geomtrico, difcil deduzir conseqncias.
(Compare 3) 1) com as outras implicaes.)

Adaptado do artigo
Qual mesmo a definio de polgono convexo?
Elon Lages Lima, RPM 21.
RPM OBMEP

134
A soluo de Tartaglia para a
equao do 3o grau e a emergncia
dos nmeros complexos

Introduo
A histria da resoluo da equao de terceiro grau
muito pitoresca, plena de lances dramticos, paixes e
disputas pela fama e a fortuna que seu achado poderia
trazer a seus autores.
Uma das personagens dessa histria Niccol Fontana
(1500-1557 aprox.). Em 1512 os franceses saquearam
Brescia, sua cidade natal. Sua me buscou refgio para o
filho na igreja, mas os soldados tambm invadiram o
santurio, e a criana foi ferida no rosto. O ferimento lhe
causou uma gagueira permanente, que lhe valeu o apelido
de Tartaglia (gago, em italiano), pelo qual se tornou
conhecido. Ele no foi o primeiro a obter o mtodo de
resoluo dessas equaes; Scipione del Ferro (1465-
1562 aprox.), que foi professor na Universidade de
Bolonha e cuja biografia pouco conhecida, foi o
verdadeiro descobridor. Antes de morrer, del Ferro
ensinou seu mtodo a dois discpulos, Annibale della
Nave - seu futuro genro e sucessor na ctedra em Bolonha
- e Antnio Maria Fior (ou Floridus, em latim).
RPM OBMEP

Em 1535 houve uma disputa matemtica entre Fior e


Tartaglia. Tais confrontos intelectuais no eram infre-
quentes na poca e, muitas vezes, a permanncia de um
matemtico numa ctedra dependia de seu bom
135
desempenho nesses encontros. Cada um dos adversrios props ao outro
trinta problemas e foi combinado que o perdedor deveria pagar trinta
banquetes ao ganhador. Tartaglia preparou questes variadas, mas todos
os problemas propostos por Fior implicavam equaes do tipo X3 + aX
= b. Precisamente na noite de 12 para 13 de fevereiro, Tartaglia conseguiu
descobrir o mtodo de resoluo de tais equaes e, na hora do confronto,
verificou-se que Tartaglia tinha resolvido todas as questes propostas
por Fior, enquanto este no tinha conseguido resolver a maioria das
questes submetidas por Tartaglia. Declarado vencedor, Tartaglia
voluntariamente renunciou aos trinta banquetes.
A notcia do triunfo de Tartaglia logo se espalhou e chegou aos
ouvidos de Girolamo Cardano (1501-1576), que, na poca, ocupava
uma cadeira de medicina na Universidade de Pavia e era membro do
Colgio Mdico de Milo. De todos os participantes da nossa histria,
talvez seja Cardano o mais enigmtico, aquele cuja vida mais pitoresca
e, certamente, que teve uma formao mais universal.
Para termos uma idia de quo extenso e profundo era seu
conhecimento, citamos a seguir os comentrios de Gabriel Naud (1600-
1653), que publicou a autobiografia de Cardano pela primeira vez em
1643:
No somente era ele inquestionavelmente um mdico notvel,
como foi tambm provavelmente o primeiro e nico homem a se
distinguir em todas as cincias ao mesmo tempo. uma das
ilustraes da Natureza daquilo que um homem capaz de atingir.
Nada de significativo lhe era desconhecido em filosofia, medicina,
astronomia, matemtica, histria, metafsica ou as cincias
sociais, ou em outras reas mais remotas do conhecimento. Ele
tambm errava, claro, isto apenas humano; maravilhoso,
porm, quo raramente ele errava.
Na poca da descoberta de Tartaglia, Cardano gozava de boa posio
em Milo e o convidou a sua casa. Uma vez l, com muita insistncia
RPM OBMEP

Cardano conseguiu que lhe fosse revelado o segredo da resoluo das


equaes do terceiro grau.
Tartaglia consentiu em lhe ensinar a regra de resoluo (embora no
lhe ensinasse a demonstrao da mesma), sob forma de versos, em troca

136
do juramento solene de que Cardano jamais publicaria esse segredo.
Conhecendo um mtodo de resoluo, Cardano procurou e achou
uma demonstrao que o justificasse. De posse da soluo, Cardano
deve ter se sentido fortemente tentado a public-las. Em 1544 fez uma
viagem a Florena e, no caminho, visitou Annibale delia Nave, em
Bologna, que lhe mostrou um manuscrito de del Ferro que continha a
famosa regra de Tartaglia, manuscrito este que ainda se conserva.
Aparentemente, ao saber que a frmula de Tartaglia existia j desde
trinta anos antes, Cardano se sentiu desobrigado de cumprir seu juramento
e publicou, em 1545, em Nuremberg, uma obra intitulada Ars Magna,
que o tornou verdadeiramente famoso em todo o continente. Nas palavras
de C. Boyer, ele provavelmente era o matemtico mais competente da
Europa. Nessa obra aparecem, pela primeira vez, as regras de resoluo
das equaes do terceiro e quarto graus. A seu favor, podemos dizer que
Cardano no esquece de fazer as devidas atribuies de mrito aos
respectivos descobridores.
A seguir, faremos uma anlise do mtodo que Tartaglia confiou a
Cardano.

Os Versos de Tartaglia
Como dissemos acima, Tartaglia comunicou a Cardano o segredo da
sua descoberta por meio de versos. Tal idia no to estranha quanto
pode parecer a princpio; devemos lembrar que, na poca, os autores
no dispunham ainda de uma notao adequada para tratar as equaes
em sua generalidade e no podiam, portanto, expressar seus mtodos
resumidamente mediante frmulas, como fazemos hoje em dia.
A seguir, reproduzimos os versos na sua verso original, tal como
transcritos na edio de 1554 dos Quesiti et inventione diverse de
Tartaglia.
1. Quando chel cubo con le cose appreso
Se aggaglia a qualque nmero discreto
RPM OBMEP

Trovati due altri differenti in esso

2. Depoi terrai questo por consueto


Chel lor produtto sempre sia eguale
Al terzo cubo delle cose neto
137
3. El resduo poi suo generale
Delli lor lati cubi ben sostratti
Verra la tua cosa principale

4. In el secondo de coiesti aiti


Quando chel cubo restasse lui solo
Tu osserverai questaltri contratti

5. Del nmero farai due, tal parta volo


Cha luno e laltro si produca schietto
El terzo delle cose in stelo

6. Delle qual poi, per commun precetto


Torrai li lati cubi incieme gionti
Et cotal somma sera il tuo concetto

7. El terzo poi de questi nostri conti


Se solve con secondo, se ben guardi
Che ser natura son quasi congiontri

8. Questi trovai, et non con passi tardi


nel mille cinquecento quatro et trinta
Nella citt dal mare intorno centa.
Uma traduo para o portugus ficaria, mais ou menos, assim:
1. Quando o cubo com a coisa em apreo
Se igualam a qualquer nmero discreto
Acha dois outros diferentes nisso

2. Depois ters isto por consenso


Que seu produto seja sempre igual
Ao cubo do tero da coisa certo
3. Depois, o resduo geral
Das razes cbicas subtradas
Ser tua coisa principal
RPM OBMEP

4. Na segunda destas operaes,


Quando o cubo estiver sozinho
Observars estas outras redues

138
5. Do nmero fars dois, de tal forma
Que um e outro produzam exatamente
O cubo da tera parte da coisa

6. Depois, por um preceito comum


Toma o lado dos cubos juntos
E tal soma ser teu conceito

7. Depois, a terceira destas nossas contas


Se resolve como a segunda, se observas bem
Que suas naturezas so quase idnticas

8. Isto eu achei, e no com passo tardo


No mil quinhentos e trinta e quatro
Com fundamentos bem firmes e rigorosos
Na cidade cingida pelo mar
Analisaremos, a seguir, esses versos numa linguagem acessvel ao
leitor contemporneo. Antes de tudo, conveniente lembrar que Tartaglia
(assim como depois faria tambm Cardano) no utiliza coeficientes
negativos em suas equaes. Ento, em vez de uma equao geral do
terceiro grau, ele deve considerar trs casos possveis:
x3 + ax = b
x3 = ax + b
x3 + b = ax .
Tartaglia chama cada um desses casos de operaes e afirma que ir
considerar, de incio, equaes do primeiro tipo: cubo e coisa igual a
nmero. No quarto verso comea a considerar o segundo tipo quando
o cubo estiver sozinho e, no stimo, faz referncia ao terceiro caso.
Vejamos agora como se prope a resolver o primeiro caso, nos trs
versos iniciais, para depois justificar seu mtodo, de uma forma simples.
O nmero se refere ao termo independente, que ns denotamos aqui
por b. Quando diz acha dois outros diferentes nisso, est sugerindo
RPM OBMEP

tomar duas novas variveis cuja diferena seja precisamente b, i.e.,


escolher U e V tais que:
U V = b.

139
A frase ... que seu produto seja sempre igual a cubo da tera parte
da coisa significa que U e V devem verificar:
a
UV = ( )3 .
3
Finalmente, o resduo geral das razes cbicas subtradas ser tua
coisa principal significa que a soluo estar dada por
x = 3U 3V .
Os outros dois casos carecem de interesse para o leitor moderno,
uma vez que podemos reduzi-los ao primeiro, mudando termos de um
membro a outro da equao.
A frase final ... a cidade cingida pelo mar uma referncia a Veneza,
onde realizou suas descobertas.

A Resoluo da Equao do Terceiro Grau


Nesta seo veremos como justificar a frmula de Tartaglia para
resolver equaes do terceiro grau. Naturalmente, utilizaremos mtodos
e notaes modernos, o que nos permitir dar uma exposio
relativamente simples.
Vamos considerar uma equao do terceiro grau escrita na forma:
x3 + ax = b.
para compar-la com a primeira destas operaes . . . cubo e coisa igual
a nmero, discutida nos trs primeiros versos de Tartaglia. Na verdade,
h um caminho muito simples para ach-la. Comecemos por lembrar a
frmula do cubo de um binmio:
(u v)3 = u3 3u2v + 3uv2 v3.
Pondo em evidncia o produto uv, temos:
(u v)3 = 3uv(u v) + (u3 v3),
isto ,
(u v)3 + 3uv(u v) = u3 v3 .
RPM OBMEP

Se podemos escolher, de alguma forma, u e v de modo que verifiquem:


uv = a/3
u v3 = b,
3

140
a relao acima se transformar em:
(u v)3 + a(u v) = b
o que significa que x = u v ser uma soluo da equao dada.
Em outras palavras, se conseguirmos achar u e v que sejam solues
do sistema acima, tomando x = u v obter-se- uma soluo da equao
proposta. Resta-nos ento o problema de resolver o sistema. Para isso,
observemos que, elevando ao cubo a primeira equao, ela se transforma
em:
u3v3 = (a/3)3
u3 v3 = b.
Finalmente, fazendo u3 = U e v3 = V, temos:
UV = (a/3)3
U V = b.
Isso muito fcil de resolver; U e V so as razes da equao:
X2 bX + (a/3)3 = 0
que so dadas por:

a 3
b b 2 4( )
3 b b a
X= = ( ) 2 + ( )3 .
2 2 2 3

Podemos tomar uma dessas razes como sendo U e a outra como


V, logo temos u = 3 U e v = 3 V . Portanto, obtemos precisamente a
soluo enunciada por Tartaglia:
x = 3U 3V .
Mais explicitamente, substituindo U e V pelos seus respectivos
valores, resulta a conhecida frmula que, nos textos, chamada de
frmula de Cardano ou de Tartaglia:

b b a b b a
x=3 + ( ) 2 + ( )3 + 3 ( ) 2 + ( )3 .
RPM OBMEP

2 2 3 2 2 3
Uma observao final: a equao geral do terceiro grau, que podemos
escrever na forma:

141
x3 + a1x2 + a2x + a3 = 0,
pode-se reduzir ao caso acima, mediante a mudana de varivel
x = y (a1/3). Alis, essa reduo era conhecida por Tartaglia, mas no
por Fior, e foi justamente esse fato que determinou a vitria do primeiro.
Isso significa que, na verdade, Tartaglia conhecia um mtodo geral para
resolver qualquer equao do terceiro grau.

A emergncia dos nmeros complexos


Os nmeros complexos desempenham um papel sumamente
importante nos mais diversos ramos da Matemtica e, atravs destes,
em muitas das aplicaes a outras reas do conhecimento.
Em geral, o estudante se depara com eles, pela primeira vez, ainda
no curso secundrio e sua introduo justificada pela necessidade de
resolver equaes de segundo grau com discriminante negativo. Isso
cria uma falsa impresso, j que, historicamente, no foram as equaes
de segundo grau que levaram introduo dos nmeros complexos.
Neste texto analisaremos essa questo e alguns outros aspectos ligados
ao desenvolvimento do assunto.
O fato de que um nmero negativo no tem raiz quadrada parece ter
sido sempre claro para os matemticos que se depararam com a questo.
As equaes de segundo grau apareceram na Matemtica j nas
tabuletas de argila da Sumria, aproximadamente 1700 anos antes de
Cristo e, ocasionalmente, levaram a radicais de nmeros negativos;
porm, no foram elas, em momento algum, que sugeriram o uso de
nmeros complexos.
Em rigor, uma equao era vista como a formulao matemtica de
um problema concreto; assim, se no processo de resoluo aparecia uma
raiz quadrada de um nmero negativo, isso era interpretado apenas como
uma indicao de que o problema originalmente proposto no tinha
soluo. Como veremos adiante, foram s as equaes de terceiro grau
que impuseram a necessidade de trabalhar com esses nmeros.
RPM OBMEP

Vejamos inicialmente alguns antecedentes. Na Arithmetica, de


Diophanto, aproximadamente no ano de 275 d.C. ele considera o seguinte
problema:

142
Um tringulo retngulo tem rea igual a 7 e seu permetro de 12
unidades. Encontre o comprimento dos seus lados.
Chamando de x e y o comprimento dos catetos desse tringulo,
temos, na nossa notao atual:
1
xy = 7; x 2 + y 2 = (12 x y ) 2 .
2
Substituindo y em funo de x, obtemos a equao
24x2 172x + 336 = 0.
Nesse ponto Diophanto observa que s poderia haver soluo se
172 2
( ) 24 336 . Nesse contexto, claro que no h necessidade
2
alguma de introduzir um sentido para a expresso 167 , sendo 167
o discriminante da equao.
Na verdade, o primeiro registro de um radical de um nmero negativo
um pouco anterior: ele aparece na Estereometria de Heron, matemtico
grego do perodo Alexandrino, publicada aproximadamente em 75 d.C.
Num clculo sobre o desenho de uma pirmide surge a necessidade de
avaliar 81 144 . A questo parece no causar nenhum problema
simplesmente porque logo em seguida os nmeros apresentam-se
trocados: 144 81 , resultando 63 , que calculado como aproxima-
15
damente igual a 7 .
16
Encontram-se novas referncias questo na Matemtica indiana.
Aproximadamente no ano de 850 d.C, o matemtico indiano Mahavira
afirma:
... como na natureza das coisas um negativo no um quadrado, ele
no tem, portanto, raiz quadrada.
J no sculo XII, o famoso matemtico Bhaskara (1114-1185 aprox.)
RPM OBMEP

escreve:
O quadrado de um afirmativo afirmativo; e a raiz quadrada de um
afirmativo dupla: positiva e negativa. No h raiz quadrada de um
negativo; pois ele no um quadrado.

143
Tambm na Matemtica europia aparecem observaes dessa
natureza; Luca Paccioli, na sua Summa de arithmetica, geomtrica,
proportioni et proportionalita, publicada em 1494, escreve que a equao
1 2
x2 + c = bx solvel somente se b c , e o matemtico francs
4
Nicolas Chuquet (1445-1500 aproximadamente) faz observaes
semelhantes sobre solues impossveis num manuscrito, no
publicado, de 1484.
O prprio Cardano se deparou com esse tipo de questes e, embora
mantivesse a atitude dos seus contemporneos, no sentido de entender
que razes de nmeros negativos indicavam apenas a no-existncia de
solues de um determinado problema, pelo menos em um caso ele deu
um passo a mais. No Captulo 37 do Ars Magna, ele considera o problema
de dividir um segmento de comprimento 10 em duas partes cujo produto
seja 40.

Se chamamos de x o comprimento de uma das partes, a outra ter


comprimento 10 x, e a condio do problema se traduz na equao:
x(10 x) = 40.

Isso leva equao x2 l0x + 40 = 0, cujas solues so 5 15 .


Cardano reconhece que o problema dado no tem soluo mas, talvez a
ttulo de curiosidade, observa que, trabalhando com essas expresses
como se fossem nmeros, deixando de lado as torturas mentais
envolvidas e multiplicando 5 + 15 por 5 15 , obtm-se
25 (15), que igual a 40.
Em consequncia, ele chama essas expresses de razes sofsticas da
equao e diz, a respeito delas, que so to sutis quanto inteis.
RPM OBMEP

A necessidade dos nmeros complexos


Raphael Bombelli (1526-1573) era um admirador da Ars Magna de
Cardano, mas achava que seu estilo de exposio no era claro (ou, em
suas prprias palavras, ma nel dire f oscuro). Decidiu, ento, escrever

144
um livro expondo os mesmos assuntos, mas de forma tal que um
principiante pudesse estud-los sem necessidade de nenhuma outra
referncia. Publicou lAlgebra, em trs volumes, em 1572, em Veneza,
obra que viria a se tornar muito influente. No captulo II dessa obra, ele
estuda a resoluo de equaes de grau no superior a quatro. Em
particular na pgina 294 e nas seguintes, ele considera a equao
x3 = 15x+ 4. Ao aplicar a frmula de Cardano para o clculo de uma
raiz, ele obtm:

x = 3 2 + 121 + 3 2 121 .
Seguindo Cardano, ele tambm chama essa expresso de sofstica,
mas, por outro lado, ele percebe que x = 4 , de fato, uma raiz da
equao proposta.
Assim, pela primeira vez, nos deparamos com uma situao em que,
apesar de termos radicais de nmeros negativos, existe verdadeiramente
uma soluo da equao proposta. necessrio, ento, compreender o
que est acontecendo.
Bombelli concebe ento a possibilidade de que exista uma expresso
da forma a + b que possa ser considerada como raiz cbica de

2 + 121 i.e., que verifique (a + b )3 = 2 + 121 . A forma em


que ele calcula essa raiz um tanto peculiar; ele assume que a raiz cbica
de 2 121 seja da forma a b . Como ele sabe que 4 deve ser
raiz da equao, necessariamente a + b + a b = 4 . Nesse ponto,
felizmente, as quantidades no existentes se cancelam e obtemos
a = 2. Com esse resultado, muito fcil voltar equao
(a + b )3 = 2 + 121 e deduzir que b = 1. Assim, ele obtm que
3
2 + 121 = 2 + 1 e que:

x = 2 + 1 + 2 1 = 4
RPM OBMEP

uma soluo da equao dada.


Bombelli percebeu claramente a importncia desse achado. Ele diz:
Eu achei uma espcie de raiz cbica muito diferente das outras, que
aparece no captulo sobre o cubo igual a uma quantidade e um nmero.
145
... A princpio, a coisa toda me pareceu mais baseada em sofismas que
na verdade, mas eu procurei at que achei uma prova... .
Isto pode parecer muito sofisticado mas, na realidade, eu tinha essa
opinio, e no pude achar a demonstrao por meio de linhas [i.e.
geometricamente], assim, tratarei da multiplicao dando as regras
para mais e menos.
Ele utiliza a expresso pi di meno para se referir ao que ns
denotaramos como +i e meno di meno para i. Ele enuncia ento o
que chama de regras do produto, que citamos abaixo junto com sua
traduo na nossa simbologia:
Pi via pi di meno fa pi di meno, +.(+i) = +i
Meno via pi di meno fa meno di meno, .(+i) = i
Pi via meno di meno fa meno di meno, +.(i) = i
Meno via meno di meno fa pi di meno, .(i) = +i
Pi di meno via pi di meno fa meno, (+i).(+i) =
Meno di meno via pi di meno fa pi, (i).(+i) = +
Meno di meno via meno di meno fa meno. (i).(i) =
E interessante notar que Bombelli se deparava com a dificuldade
adicional de no dispor de uma boa notao. Ele utilizava p (plus) para
indicar a soma; m (minus) para a subtrao; R (radix) para raiz quadrada
e R3 para a raiz cbica. Tambm no dispunha de parnteses; nos seus
manuscritos sublinhava expresses para indicar quais os termos afetados
3
por um radical. Assim, por exemplo, a expresso 2 + 121 era

escrita na forma R3 | 2 pR| 0 121 | | .


Note que, como no escrevia diretamente nmeros negativos, ele
escreveu 121 como 0 121. Dessa forma, a soluo da equao
discutida acima aparecia como: R3 | 2 pR| 0 121 | | pR3 | 2mR| 0 121 | | .
RPM OBMEP

Progressos ulteriores
Faremos aqui um pequeno resumo da evoluo dos nmeros
complexos, para que o leitor tenha uma viso global da histria do

146
assunto. Comearemos listando alguns progressos na notao para depois
nos ocuparmos da evoluo dos conhecimentos.

O smbolo 1 foi introduzido em 1629 por Albert Girard.


O smbolo i foi usado pela primeira vez para representar 1 por
Leonhard Euler em 1777, apareceu impresso pela primeira vez em
1794 e se tornou amplamente aceito aps seu uso por Gauss em 1801.
Os termos real e imaginrio foram empregados pela primeira vez por
Ren Descartes em 1637.
O expresso nmero complexo foi introduzida por Carl Friederich
Gauss em 1832.
Como observamos na seo anterior, a partir do trabalho de Bombelli,
os nmeros complexos comearam a ser utilizados devido a sua bvia
utilidade para resolver equaes de terceiro grau mas, ao mesmo tempo,
era claro que tais nmeros no poderiam existir. A primeira tentativa de
legitimao, via uma interpretao geomtrica, devida a John Wallis
(1616-1703), contemporneo de Newton e professor na Universidade
de Oxford. Em 1673 ele publicou um tratado intitulado lgebra, em
cujo captulo LXVI discute a impossibilidade da existncia de
quantidades imaginrias e compara essa questo com a da existncia
de quantidades negativas.
Essas quantidades imaginrias (como so freqentemente chamadas)
surgem das supostas razes de um quadrado negativo (quando aparecem)
e se considera que implicam que o caso proposto impossvel.
E assim , de fato, no sentido estrito do que foi proposto. Pois no
possvel que qualquer nmero (negativo ou afirmativo), multiplicado
por si mesmo, possa produzir (por exemplo) 4. Pois sinais iguais
(tanto + quanto ) produziro +; e portanto no 4.
Mas tambm impossvel que qualquer quantidade (embora no um
suposto quadrado) possa ser negativa. Pois no possvel que qualquer
magnitude possa ser menos que nada, ou qualquer nmero menor que
nada.
RPM OBMEP

Porm, no esta suposio (das quantidades negativas) nem intil


nem absurda, quando corretamente compreendida. E, embora para a
simples notao algbrica representa uma quantidade menor do que

147
nada, quando se trata de uma aplicao fsica, denota uma quantidade
to real como se o sinal fosse +; mas interpretada no sentido contrrio.
Depois de considerar diversos exemplos de nmeros negativos
interpretados em termos de segmentos sobre uma reta orientada, ele tenta
uma interpretao para as quantidades imaginrias:
Suponhamos que num local ganhamos do mar 30 acres, mas perdemos
em outro local 20 acres: se agora formos perguntados quantos acres
ganhamos ao todo a resposta 10 acres, ou +10 (pois 30 20 = 10).
... Mas se num terceiro local perdemos mais 20 acres, a resposta deve
ser 10 (pois 30 20 20 = 10) ... . Mas agora, supondo que esta
plancie negativa de 1600 square perches [20 acres correspondem a
1600 square perches, uma outra medida inglesa da poca] tem a forma
de um quadrado, no devemos supor que este quadrado tem um lado?
E, assim, qual ser esse lado?

No podemos dizer que 40, nem 40 ... Mas sim que 1600 (a
suposta raiz de um quadrado negativo) ou 10 16 ou 20 4 ou
40 1 .
Como era de se esperar, essa interpretao no teve uma grande
acolhida entre seus contemporneos e nenhuma repercusso posterior.
Notemos que, at aqui, nada garante que razes cbicas - ou, em
geral, razes n-simas de complexos sejam, de fato, complexos. Tal
como assinala M. Kline, no comeo do sculo XVIII, a maioria dos
matemticos ainda acreditava que razes de diferente ordem de nmeros
complexos levariam introduo de diferentes tipos de complexos.
Jean Le Rond dAlembert (1717-1783), aps estudar Direito e
Medicina, decidiu dedicar sua vida Matemtica. Trabalhou em lgebra,
clculo e suas aplicaes, equaes diferenciais ordinrias e parciais,
funes de varivel complexa, mecnica e dinmica. Em 1747 publicou
Reflxions sur Ia cause gnrale des vents, em que afirmou que toda
expresso construda algbricamente a partir de um nmero complexo
RPM OBMEP

(onde inclua tambm a extrao de razes) da forma a + b 1 . No


formulou uma prova satisfatria no caso de expresses da forma
(a + bi)c+di, tarefa que seria completada por Euler.

148
DAlembert foi amigo de Voltaire e colaborou com diversos artigos
para a Enciclopdie, mas manteve nessa um discreto silncio sobre os
nmeros complexos.
Roger Cotes (1682-1716) foi um jovem professor no famoso Trinity
College de Cambridge e, aps sua prematura morte, dele disse Newton:
Se Cotes tivesse vivido, teramos aprendido alguma coisa. Em 1714 ele
obteve um importante resultado, relacionado com a obteno de razes
n-simas da unidade que, na notao moderna, poderamos explicitar
como:
loge(cos + isen) = i.
Isso poderia ter levado famosa relao de Euler:
cos + isen = ei.
que, por sua vez, implica a frmula de De Moivre:
(cos + isen)n = cos(n) + isen(n)
o que resolveria o problema de achar razes.
Porm, o caminho foi outro. Abraham De Moivre (1667-1754)
nasceu na Frana, mas viveu na Inglaterra a partir dos dezoito anos.
Estudou Matemtica sozinho, aps ler os Principia de Newton, chegando
a se tornar membro da Royal Society e das academias de Paris e Berlim.
Seu trabalho versou fundamentalmente sobre trigonometria,
probabilidade e clculo de anuidades. Em 1722, utilizando fatos que j
havia publicado em 1707, ele obteve um resultado que implicou a frmula
que leva seu nome, embora tenha se limitado a casos particulares e nunca
tenha chegado a enunciar ou demonstrar a frmula no caso geral.
Essa tarefa coube a Leonhard Euler (1707-1783), considerado o
mais prolfico matemtico de todos os tempos. Numa carta endereada
a Jean Bernoulli, datada de 18 de outubro de 1740, ele afirma que
y = 2 cos e y = eix + eix eram ambas solues da mesma equao
diferencial (o que reconheceu atravs do desenvolvimento em srie das
solues) e que, portanto, deviam ser iguais. Publicou esse resultado em
RPM OBMEP

1743; explicitamente:
ei + e i ei e i
cos = e sen = .
2 2i

149
Em 1748 ele redescobriu o resultado de Cotes, demonstrou a frmula
de De Moivre e estendeu sua validade para todo exponente n real. Com
isso, a existncia de razes no campo complexo ficou definitivamente
estabelecida.
Obviamente, Euler compreendia e utilizava muito bem os nmeros
complexos. O fato de ele prprio ter grandes dvidas quanto a sua
legitimidade ilustra claramente o status desse corpo numrico na poca.
Em Vollstndige Anleitung zur Algebra, publicada primeiro em russo,
em 1768-69, e depois em alemo, em 1770, que se tornou uma referncia
clssica nessa rea nos dois sculos seguintes, Euler escreve:
Uma vez que todos os nmeros concebveis so maiores do que 0, ou
menores do que 0 ou iguais a 0, claro que a raiz quadrada de um
nmero negativo no pode ser includa entre os nmeros possveis.
Consequentemente, devemos dizer que esses so nmeros impossveis.
E essa circunstncia nos conduz a tais nmeros, que por sua natureza
so impossveis, e que so chamados costumeiramente de imaginrios,
pois eles s existem na imaginao.

A representao grfica
A representao geomtrica dos nmeros complexos mediante pontos
do plano foi decisiva para sua aceitao. A possibilidade dessa repre-
sentao era clara para vrios autores, como Cotes, De Moivre, Euler e
Vandermonde; todos eles tentaram resolver a equao xn 1 = 0 pensando
em suas solues como vrtices de um polgono regular de n lados.
Essa ideia era ainda incompleta, pois nenhum desses autores achou
tambm uma interpretao geomtrica para as operaes com complexos.
O primeiro a formular uma tal interpretao foi um agrimensor
noruegus chamado Caspar Wessel (1745-1818), um autodidata. Ele
autor de um artigo intitulado Sobre a representao analtica da direo:
uma tentativa, que foi publicado em 1799 nas memrias da Real
Academia da Dinamarca. Ali, escreveu:
Vamos designar por +1 a unidade retilnea positiva, por + outra
RPM OBMEP

perpendicular primeira, com a mesma origem; ento o ngulo de


direo de +1 ser 0, o de 1 ser 180, o de ser 90 e o de
ser 90 ou 270.

150
Tal como fazemos hoje em dia, ele representa o complexo a + bi
pelo vetor do plano com origem O a origem do sistema de eixos coor-
denados e com extremo no ponto P de coordenadas (a, b). Depois d
uma representao geomtrica da soma de dois complexos a + bi e
c + di, representando-os pelos vetores OP e OQ, respectivamente, e
observando que a soma estar respresentada pela diagonal do
paralelogramo construdo sobre OP e OQ.
De forma anloga, o produto desses complexos estar representado
por um vetor OR tal que o comprimento de OR o produto dos
comprimentos de OP e OQ, e o ngulo que OR forma com o eixo Ox
igual soma dos ngulos formados por OP e OQ com esse eixo.
Uma representao semelhante foi dada por Jean-Robert Argand
(1768-1822), um bibliotecrio suo, tambm autodidata, que em 1806
publicou um pequeno livro intitulado Essai sur la manire de reprsenter
les quantits imaginaires dans les constructions gomtriques. Ele
observa que se multiplicamos +1 por i obtemos i e se multiplicamos
esse resultado novamente por i obtemos 1. Ele pensa, ento, em
representar i por uma operao que aja de modo anlogo. Assim, podemos
representar i por uma rotao de 90 em sentido anti-horrio.
A partir daqui, tal como Wessel, ele d interpretaes para nmeros
da forma a + bi e para as operaes com complexos, aplicando seus
resultados demonstrao de teoremas de lgebra, geometria e
trigonometria.
Esses trabalhos tiveram pouco ou nenhum efeito sobre os matemticos
da poca; a memria de Wessel s foi notada quando publicada em
traduo francesa em 1897, e o livro de Argand, embora causasse uma
certa controvrsia, teve pouco impacto, talvez por ser a nica contribuio
de seu autor Matemtica. Quem verdadeiramente tornou a interpretao
geomtrica amplamente aceita foi Carl Friederich Gauss (1777-1855).
A julgar pelas suas demonstraes do teorema fundamental da lgebra,
ele j conhecia a interpretao grfica dos complexos em torno de 1815,
RPM OBMEP

embora escrevesse, numa carta de 1825, que a verdadeira metafsica de


1 elusiva. Finalmente, em 1831, ele escreveu um artigo muito
explcito sobre a questo. Diz na introduo:

151
O autor tem considerado h vrios anos essa parte importante da
Matemtica sob um ponto de vista diferente, que permite conferir s
quantidades imaginrias, como as negativas, uma existncia objetiva.
O significado intuitivo dos nmeros complexos fica completamente
estabelecido e no se precisa mais para admitir estas quantidades no
domnio da aritmtica.

Ele observa tambm que se as unidades 1, 1, 1 no fossem


chamadas de positiva, negativa e imaginria, mas direta, inversa e lateral,
as pessoas no teriam tido a impresso de que h algo de misterioso
nesses nmeros.
A observao de Gauss a respeito da existncia objetiva dos nmeros
complexos ilustra a viso da Matemtica na poca. Parece que o fato de
esses nmeros poderem ser representados geometricamente lhes d essa
existncia. Em outras palavras, parece que, para os matemticos daquele
perodo, os entes geomtricos tinham um tipo de realidade que faltava
aos objetos da aritmtica.
Finalmente, a formalizao completa dos nmeros complexos como
pares ordenados de nmeros reais ser desenvolvida por William Rowan
Hamilton (1805-1865) em 1833, e ainda Agustin Cauchy (1789-1857)
daria outro tipo de formalizao em 1847.

Adaptado dos artigos


A soluo de Tartaglia para a equao do terceiro grau
Csar Polcino Milies, RPM 25.
A emergncia dos nmeros complexos
Csar Polcino Milies, RPM 24.
RPM OBMEP

152
Grandezas incomensurveis
e nmeros irracionais

Existem, em Matemtica, conceitos que parecem muito


simples a uma viso superficial, mas que, submetidos a
uma anlise mais cuidadosa, revelam aspectos verdadeira-
mente surpreendentes.
Vamos tratar aqui da reta na sua representao numrica
em termos das abscissas de seus pontos para mostrar que
esses conceitos de reta e de nmero no tm uma simpli-
cidade to inocente como parecem revelar a uma viso
menos profunda. Exploraremos alguns fatos notveis e
inesperados, que esto ligados primeira grande crise
do desenvolvimento da Matemtica, ocorrida no final do
5o sculo a.C.
Uma questo com que lidavam os matemticos gregos
daquela poca era a de comparar grandezas da mesma
espcie, como dois segmentos de reta, duas reas ou dois
volumes. No caso de dois segmentos retilneos AB e
CD, dizer que a razo AB/CD o nmero racional
m/n, significava para eles (e ainda significa para ns)
que existia um terceiro segmento EF tal que AB fosse
m vezes EF e CD n vezes esse mesmo segmento EF.
RPM OBMEP

Na figura 1 ilustramos essa situao com m = 8 e n = 5.


A B
AB 8
figura 1 =
CD 5
C D E F

153
No tempo de Pitgoras (580 500 a.C. aproximadamente) e mesmo
durante boa parte do 5o sculo a.C. pensava-se que os nmeros racionais
fossem suficientes para comparar segmentos de reta; isto , dados dois
segmentos AB e CD, seria sempre possvel encontrar um terceiro
segmento EF contido um nmero inteiro de vezes em AB e outro
nmero inteiro de vezes em CD, situao esta que descrevemos dizendo
que EF um submltiplo comum de AB e CD. Uma simples reflexo
revela que essa uma idia muito razovel. Afinal, se EF no serve,
podemos imaginar um segmento menor, outro menor ainda, e assim por
diante. Nossa intuio geomtrica parece dizer-nos que h de existir um
certo segmento EF, talvez muito pequeno, mas satisfazendo aos
propsitos desejados. Na figura 2 ilustramos uma situao com segmento
EF bem menor que o da figura 1. O leitor deve ir muito alm, imaginando
um segmento EF to pequeno que nem se possa mais desenhar, para se
convencer, pela sua intuio geomtrica, da possibilidade de sempre
encontrar um submltiplo comum de AB e CD.
A B
AB 29
=
C DE F CD 26

figura 2
Dois segmentos nessas condies so ditos comensurveis, justamente
por ser possvel medi-los ao mesmo tempo, com a mesma unidade EF.
Entretanto, no verdade que dois segmentos quaisquer sejam sempre
comensurveis. Em outras palavras, existem segmentos AB e CD sem
unidade comum EF, os chamados segmentos incomensurveis. Esse
um fato que contraria nossa intuio geomtrica, e por isso mesmo a
descoberta de grandezas incomensurveis na Antiguidade representou
um momento de crise no desenvolvimento da Matemtica.
Foram os prprios pitagricos que descobriram grandezas incomen-
surveis, provavelmente entre 450 e 400 a.C.; e, ao que tudo indica, isto
se fez atravs de um argumento geomtrico, como o que apresentaremos
RPM OBMEP

a seguir, demonstrando que o lado e a diagonal de um quadrado so


segmentos incomensurveis.

154
C A

E
F
D
B
figura 3
Na figura 3 representamos um quadrado com diagonal = AB e
lado = AC. Suponhamos que e sejam comensurveis. Ento
existir um terceiro segmento que seja submltiplo comum de e .
p com centro
Fazemos agora a seguinte construo: traamos o arco CD
em A e o segmento ED tangente a esse arco em D. Ento, nos
tringulos retngulos ACE e ADE, os catetos AC e AD so iguais e
a hipotenusa AE comum, logo so tambm iguais os catetos CE e
DE (= BD).
Portanto,
= AB = AD + BD = + BD
= BC = BE + EC = BE + BD
ou seja,
= + BD (1)
= BE + BD. (2)
Se um segmento submltiplo comum de e , conclumos, por
(1), que tambm submltiplo de BD. Daqui e de (2) segue-se que
tambm submltiplo de BE. Provamos assim que se houver um
segmento que seja submltiplo comum de = AB e = AC, ento
o mesmo segmento ser submltiplo comum de BE e BD, segmentos
esses que so a diagonal e o lado do quadrado BDEF. Ora, a mesma
construo geomtrica que nos permitiu passar do quadrado original ao
quadrado BDEF pode ser repetida com este ltimo para chegarmos a
um quadrado menor ainda; e assim por diante, indefinidamente; e esses
RPM OBMEP

quadrados vo se tornando arbitrariamente pequenos, pois, como fcil


ver, as dimenses de cada quadrado diminuem em mais da metade quando
passamos de um deles a seu sucessor. Dessa maneira, provamos que o

155
segmento dever ser submltiplo comum do lado e da diagonal de um
quadrado to pequeno quanto desejemos. Evidentemente, isso um
absurdo! Somos, pois, levados a rejeitar a suposio inicial de que o
lado AC e a diagonal AB do quadrado original sejam comensurveis.
Conclumos, pois, que o lado e a diagonal de qualquer quadrado so
grandezas incomensurveis C.Q.D.
A descoberta dos incomensurveis representou, no 5o sculo a.C.,
uma derrota para os pitagricos. De fato, para eles o nmero era a essncia
de tudo. Eles acreditavam na possibilidade de explicar todos os
fenmenos do mundo sensvel em termos dos nmeros e de suas relaes,
tanto na Geometria como na Msica, na Astronomia ou na Fsica, enfim,
o nmero seria a essncia ltima do ser e de todos os fenmenos. Mas
por nmero eles entendiam apenas o que chamamos hoje de nmeros
naturais, ou inteiros positivos: 1, 2, 3, 4, .... Nem as fraes eram
nmeros, j que elas apareciam como relaes entre grandezas da mesma
espcie. Agora que haviam sido descobertas grandezas incomensurveis,
estava claro que os nmeros (naturais) eram insuficientes at mesmo
para definir a razo entre duas grandezas, o que se constitua num srio
entrave Filosofia Pitagrica.
Ao mesmo tempo em que essas coisas aconteciam, outros argumentos
propostos pelos filsofos da poca dentre os quais os de Zeno so os
mais famosos tambm apontavam dificuldades na suposta harmonia
entre a Geometria e os nmeros. Tudo isso culminou numa crise no
desenvolvimento da Matemtica, crise essa que s foi definitivamente
superada com a criao da teoria dos nmeros reais (racionais e
irracionais) no sculo passado, devido, sobretudo aos trabalhos do
matemtico alemo Richard Dedekind (1831-1916).
Uma consequncia da existncia de grandezas incomensurveis a
existncia de pontos na reta sem abscissas racionais.
A
RPM OBMEP

O U P
figura 4

156
De fato, com referncia figura 4, basta tomar OP = AO, onde AO
a diagonal de um quadrado de lado unitrio OU. Como OP e OU
so incomensurveis, no possvel expressar a razo OP/OU como
um nmero racional.
Que nmero seria a abscissa de P? Pelo teorema de Pitgoras,
OA2 = OU2 + UA2.
Como AO = OP e UA = OU = 1, obtemos
OP2 = 2OU2 = 2
ou seja, OP = 2 .
essa a abscissa de P, tomando OU como unidade de comprimento.
interessante analisar essas questes do ponto de vista moderno dos
nmeros como abscissas dos pontos de uma reta. Para maior simplicidade,
vamos restringir-nos apenas a uma semirreta OU, tomando o segmento
OU como unidade de comprimento (figura abaixo).
O U P
0 1 x
figura 5
Ento, todo ponto P da semirreta, que no seja a origem O, tem
abscissa positiva x, que a razo OP/OU.
Evidentemente, se todos os pares de segmentos OU e OP fossem
comensurveis, bastariam os nmeros racionais no-negativos para
caracterizar os pontos da semirreta, isto , os nmeros da forma m/n,
com m e n inteiros, m > 0 e n > 0. E bom observar que isso condiz
muito bem com nossa intuio geomtrica: afinal, esses nmeros ficam
densamente distribudos ao longo da semirreta, de tal forma que entre
dois deles h sempre uma infinidade de nmeros do mesmo tipo. Assim,
entre os pontos A e B de abscissas 7 e 8 existem 9 nmeros do tipo
n ,
7+
10
RPM OBMEP

com n variando de 1 a 9. Isso porque dividimos o intervalo AB em 10


subintervalos de comprimento 1/10 cada um (figura 6). Mas podemos

157
dividir esse intervalo em 100 subintervalos, cada um de comprimento
0,01; ou 1000 subintervalos, cada um de comprimento 0,001;
A B
7 7,09 7,5 7,630589 8
figura 6
e assim por diante. Se, digamos, adotarmos a diviso em 1.000.000
subintervalos iguais, encontraremos entre A e B, 999.999 pontos com
abscissas racionais do tipo
n
7+ ,
1.000.000
com n variando de 1 at 999 999. Na figura 6 ilustramos um desses
pontos, aquele que tem abscissa 7,630598.
Pois bem, vamos confiar ainda que provisoriamente na suposio
de que todos os pontos da semirreta tenham abscissas racionais e ver
onde isso nos leva. Uma primeira consequncia que os pontos da semi-
reta formam um conjunto enumervel, pois o conjunto dos nmeros
racionais enumervel (ver textos de Anlise Real). Se r1, r2, r3, ...
uma enumerao dos racionais, faremos uma cobertura da semirreta por
meio de segmentos, da seguinte maneira: cobrimos o ponto r1 com um
segmento de comprimento c/2, centrado em r1; cobrimos r2 com um
segmento de comprimento c/22, centrado em r2; fazemos o mesmo
com r3, utilizando agora um segmento de comprimento c/23; com r4
utilizamos um segmento de comprimento c/24; e assim por diante.
Dessa maneira a semirreta ficar toda coberta com uma famlia infinita
de segmentos no necessariamente disjuntos.
Vamos agora somar os comprimentos dos segmentos dessa famlia.
Por simplicidade e para enfatizar a visualizao geomtrica colocamos
os segmentos em fila, um em seguida ao outro e na ordem em que
aparecem, como ilustra a figura 7. Isso o bastante para nos convencer
de que a soma de todos os seus comprimentos exatamente igual a c,
RPM OBMEP

pois comeamos com um segmento de comprimento c/2, adicionamos


sua metade, depois a metade deste ltimo e assim por diante.

158
O c

c/2
c/22
c/23
figura 7
O que acabamos de demonstrar uma impossibilidade! Certamente
no possvel cobrir a semirreta com um a famlia de segmentos cuja
soma total dos comprimentos seja um nmero finito c! (E o nmero c
arbitrrio!) Afinal, a semirreta tem comprimento infinito! Para sairmos
dessa contradio temos de voltar atrs em nossa hiptese inicial de que
os pontos da reta numrica tm todos eles abscissas racionais. Em outras
palavras, os nmeros racionais so insuficientes para marcar todos os
pontos de uma reta; ou ainda, em termos mais inteligveis aos gregos da
Antiguidade, existem segmentos AB e CD para os quais impossvel
encontrar um segmento EF que seja submltiplo comum de AB e CD.
Como se v, acabamos de estabelecer a existncia de segmentos
incomensurveis com um raciocnio tpico da Anlise moderna! Ele
certamente causaria, na Antiguidade, tanta controvrsia quanto causaram
os famosos argumentos de Zeno. Talvez mais ainda, pois os argumentos
de Zeno foram rebatidos por Aristteles que, atravs de seus escritos,
f-los chegar at ns. Mas como rebater o argumento que demos acima
sobre a cobertura dos pontos de abscissas racionais? Seria necessrio
admitir a existncia de uma infinidade muito maior (uma infinidade no
enumervel) de pontos sem abscissas racionais! claro que isso seria
totalmente inaceitvel para quem j tinha srias objees ao infinito
enumervel. Mesmo hoje muito surpreendente que se possa cobrir
todos os pontos de abscissas racionais numa reta com uma famlia de
segmentos cuja soma total dos comprimentos seja to pequena quanto
desejemos!
Esses pontos da reta sem abscissas racionais tm por abscissas
nmeros irracionais (desde que esses nmeros sejam criados!) e 2
RPM OBMEP

um deles, como decorre do argumento que demos antes referente


figura 4. No entanto, para completar este artigo, vamos reproduzir aqui
a demonstrao desse fato com um argumento puramente numrico e
bem conhecido.

159
Comeamos supondo que existisse uma frao irredutvel m/n tal
que 2 = m / n . Ento
m 2 m2 = 2n2.
2=
n2
Daqui segue-se que m2 um nmero par, portanto o mesmo verdade
para m, isto , m = 2r, sendo r outro nmero inteiro. Substituindo
m = 2r em m2 = 2n2 obtemos
4r2 = 2n2 n2 = 2r2.
Mas essa ltima relao nos diz que n2 nmero par, logo n tambm
par. Chegamos a um absurdo, pois m/n frao irredutvel, no sendo
possvel que m e n sejam ambos pares. Somos, assim, forados a
rejeitar a suposio inicial de que 2 seja um nmero racional m/n.
A demonstrao que acabamos de dar est baseada num argumento
que, segundo Aristteles, teria sido usado na descoberta de grandezas
incomensurveis. um argumento que encerra um alto grau de abstrao,
razo pela qual muitos historiadores da Cincia acreditam que a
descoberta dos incomensurveis tenha ocorrido com um raciocnio mais
concreto, como o argumento geomtrico da figura 3.
Demonstraes como as que apresentamos, da incomensurabilidade
do lado e da diagonal do quadrado, ou da irracionalidade de 2 , foram
as primeiras demonstraes por reduo ao absurdo que se fizeram na
Antiguidade. notvel que por volta de 400 a.C. a Matemtica j tivesse
alcanado to avanado grau de sofisticao. O mesmo no aconteceu
com outras cincias, como a Fsica, que somente no sculo XVII, com
os trabalhos de vrios cientistas, notadamente Galileu e Newton,
alcanaria desenvolvimento comparvel ao da Matemtica de dois
milnios antes.
Finalmente, um ltimo comentrio sobre a crise desencadeada com a
descoberta dos incomensurveis. De imediato isso tornou impossvel
RPM OBMEP

falar em razo entre duas grandezas quando essas fossem incomen-


surveis.

160
Havia a necessidade de se inventarem os nmeros irracionais, o que
s ocorreu nos tempos modernos. Mas os gregos souberam contornar
esse problema, logo na primeira metade do 4o sculo a.C., e com muita
genialidade! Foi Eudoxo (408? 355? a.C.), da escola de Plato, quem
desenvolveu, de maneira brilhante, uma teoria das propores, com a
qual foi possvel superar a dificuldade dos incomensurveis, usando
apenas os nmeros inteiros positivos. Mas isso uma outra histria...

Adaptado do artigo
Grandezas incomensurveis e nmeros irracionais
Geraldo vila, RPM 05.

RPM OBMEP

161
A outra face da moeda honesta

No outro dia, brincava com a minha filha Ana Letcia o


seguinte jogo: cada uma tinha um peo avanando por
uma linha de um tabuleiro de xadrez. A regra para avanar
baseava-se no lanamento de uma moeda honesta: se
sasse cara, o peo avanava uma casa, se fosse coroa,
avanava duas casas. Quem chegasse primeiro ao fim (ou
passasse) da linha, ganhava.
O jogo muito rpido, de modo que depois de algumas
revanches, a Ana me disse: u, no ltimo lanamento s
sai coroa?.
De fato, comecei a perceber que, no ltimo lanamento,
o nmero de coroas era muito maior que o de caras. No
entanto, ao observar todos os lanamentos, o nmero de
caras e o de coroas eram praticamente iguais, o que indica
a honestidade da moeda.
A Ana com seus sete anos foi brincar de outra coisa e eu
fiquei pensando no assunto: o lanamento final poder
ser cara somente se o peo estiver na penltima casa, j
que com cara o peo avana apenas uma casa. Enquanto
que poder ser coroa, tanto se o peo estiver na penltima
RPM OBMEP

quanto na antepenltima casa.


Portanto, a face coroa tem realmente mais chance de
aparecer no ltimo lanamento que a face cara!!!

162
Mais formalmente, denotemos por N o nmero de lanamentos
necessrios para chegar oitava casa (ou passar), por SN1 a posio do
peo antes do lanamento final, e por XN a face nele observada. Se
SN1 = 7, ento o lanamento seguinte pode ser cara ou coroa, com
mesma chance. Por outro lado, SN1 = 6 implica que o lanamento
seguinte deva ser coroa, seno este no seria o ltimo.
Assim, pela lei da probabilidade total,
1
P ( X N = cara ) = P ( S N 1 = 7) (1)
2
1
P( X N = coroa ) = P ( S N 1 = 7) + P ( S N 1 = 6) 1
2
> P ( X N = cara ).
A ltima desigualdade encerra o assunto, mas eu ainda queria
conhecer a probabilidade de obter coroa no ltimo lanamento.
O evento [SN1 = 6] ocorre se e somente se forem observadas as
seguintes seqncias nos lanamentos:
{(2, 2, 2, 2), (1, 1, 2, 2, 2), (1, 1, 1, 1, 2, 2), (1, 1, 1, 1, 1, 1, 2)}
ou suas possveis permutaes (lembrando que o ltimo lanamento deve
ser coroa). Portanto,
1 4 1 5 1 1
P ( S N 1 = 6) = 4
+ 5 + 6 + 7 ,
2
2 2 2 1 2
que aproximadamente 1/3. Como a penltima posio s pode ser 6
ou 7, ento P(SN1 = 7) aproximadamente 2/3.
Substituindo esse resultado em (1), temos que a probabilidade de
obter coroa no lanamento final aproximadamente 2/3.
A primeira concluso disso tudo que minha filha algo exagerada
em seus comentrios. A segunda que moeda honesta quando chega ao
fim da linha...
RPM OBMEP

Comentrios
O problema abordado neste texto uma verso discreta do conhecido
Paradoxo do tempo esperado, definido originalmente no contexto de
processos de renovao a tempo contnuo.

163
A novidade nesta apresentao que inspira a construo fsica de
um mecanismo que permite visualizar esse resultado: por exemplo,
mediante uma trilha na qual se avana de acordo com o lanamento de
um dado equilibrado, ou a prpria corrida de oito passos no tabuleiro de
xadrez.
A reao usual das pessoas de espanto ao constatar o desequilbrio
da distribuio obtida, ou de dvida a respeito da honestidade da moeda.
Mesmo sendo contra-intuitivo, o resultado natural se levarmos em
considerao que h outra varivel aleatria envolvida no processo, que
o nmero de lanamentos necessrios para se alcanar o fim da
linha, N.
Se bem verdade que o resultado de qualquer lanamento tpico,
XN, uniforme no conjunto {cara, coroa} e independente dos demais
lanamentos, o mesmo no ocorre com o lanamento final, XN. A varivel
aleatria XN no independente dos lanamentos anteriores, depende
da penltima posio.
Uma outra forma de entender o resultado que mais fcil cobrir a
ltima casa com um passo grande (de duas casas) que com um passo
pequeno (de uma casa). Assim, a diferena entre a frequncia de caras e
coroas fica ainda mais evidente se avanarmos uma casa ao obter cara e
trs ao obter coroa, por exemplo. No caso extremo de avanarmos uma
casa com cara e oito casas com coroa, a chance de obter cara no ltimo
lanamento 1/256, menos que 4 em 1000!!!

Adaptado do artigo
A outra face da moeda honesta
Laura L. R. Rifo, RPM 64.
RPM OBMEP

164
Nmeros de regies:
um problema de contagem

Muitos problemas em Matemtica envolvem processos


adequados de contagem que, frequentemente,
conduzem a frmulas gerais extremamente teis; por
exemplo, para contar de quantas maneiras podemos
combinar n objetos em grupos de r desses objetos, usamos
a conhecida frmula que d o nmero de combinaes
de n objetos tomados r a r, a saber:
n n!
C ( n, r ) = = .
r r !(n r )!
Vamos analisar um problema de contagem do nmero de
regies no plano que pode ser resolvido de maneira direta,
simples e interessante. Trata-se do seguinte:
Considere 100 pontos distribudos sobre uma
circunferncia, de tal modo que o segmento ligando dois
quaisquer desses pontos no passe pelo ponto de
interseco de outros dois segmentos. Calcular o nmero
R de regies obtidas no crculo quando todos os 100
pontos estiverem ligados.
Inicialmente, tentamos resolver o problema com um
RPM OBMEP

nmero menor de pontos. Examinando os casos 2, 3, 4


e 5 pontos, temos:

165
R2 = 2 R3 = 4 R4 = 8 R5 = 16
figura 1
Observamos que:
2 pontos: 21 regies;
3 pontos: 22 regies;
4 pontos: 23 regies;
5 pontos: 24 regies.
Os resultados levam a acreditar que 6 pontos fornerceriam 25 = 32
regies, logo 100 pontos forneceriam 299 regies, e, por analogia
(incorreta, como veremos) n pontos determinariam 2n1 regies! Mas,
ao verificar diretamente o que acontece com 6 pontos, vemos que ficam
determinadas 31 regies, e no 32.
Logo, a generalizao pretendida no verdadeira.
1 A 2
B

6 3

5 4
figura 2

Como determinar uma frmula que fornea o nmero de regies


obtidas com 100 (ou um outro nmero qualquer) de pontos?
RPM OBMEP

Soluo 1
Os segmentos ligando dois a dois os 100 pontos sero chamados
diagonais; como para cada dois pontos temos uma diagonal, o nmero

166
100
delas C(100, 2) = , e o nmero de pontos de interseco das
2

100
diagonais C(100, 4) = , visto que cada 4 pontos determinam
4
duas diagonais, as quais tm um ponto em comum.
Vamos descrever um processo que nos permite obter o nmero de
regies pela eliminao sucessiva de diagonais.
Ao retirarmos uma das diagonais, o nmero de regies vai diminuir,
visto que duas regies que tm em comum um segmento da diagonal
retirada fundem-se em uma nica regio.
Por exemplo, na figura 2, a retirada da diagonal D12, que liga os
pontos 1 e 2, faz com que as regies A e B se transformem em uma
nica regio; a retirada da diagonal D35 transforma em quatro as oito
regies que tm partes dessa diagonal como arestas.
Podemos observar que, ao retirarmos uma diagonal, o nmero de
regies decresce conforme o nmero de pontos de interseco dessa
diagonal com aquelas que ainda no foram removidas, mais um. Com
efeito, esse o nmero de segmentos nos quais os referidos pontos de
interseco dividem a diagonal, e a remoo de cada um desses segmentos
transforma duas regies em uma. Assim, a remoo da diagonal D12,
que no tem ponto de interseco com as demais, produz um decrscimo
de apenas um no nmero total de regies; j a retirada da diagonal D35,
que tem 3 pontos de interseco com as demais diagonais, produz um
decrscimo de 4 regies.
Notemos que, no processo de retirada sucessiva das diagonais,
considera-se o nmero de pontos de interseco de cada diagonal com
aquelas que ainda no foram retiradas; no final do processo, ao serem
retiradas, sucessivamente, todas as diagonais, tal nmero igual ao
nmero total de pontos de interseco de todas as diagonais, ou seja
RPM OBMEP

100
C(100, 4) = ; ao mesmo tempo, o nmero de regies decresce at
4
reduzir-se a uma nica regio, quando todas as diagonais tiverem sido

167
eliminadas. Podemos ento concluir que o nmero de regies eliminadas
no processo de retirada sucessiva de todas as diagonais dado pelo
nmero total de pontos de interseco de todas as diagonais, ou seja
100
C(100, 4) = , acrescido de tantas parcelas iguais a 1 quantas so
4

100
as diagonais, ento, C(100, 2) = . Portanto, o nmero inicial de
2
regies, que igual ao nmero de regies eliminadas mais uma, a que
restou no final do processo, dado por
100 100 100
R100 = + + = 3.926.176.
0 2 4
Observe que, para n pontos, temos a mesma expresso, apenas
trocando o 100 por n. E, para 6 pontos, a frmula obtida fornece
6 6 6
R6 = + + = 31, como havamos verificado!
0 2 4
Soluo 2 (No volume anterior (2007) do nmero especial da RPM para a
OBMEP esta soluo, com menos detalhes, aparece na pgina 93.)

Em Geometria, uma das frmulas mais notveis a chamada frmula


de Euler, que estabelece uma relao entre o nmero de vrtices, arestas
e faces de um poliedro: V A + F = 2.
Mostraremos, em seguida, como a frmula que fornece o nmero de
regies determinadas por n pontos, distribudos em uma circunferncia,
pode ser obtida a partir da frmula de Euler, o que era de se esperar, pois
a demonstrao mais conhecida da frmula de Euler, devida a Cauchy,
comea removendo uma face do poliedro e deformando a parte restante
em uma regio plana que um polgono subdividido pelas arestas do
poliedro. Para poliedros planos, como o da figura 2, obtidos pela
interligao de n pontos na circunferncia, a frmula de Euler se reduz
RPM OBMEP

a V A + F = 1. (1)
Vamos calcular, separadamente, V, A e F em funo de n e
substitu-los na frmula (1) para obter Rn .

168
Clculo do nmero de vrtices
Para cada 4 vrtices na circunferncia existem dois, e apenas dois,
segmentos que se cruzam, e portanto determinam um vrtice chamado
n
interno, de modo que o nmero desses vrtices C (n, 4) = , ou
4

n
seja: V = n + . (2)
4
Clculo do nmero de arestas
Cada vrtice externo contribui com (n 1) arestas, cada vrtice
interno com 4 arestas e como cada aresta contm 2 vrtices:
n
2 A = n(n 1) + 4 e, portanto,
4

n(n 1) n n n
A= + 2 = + 2 . (3)
2 4 2 4

Clculo do nmero de regies


O nmero Rn obtido acrescentando-se a F o nmero n de
regies compreendidas entre o poliedro plano e a circunferncia, de
modo que
F = Rn n. (4)
Basta agora substituir (2), (3) e (4) na frmula (1) para se obter o
valor de Rn , na mesma expresso da soluo 1.
Observamos que o argumento usado na soluo 1 serve tambm para
demonstarr a frmula de Euler.

Adaptado do artigo
Nmero de regies: um problema de contagem
RPM OBMEP

Antnio C. Patrocnio, RPM 12.

169
Intuio e Probabilidade

Em muitas situaes do cotidiano estamos interessados


no clculo de determinadas probabilidades. Ocorre que,
em certos casos, especialmente aqueles envolvendo o
conceito de probabilidade condicional, esses clculos
levam a concluses que podem contrariar a intuio.
Apresentamos, neste texto, um exemplo dessa situao.
Num pas, 10% da populao portadora de um vrus.
Um teste para detectar a presena do vrus d 90% de
acertos quando aplicado a portadores e 80% de acertos
quando aplicado a no portadores.
Qual o percentual de pessoas realmente portadoras do
vrus, dentre aquelas que o teste classificou como
portadoras?
Vejamos uma soluo que no cita teoremas de
Probabilidade ou Estatstica.
Considere que o teste foi aplicado aos I habitantes do
pas. O nmero de testes que indicou a presena do vrus
foi:
0,9 0,1 I + 0,2 0,9 I = 0,09 I + 0,18 I = 0,27 I,
RPM OBMEP

sendo que a primeira parcela representa os 90% que so


realmente portadores e a segunda parcela representa os
20% que no so portadores. Logo, do total, 0,27 I, so
portadoras 0,09 I.

170
Assim, so realmente portadoras do vrus 0,09I / 0,27 I = 1/3
33,3% das pessoas que o teste classificou como portadoras.
Esse nmero no mnimo curioso e mostra que uma pessoa que fez
o teste e foi classificada como portadora tem grande possibilidade de
ser um falso-positivo (normalmente, quando uma pessoa faz um teste
desse tipo e o resultado positivo, os mdicos recomendam um novo
teste).
Por exemplo, o nmero de testes que indicaram a ausncia do vrus
foi 0,73 I e, dentre esses, 0,72 I no so portadores, o que d
0,72 I / 0,73I 98,6% de no portadores dentre os classificados
como no portadores.
Algumas variaes nos dados tambm originam resultados
interessantes. Por exemplo:
Se 0,5% da populao portadora e o teste acerta em 98% dos
casos, ento somente 20% das pessoas que o teste classificou como
portadoras so realmente portadoras.

Adaptado do artigo
Intuio e Probabilidade
Raul F. W. Agostino, RPM 27.

Resposta do Desafio
Como cada um dos pontos marcados na figura pertence a duas
circunferncias, a soma total dos nmeros colocados nas quatro
circunferncias (contando cada circunferncia em separado) igual a
2(1 + 2 + 3 + 4 + 5 + 6 + 7 + 8 + 9) = 90.
Para que a soma S dos nmeros pertencentes circunferncia externa
RPM OBMEP

seja exatamente igual soma (S) dos nmeros pertencentes a cada uma
das circunferncias internas, deveramos ter 90 = 4S, o que impossvel,
uma vez que 90 no divisvel por 4.
O problema no tem soluo.

171
Problemas I: problemas interessantes
com nmeros primos

P1. Escreva o nmero 91 como soma de dois nmeros primos.

P2. Eu e meu irmo caula temos idades entre 10 e 20 anos e hoje nossas
idades so expressas ambas por nmeros primos, fato que se repetir
pela prxima vez daqui h 18 anos. Determine minha idade sabendo
que a idade de nosso irmo mais velho, que, hoje, tambm um nmero
primo, uma unidade maior do que a soma das nossas idades.

P3. Uma equao do 2o grau, cujos coeficientes so todos nmeros


primos, pode apresentar duas razes iguais?

P4. Os nmeros a, b e logba podem ser todos primos?

A resposta aos dois problemas acima no, e eles no devem apresentar


maiores dificuldades ao leitor.

P5. Quantos pontos da reta y = x + 51 so tais que as suas duas coorde-


nadas so nmeros primos?

Observe-se que, trocando o nmero 51 por outro valor, o problema


pode tornar-se muito mais difcil. Para a reta y = x + 2 somos conduzidos
ao conceito de primos gmeos (diferem por 2 unidades). At hoje
um problema em aberto saber se existem ou no infinitos pares de
primos gmeos.

Se tomssemos a reta y + x = 40 obteramos seis solues: (3, 37),


(37, 3), (11, 29), (29, 11), (17, 23) e (23, 17), todas no primeiro quadrante
e que podem ser obtidas por inspeo direta.

Neste instante natural lembrar que a famosa conjectura de Goldbach


todo nmero natural par pode ser escrito como soma de dois nmeros
RPM OBMEP

primos ainda no foi provada e nem se encontrou um contra-exemplo.

P6. As medidas dos lados de um tringulo retngulo (numa mesma


unidade) podem ser nmeros primos?

172
P7. Para quantos pontos da circunferncia x2 + y2 = 361 as duas coorde-
nadas so nmeros primos?

P8. Para quantos pontos da circunferncia x2 + y2 = 461 as duas coorde-


nadas so nmeros inteiros?

Esse problema se assemelha ao anterior, embora seja mais difcil que


ele. Para resolv-lo sugerimos a leitura de um artigo de Gilberto Garbi,
Outro belo teorema de Fermat, publicado na RPM 38.

P9. Determine as medidas, em graus, dos ngulos internos de um


tringulo acutngulo, sabendo que elas so expressas por nmeros
primos.

A mesma pergunta sem a hiptese de ser acutngulo exige um pouco


mais de trabalho.

P10. Quantos divisores positivos possui o nmero 2 420?

Esse exerccio uma aplicao clssica do Teorema Fundamental da


Aritmtica e do Princpio Fundamental da Contagem.

P11. Verifique que todos os n 1 nmeros da sequncia n! + 2, n! + 3,


..., n! + n so nmeros compostos so nmeros compostos, isto , nenhum
deles um nmero primo.

P12. Quantos so os nmeros naturais, de 1 a 100, que podem ser escritos


como um produto de dois nmeros naturais distintos entre si e diferentes
de 1?

P13. Apresente algum nmero natural n para o qual o valor numrico


p(n) do polinmio p(n) = x2 + x + 41 no seja um nmero primo.

P14. Quantos polgonos regulares, com nmero par de lados, podem ter
todas as diagonais expressas (numa mesma unidade) por nmeros
primos?
RPM OBMEP

P15. H dois anos, ano em que finalmente conclu meu Doutorado em


Matemtica, nasceu meu segundo filho e ocorreu uma notvel
coincidncia: eu e meus dois filhos passamos a fazer aniversrio no
mesmo dia do ano. A partir da outras coincidncias aconteceram. No

173
ano passado nossas trs idades foram representadas por quadrados
perfeitos e hoje, dia em que estamos comemorando mais um aniversrio,
percebo que nossas idades so representadas por trs nmeros primos.
Supondo que vivamos cem anos cada um, pergunto: qual minha idade
hoje? Nos prximos anos, quantas vezes todas as nossas idades voltaro
a ser representadas por nmeros primos?

Adaptado do artigo
Os primos esquecidos
Chico Nery e Cludio Possani, RPM.
RPM OBMEP

174
Problemas II: problemas do PISA*

*Programme for International Student Assessment


Programa Internacional de Avaliao de Alunos 2000/2003
http://www.inep.gov.br/internacional/pisa/novo (julho de 2009)

Questes
Estantes
Para construir uma estante completa, um marceneiro precisa do seguinte
material:
4 pranchas grandes de madeira,
6 pranchas pequenas de madeira,
12 braadeiras pequenas,
2 braadeiras grandes e
14 parafusos.
O marceneiro possui em estoque 26 pranchas grandes de madeira, 33
pranchas pequenas de madeira, 200 braadeiras pequenas, 20 braadeiras
grandes e 510 parafusos. Quantas estantes completas o marceneiro poder
fazer?

Bombons coloridos
A me de Roberto permite que ele pegue
um bombom de um saco. Ele no consegue
ver os bombons. O grfico ao lado mostra
o nmero de bombons de cada cor contidos
no saco.
Qual a probabilidade de Roberto pegar
um bombom vermelho?
A) 10% B) 20% C) 25% D) 50%
RPM OBMEP

Carpinteiro
Um carpinteiro tem 32 metros de tbua para cercar um canteiro em uma
horta. Ele est pensando em utilizar um dos seguintes modelos para o
canteiro.

175
Na tabela abaixo, faa um crculo em Sim ou No para cada modelo,
indicando se ele pode ou no ser feito com 32 metros de tbuas.

Modelo de canteiro Usando esse modelo, o canteiro pode ser construdo


com 32 metros de tbua?
modelo A Sim/No
modelo B Sim/No
modelo C Sim/No
modelo D Sim/No

Dados
O desenho da direita representa dois dados.
Os dados so cubos com faces numeradas de acordo com
a seguinte regra: em um mesmo dado, o nmero total de
pontos de duas faces opostas sempre sete.
Questo 1
direita, vemos trs dados empilhados um sobre
o outro. O dado 1 possui quatro pontos na face
superior.
Quantos pontos h, no total, nas cinco faces
horizontais que no podemos ver (face inferior
do dado 1, faces superiores e inferiores dos dados
2 e 3)?
RPM OBMEP

Questo 2
Voc pode fazer um dado, cortando, dobrando e colando uma cartolina.
Isso pode ser feito de diversas maneiras. Na figura a seguir, h quatro

176
opes que podem ser utilizadas para fazer dados.
Quais dos formatos podem ser dobrados para formar um dado que obedea
regra na qual a soma das faces opostas 7? Para cada formato, faa um
crculo em Sim ou No na tabela abaixo.

Formato Obedece regra na qual a soma dos lados opostos 7?

I Sim/No
II Sim/No
III Sim/No
IV Sim/No

Sequncia em escada
Roberto constri uma sequncia com o formato de uma escada, utilizando
quadrados. Ele segue as seguintes etapas:

Como voc pode ver, ele utiliza um quadrado na Etapa 1, trs quadrados
na Etapa 2 e seis na Etapa 3.
Quantos quadrados ele vai utilizar na Etapa 4?

Terremoto
Foi divulgado um documentrio sobre terremotos e a frequncia com
que eles ocorrem. Essa reportagem inclui uma discusso sobre a
previsibilidade dos terremotos.
Um gelogo declarou: Nos prximos vinte anos, a probabilidade de
RPM OBMEP

que ocorra um terremoto em Zedpolis de dois sobre trs.


Qual das opes a seguir exprime melhor o significado da declarao do
gelogo?

177
A) 2/3 20 = 13,3, portanto no perodo de 13 a 14 anos, a partir de hoje,
haver um terremoto em Zedpolis.
B) 2/3 maior que 1/2, portanto podemos ter certeza de que haver um
terremoto em Zedpolis nos prximos 20 anos.
C) A probabilidade de haver um terremoto em Zedpolis nos prximos
20 anos maior do que a probabilidade de no haver um terremoto.
D) No se pode afirmar o que acontecer porque ningum pode ter
certeza de quando ocorrer um terremoto.

Feira de Vero
Uma barraca de uma feira de
vero prope um jogo no qual se
utiliza primeiro uma roleta. Em
seguida, se a roleta parar em um
nmero par, o jogador poder
pegar uma bolinha de gude de
dentro de um saco. A roleta e as bolinhas de gude contidas no saco esto
representadas na ilustrao.
Os prmios so distribudos s pessoas que pegam uma bolinha de gude
preta. Qual a probabilidade de Sueli ganhar um prmio?
A) Impossvel. B) Pouco provvel.
C) Cerca de 50% de probabilidade. D) Muito provvel.
E) Certeza.

Bate-papo pela Internet


Mark (de Sydney, na Austrlia) e Hans (de Berlim, na Alemanha)
comunicam-se com frequncia por meio de uma sala de bate-papo da
Internet. Para isso, eles precisam conectar-se Internet ao mesmo tempo.
Para determinar um horrio apropriado para bater papo, Mark consultou
uma tabela de fusos horrios do mundo e encontrou o seguinte:
RPM OBMEP

178
Questo 1
Que horas so em Berlim quando so 19 horas em Sydney?
Questo 2
Mark e Hans no podem bater papo das 9h s 16h30 de seus horrios
locais respectivos, porque eles devem ir para a escola. Alm disso, no
podero bater papo entre 23h e 7h porque estaro dormindo.
Qual seria um bom horrio para Mark Local Horrio
e Hans baterem papo? Escreva os Sydney
horrios locais na tabela ao lado: Berlim

Assaltos
Um reprter de TV apresentou o grfico
ao lado e disse:
O grfico mostra que, de 1998 para
1999, houve um grande aumento no
nmero de assaltos.
Voc considera que a afirmao do
reprter uma interpretao razovel do
grfico? D uma explicao que justifique
a sua resposta.

Colnia de frias
Os Servios Comunitrios de Zedpolis esto organizando um acampa-
mento para um perodo de cinco dias. Inscreveram-se 46 crianas (26
meninas e 20 meninos) e 8 adultos (4 homens e 4 mulheres) apresentaram-
se como voluntrios para acompanh-las e organizar o acampamento.

Tabela 1: Adultos Tabela 2: Dormitrios


Sra. Marlia Nome Nmero de camas
Sra. Carolina Vermelho 12
Sra. Graa Azul 8
Srta. Ktia Verde 8
RPM OBMEP

Sr. Slvio Roxo 8


Sr. Nelson Laranja 8
Sr. William Amarelo 6
Sr. Pedro Branco 6

179
Regra dos dormitrios
1. Os meninos e as meninas devem dormir em dormitrios separados.
2. Em cada dormitrio deve dormir, pelo menos, um adulto.
3. O(s) adulto(s) que ficar(em) nos dormitrios deve(m) ser do mesmo
sexo que as crianas.
Preencha a tabela abaixo, distribuindo as 46 crianas e os 8 adultos
nos dormitrios, de maneira que todas as regras sejam obedecidas.

Nome Nmero de meninos Nmero de meninas Nome(s) do(s) adulto(s)


Vermelho
Azul
Verde
Roxo
Laranja
Amarelo
Branco

O leitor deve ter observado que, em linhas gerais, as questes so


diferentes das que normalmente so apresentadas aos nossos alunos em
sala de aula. Elas exigem pouco contedo, pouca memria, mas, nas
palavras dos idealizadores do PISA, examinam a capacidade dos alunos
de analisar, raciocinar e refletir ativamente sobre seus conhecimentos e
experincias, enfocando competncias que sero relevantes para suas
vidas futuras.
No site indicado, alm das questes de 2003, esto algumas questes
de 2000, e tambm uma descrio dos mecanismos utilizados para a
seleo dos estudantes que participaro do teste, normas de avaliao,
algumas explicaes sobre o mau desempenho dos brasileiros e outros
temas correlatos.
RPM OBMEP

180
PROBLEMAS III

A numerao entre parnteses a original dos exemplares da RPM

1. (184) Os nmeros reais a, b e c so tais que a + b + c = 3,


a2 + b2 + c2 = 13 e a3 + b3 + c3 = 27. Determine a4 + b4 + c4.

2. (205) Trs aranhas caminham pelos lados de um tringulo ABC e


movimentam-se de modo que em qualquer instante formam um
tringulo e o baricentro de todos os tringulos formados sempre o
mesmo ponto (fixo) P. Sabendo-se que uma das aranhas percorre
todo o tringulo ABC, mostrar que P tambm o baricentro do
tringulo ABC.

3. (207) Um rapaz esqueceu o ltimo algarismo do telefone da namorada


e resolveu tentar falar com ela escolhendo ao acaso o ltimo dgito.
Se ele est num telefone pblico e s tem duas fichas, qual a
probabilidade de que ele consiga conversar com a namorada?

4. (219) Dados x e y nmeros inteiros positivos, mostre que, se


x2 + y2 + xy divisvel por 10, ento divisvel por 100.

5. (220) Considere duas retas paralelas que


distam a entre si e um quadrado ABCD, de
lado a, situado no plano das paralelas numa
posio tal que os vrtices A e C estejam em
lados opostos do plano dividido pela faixa
das paralelas. Calcule a soma dos permetros
dos tringulos sombreados.

6. (236) Achar todos os nmeros m e n naturais que resolvam


n2n1 + 1 = m2.
RPM OBMEP

7. (265) Seja ABC um tringulo tal que = 60o. Seja H o seu


ortocentro e J AC tal que AJ = 2JC e JC = JH. Mostre que o
tringulo ABC equiltero.
181
8. (266) Seja p(x) o polinmio de grau 2007, com coeficientes reais,
2 2 2007
p ( x) = ( x sen + cos ) .
2007 2007
Determine o resto da diviso de p(x) por x2 + 1.
9. (267) Numa folha quadrada de papel desenhe ou dobre um ngulo ,
marque a metade da folha e a metade da metade. Dobre a folha de
modo que A caia em um ponto A pertencente a r e B em um
ponto B pertencente a s (ver figura). Marque os pontos A, B e
C, o correspondente de C na dobra. Prove que AB, AA e AC
trisseccionam o ngulo .

10. (268) Seja f: N N uma funo tal que


f(f(x)) = x para todo x N.
a) Mostre que f bijetora.
b) Exiba uma funo f com a propriedade acima e tal que
f(x) x para todo x N.

11. (270) Dispe-se de 2007 moedas viciadas M1, M2, ..., M2007.
Sabe-se que, em um lanamento, a probabilidade de se obter cara
1
na moeda Mi, i = 1, 2, ... 2007, . Se as 2007 moedas
(2i + 1)
so lanadas simultaneamente, qual a probabilidade de que o
nmero de caras obtidas seja mpar?
A

RPM OBMEP

12. (271) Na figura, AD = 2BD.


Determine .
D
o
45 15
o

B C
182
13. (272) Encontre as razes reais da equao

x + 2 x 1 + x 2 x 1 = 2.

14. (274) Prove que:


a) num tringulo retngulo a medida da mediana relativa
hipotenusa igual metade da medida da hipotenusa.
b) todo tringulo pode ser decomposto em n tringulos issceles,
para todo n > 4.

15. (276) Determine todas as funes f: R R satisfazendo


2f(x) + f(1 x) = x2008 para todo x R.

16. (277) Qual a maior potncia de 2 que divide 32008 1?

17. (280) (Jogo de Kontsevich) Consideremos


o tabuleiro infinito, ilustrado na figura, que
ocupa o primeiro quadrante. Inicialmente,
h 6 peas nas casas sombreadas no canto
inferior esquerdo. Um movimento consiste
em escolher uma pea sem vizinhos nas
casas imediatamente acima e direita e
substituir essa pea por duas, colocando-
as nas casas vizinhas vagas:

movimento

O objetivo do jogo , a partir da configurao inicial, realizar uma


sequncia de movimentos de modo a deixar as 6 casas sombreadas
sem peas.
a) Atribua o peso 2ij para a casa na posio (i, j), como
RPM OBMEP

mostra a figura (a casa inicial, inferior esquerda, situa-se na


posio (0,0)). Mostre que a soma dos pesos das casas ocupadas
por peas, depois de se realizarem movimentos, constante,
isto , no se altera aps qualquer sequncia de movimentos.
183
...
1 8 1 16 1 32 1 64
...
14 1 8 1 16 1 32
...
12 14 1 8 1 16
...
1 12 14 18
...
b) Determine a soma de todos os pesos do tabuleiro.
c) Mostre que impossvel realizar o objetivo do jogo: no existe
nenhuma sequncia de movimentos que deixa as 6 casas
iniciais, sombreadas, livres de peas!

18. (281) Simplifique:


n3 1 23 1 33 1 43 1 20083 1
3
+1
= . .
23 + 1 33 + 1 43 + 1
. .
20083 + 1
.
2 n 2008 n

19. (282) Prove que, se x e y so inteiros tais que


N = (x + 6y)(2x + 5y)(3x + 4y)
mltiplo de 7, ento N mltiplo de 343.

20. (284) Dado um ngulo qualquer AOB com AO = BO, dividimos


AB em 3 partes iguais: AC = CD = BD. Pergunta-se: existe algum
AOB para o qual os ngulos AOC , COD e BOD so
congruentes?
RPM OBMEP

184
Solues dos Problemas I

P1.
Os alunos no devero ter dificuldade em perceber que, como a soma de
dois mpares par e como 2 o nico primo par, os nmeros so 2 e 89.
Alis, esse pode ser um bom momento para recordar com os alunos os
testes de primalidade, para verificar que 89, efetivamente, primo.
P2.
As duplas de primos entre 10 e 20 so:
11 e 13, 11 e 17, 11 e 19, 13 e 17, 13 e 19 e 17 e 19.
Como a soma dos nmeros adicionada de 1 deve resultar um primo,
descarto as duplas 11 e 13 e 13 e 19. Como daqui a 18 anos as idades
voltam a ser representadas por nmeros primos, descarto as duplas que
incluem o 17. Resta apenas uma possibilidade: minha idade 19 anos e
a do meu irmo 11 anos.
P3.
Para que a equao ax2 + bx + c = 0 (com a, b e c primos) admita duas
razes iguais, devemos ter b2 4ac = 0 ou b2 = 4ac, o que implica b2
par. Logo, b tambm par e, como primo, b = 2. De b2 = 4ac, com
b = 2, temos ac = 1, o que absurdo para a e c primos.
Portanto, nas condies impostas, a equao no pode admitir duas razes
iguais.
P4.
Seja x = logba, portanto bx = a. Se b e x so nmeros primos, ento
bx no primo; logo, a no primo.
P5.
Se x = 2 , temos y = x + 51 = 53, que primo. Se x for qualquer outro
RPM OBMEP

primo, ser um nmero mpar, implicando y par maior que 2, logo,


no primo. Assim, existe um nico par, (2, 53), da reta de equao
y = x + 51 que tem ambas as coordenadas dadas por nmeros mpares.

185
P6.
Soluo: A resposta no. Do teorema de Pitgoras temos a igualdade
a2 = b2 + c2. Sendo a, b e c primos, no podem ser todos mpares e,
como a > b e a > c, devemos ter b = 2 ou c = 2. Digamos c = 2.
Teremos ento:
a2 b2 = 4
(a + b)(a b) = 4
e analisando os possveis valores de a + b e a b, que so 1, 2 ou 4,
conclumos que a situao impossvel.
P7.
Se x e y satisfazem a equao x2 + y2 = 361, sendo 361 mpar,
devemos ter x par e y mpar ou x mpar e y par. Se x par e primo,
ento, x = 2; logo, y2 = 357 e y no , ento, um nmero inteiro. Do
mesmo modo verificamos ser impossvel ter y par e x mpar; logo,
nenhum ponto da circunferncia de equao x2 + y2 = 361 tem ambas
as coordenadas dadas por nmeros primos.
P8.
Observamos, inicialmente, que 461 = 100 + 361 = 102 + 192, logo os
seguintes oito pontos, de coordenadas inteiras, pertencem circunferncia
de equao x2 + y2 = 461: (10, 19), (10, 19), (10, 19), (10, 19),
(19, 10), (19, 10), (19, 10) e (19, 10).
Alm disso, sendo 461 um nmero primo que dividido por 4 deixa
resto 1, o resultado de Fermat, todo nmero primo que dividido por
4 deixa resto 1 pode ser escrito como soma dos quadrados de dois
nmeros inteiros, de modo nico, a menos da ordem, prova que esses
oito so os nicos pontos de coordenadas inteiras pertencentes
circunferncia.
P9.
Se a + b + c = 180, com a, b e c primos, no possvel ter a, b e
c mpares; logo, pelo menos um deles, digamos o a, deve ser igual a 2,
RPM OBMEP

o que implica b + c = 178. Podemos ter b = c = 89, que primo e, por


verificao direta, mostra-se que no h outra possibilidade, j que o
tringulo, sendo acutngulo, implica b < 90 e c < 90.

186
Sem a hiptese de o tringulo ser acutngulo, obtemos, por tentativa, as
possibilidades: 5 e 173, 11 e 167, 29 e 149, 47 e 131 e 71 e 107.
P10.
Decompondo 2420 em fatores primos encontramos: 2420 = 22 5 112.
Os divisores positivos de 2420 so todos do tipo 2 5 11 com
valendo 0, 1 ou 2, valendo 0 ou 1 e valendo 0, 1 ou 2.
Logo, pelo Princpio Fundamental da Contagem, a quantidade de
divisores positivos de 2420 : 3 2 3 = 18.
P11.
Observemos que: n! + 2 divisvel por 2, n! + 3 divisvel por 3, ...,
n! + n divisvel por n, e assim sendo, nenhum deles primo.
Para valores grandes de n essas sequncias de nmeros naturais
consecutivos so chamadas desertos de nmeros primos.
P12.
De 1 a 100 temos 100 nmeros. Para obtermos a resposta nossa
pergunta, subtramos de 100 o nmero de primos entre 1 e 100, que
25; o nmero de quadrados de nmeros primos, que 4, e o nmero 1.
A resposta 70.
P13.
Para x = 40,
x2 + x + 41 = 402 + 40 + 41 = 40(40 + 1) + 41 = 40.41 + 41 = 41(40 + 1)
= 412 que no primo. Tambm para x = 41, x2 + x + 41 = 412 + 41 + 41
= 41(41 + 1 + 1) = 41.43, que no primo.
Prova-se que para qualquer valor inteiro de x, 40 < x < 39, tem-se
x2 + x + 41 igual a um nmero primo (ver RPM 09, p. 33).
P14.
1. Como podemos construir um quadrado com qualquer tamanho,
podemos constru-lo com suas diagonais medindo: 2 ou 3 ou 5, ...,
2
ou qualquer outro valor primo. O lado do quadrado medindo p
RPM OBMEP

2
faz a diagonal medir p, para qualquer p primo.

187
2. J no caso do hexgono regular, se traarmos E D
duas diagonais, uma passando pelo centro e
a outra no, vejamos o que acontece.
F C
Sendo ABCDEF um hexgono regular, no
tringulo ABD temos o
60
BD 3 A B
= sen60D = , implicando que as me-
AD 2
didas das diagonais BD e AD no podem ser simultaneamente
expressas por nmeros inteiros, logo no podem ser ambas nmeros
primos.
3. Para qualquer outro polgono regular com nmero
par de lados (octgono, decgono, ... etc), se P
considerarmos dois vrtices P e Q diametralmente
opostos e um vrtice M no consecutivo de P nem
de Q, eles determinaro o tringulo PQM, retngulo O M
em M, cujos lados so trs das diagonais desse
polgono e, como j foi provado no P6, as medidas
desses trs lados no podem ser simultaneamente Q
expressas por nmeros primos.
Concluso, apenas o quadrado pode ter todas as suas diagonais com
medidas expressas por nmeros primos.
P15.
No ano passado meu filho caula certamente tinha 1 ano de idade. Meu
outro filho tinha 4 ou 16 anos e eu, o pai, 36 anos.
Portanto, hoje, minha idade 37 anos.
Quando a minha idade mpar, a do meu caula par e vice-versa;
portanto, nunca mais nossas idades voltaro a ser todas simultaneamente
representadas por nmeros primos.
RPM OBMEP

188
Solues dos Problemas II

Estantes
5
Bombons coloridos
B) 20%
Carpinteiro
modelo A sim
modelo B no
modelo C sim
modelo D sim
Dados
Questo 1
17 = 21 4
Questo 2
No, Sim, Sim, No (nesta ordem).
Sequncia em escada
10
Terremoto
C) A probabilidade de haver um terremoto em Zedpolis nos prximos
20 anos maior do que a probabilidade de no haver um terremoto.
Feira de Vero
B) Pouco provvel.
Bate-papo pela Internet
Questo 1
10 da manh ou 10h.
Questo 2
Qualquer horrio ou intervalo de tempo que satisfaa a diferena de 9
RPM OBMEP

horas e que esteja compreendido entre um dos seguintes intervalos:


Sydney: 16:30 h 18:00 h; Berlim: 7:30 h 9:00 h.
ou
Sydney: 7:00 h 8:00 h; Berlim: 22:00 h 23:00 h.

189
Assaltos
No, um crescimento de aproximadamente 10 no muito grande
comparado a um total de aproximadamente 500.
Colnia de frias: uma soluo
Nome meninos meninas adultos
Vermelho 10 HH
Azul 7 M
Verde 7 M
Roxo 6 H
Laranja 7 M
Amarelo 4 H
Branco 5 M
Total 20 26
H = homem
M = mulher
RPM OBMEP

190
Solues dos Problemas III

1.
Vamos observar inicialmente que (a + b + c)3 = a3 + b3 + c3.
Segue-se, ento, que: a2(b + c) + b2(a + c) + c2(a + b) + 2abc = 0 ou
equivalentemente, (a + b)(a + c)(b + c) = 0.
Para que isso ocorra, pelo menos um dos fatores deve ser nulo. Supondo
a + b = 0, a primeira equao nos d c = 3. Substituindo na segunda,
obtemos a = 2 e b = 2 . Segue-se que a4 + b4 + c4 = 89.

2.
Suponhamos que seja a aranha A1 que percorre todo o ABC e
consideremos o instante em que ela est em A. Seja M o ponto mdio
do lado A2A3 do A1A2A3 formado pelas trs aranhas nesse instante.

Traamos r e s retas paralelas a BC e que dividem a altura AH em


trs partes iguais. Prolongamos AM at encontrar BC em M. Como
r e s dividem AM em trs partes iguais (Tales), ento o baricentro
do A1A2A3 que o ponto que dista AM/3 de M, est na regio
pintada.
RPM OBMEP

191
Repetindo o argumento quando a aranha A1 est em B e quando est
em C, conclumos que o baricentro P comum dos tringulos formados
pelas trs aranhas est em trs regies que se cortam exatamente no
baricentro do ABC, o que demonstra o resultado pedido. Para justificar
esta ltima afirmao, observamos que a reta r e as outras duas
construdas analogamente, quando a aranha A1 est em B ou C, en-
contram-se no baricentro do ABC.

3.
a) A probabilidade de que o rapaz acerte na primeira tentativa igual a
1/10, uma vez que ele escolhe ao acaso um dos dez dgitos possveis.
b) Para que ocorra a segunda tentativa necessrio que ele tenha errado
na primeira, e a probabilidade de isso acontecer igual a 9/10. Dado
que errou na primeira tentativa, a probabilidade (condicional) de que
ele acerte na segunda igual a 1/9, uma vez que, agora, o nmero de
dgitos possveis igual a 9. Logo, a probabilidade de que ele acerte
na segunda tentativa (9/10)(1/9) = 1/10.
Segue que a probabilidade de que ele consiga conversar com a namorada
igual a (1/10) + (1/10) = 1/5.

4.
Se 10 divide x2 + y2 + xy, ento 2 divide x2 + y2 + xy; logo,
x2 + y2 + xy par, implicando x e y pares, o que, por sua vez, implica
x2 + y2 + xy mltiplo de 4. Se 10 divide x2 + y2 + xy, ento 5 divide
x2 + y2 + xy.
Se mostrarmos que isso implica x e y mltiplos de 5, teremos que 25
dividir x2 + y2 + xy, que mltiplo de 4, logo 100 dividir
x2 + y2 + xy.
Prova de que x e y so mltiplos de 5
Escrevendo x = 5a + b, y = 5c + d, com a, b, c e d inteiros no
negativos e 0 < b < 4, 0 < c < 4, obtemos
x2 + y2 + xy = (5a + b)2 + (5c + d)2 + (5a + b)(5c + d) =
RPM OBMEP

5(5a2 + 2ab + 5c2 + 2cd + 5ac + ad + bc) + b2 + d2 + db.


Como 5 divide x2 + y2 + xy, temos que 5 divide b2 + d2 + bd.

192
Se b = d 0 temos b2 + d2 + bd = 3b2, que no mltiplo de 5, j que
b = 1, 2, 3 ou 4.
Se b d, fazendo todas as possveis substituies para b e d em
b2 + d2 + bd, obtemos:
12 + 22 + 1 2 = 7, 12 + 32 + 1 3 = 13, 12 + 42 + 1 4 = 21,
22 + 32 + 2 3 = 19, 22 + 42 + 2 4 = 28, 32 + 42 + 3 4 = 37.
Como nenhum dos resultados divisvel por 5, conclumos que
b = d = 0 e, portanto, x e y so mltiplos de 5.

5.
Consideremos o quadrado MNPQ, com lados paralelos s retas paralelas
dadas e contendo os vrtices ABCD, como na figura. Se o ngulo
indicado, ento DQ = asen e MD = acos.
Logo, a medida dos lados do quadrado MNPQ igual a asen + acos.
Indicando por z e z as hipotenusas dos tringulos sombreados e por h
e h as alturas desses tringulos, temos:
Q C P
x h y
z
a
D
 A
a
B
z
h 
y x

M A N

h + h + a = asen + acos ou h + h = a(sen + cos 1). Por outro


h + h'
lado, zsencos = h e zsencos = h ou z + z' = . Sendo
sen cos
x, y, x e y os catetos indicados na figura, temos
RPM OBMEP

h h' h + h'
x= e x' = , o que implica x + x' = ;
sen sen sen

193
h h' h + h'
y= e y' = , o que implica y + y' = .
cos cos cos
A soma, s, dos permetros dos tringulos sombreados ser:
s = x + x + y + y + z + z

h + h' h + h' h + h' (h + h' )(sen + cos + 1)


s= + + =
sen cos sen cos sen cos
a (sen + cos 1)(sen + cos + 1) a[(sen + cos ) 2 1]
s= =
sen cos sen cos
a 2sen cos
s= s = 2a.
sen cos

6.
Dois naturais m e n resolvem n2n1 + 1 = m2 se e s se m = 2k + 1 e
n2n3 = k(k + 1) (*) para algum natural k.
fcil verificar diretamente que, para 0 < n < 6, as nicas solues do
problema so (n, m) = (0, 1) e (n, m) = (5, 9).
Mostremos que, para n > 7, no existe soluo.
De fato, observe em (*) que 2n3 divide k ou k + 1.
Se 2n3 divide k + 1, ento k divide n e
n k +1
= n > k e k + 1 > 2n3 n + 1 > 2n3.
k 2 n 3
Se 2n3 divide k, ento k + 1 divide n e
n k
= n 3 n > k +1 e k > 2n3 n > 2n3 + 1.
k +1 2
Porm, fcil ver, por induo, que n + 1 < 2n3 para n > 7.

7.
Seja x = JC, de modo que AC = 3x. No tringulo retngulo ACC,
RPM OBMEP

temos que a medida do ngulo CAC igual a 60o e, portanto, a


medida do ngulo ACC igual a 30o. Como JH = JC, temos o ngulo
JHC igual a 30o; logo, o ngulo HJB mede 60o.

194
x
Portanto, B'J = HJ cos 60D = ,
2
x 3 x AC
implicando CB' = x + = = , isto
2 2 2
, B ponto mdio de AC.

Logo, BB' simultaneamente altura e


mediana do ABC, que , portanto,
l=l
issceles com AB = BC. Portanto, C A = 60D , o que mostra que ABC
de fato equiltero.
8.
Como x2 + 1 tem grau 2, temos que o resto da diviso de p(x) por
x2 + 1 tem a forma ax + b com a, b . Temos, portanto,
p(x) = (x2 + 1)q(x) + ax + b.
Fazendo x = i e x = i, obtemos

p (i ) p (i )
p (i ) = ai + b a = 2i
.
p ( i ) = ai + b b = p (i ) + p (i )
2
Pela frmula de Moivre, temos
2 2 2007
p (i ) = (cos + i sen ) = cos 2 + i sen 2 = 1 e
2007 2007

2 2 2007 2 2 2007
p (i ) = (cos i sen ) = [cos( ) + i sen( )] = 1.
2007 2007 2007 2007

Substituindo nas expresses de a e b, temos a = 0 e b = 1, isto , o


resto da diviso 1.
9.
RPM OBMEP

Na figura a seguir, seja M o ponto de interseco de XY e CA' .


Como MC perpendicular a AB , temos que MC' perpendicular
a A'B' . Vamos mostrar primeiramente que A, M e C so colineares,

195
e que, portanto, AC' perpendicular a A'B' . Para isso, basta mostrar
m ) = m( A' MC
que m( AMC m ' ) . Como X l' ' reto, temos
AC

m( A'm
MC' ) = 90D m( M l' ' ) = m( M l
AC A' X ) .
Agora, como MA = MA e XA = XA, temos que AMA e AXA so
issceles e assim m( M l
A' X ) = m( M l
AX ) .

Mas m( M l m ) , pois A'C paralelo a AZ , o que mostra


AX ) = m( AMC
m ) = m( A' MC
que m( AMC m ') .

Assim, os tringulos ACB e ACA so


congruentes pelo caso de congruncia LAL, j
que BC = BC = AC = AC e o ngulo C reto.
Logo, = , pois so ngulos correspondentes.
Os tringulos ACA e AZA so congruentes
pelo caso especial cateto-hipotenusa, pois AC
= AZ. Logo, = e, portanto, = = , ou
seja, o ngulo foi dividido em 3 partes iguais.
10.
Sendo f: N N tal que f(f(x)) = x, x N, temos:
a) f injetora, pois, se f(x1) = f(x2), ento f(f(x1)) = f(f(x2)), isto ,
x1 = x2. Temos tambm f sobrejetora, pois, dado y N, seja
x = f(y) N e ento f(x) = f(f(y)) = y.
n + 1, se n par
b) A funo f: N N dada por f (n) =
n 1, se n mpar
obedece condio f(f(n)) = n, nN, no caso em que
N = {0, 1, 2, 3, ..., n, ...}. E no caso que N = {1, 2, 3, ..., n, ...}, basta
tomar f(n) = n 1, se n par e f(n) = n + 1, se n mpar.
RPM OBMEP

Observao
Existem diversas outras solues. Por exemplo, sejam
A = { a1, a2, a3, ...} e B = {b1, b2, b3, ...} conjuntos de uma partio qual-

196
quer de N (isto , A B = N e A B = ). Basta ento definir
f(ai) = bi e f(bi) = ai, i natural. A soluo apresentada corresponde
partio de N nos conjuntos A e B dos nmeros pares e mpares,
respectivamente.
11.
Sejam Mi, i = 1, 2, ..., n, cada uma das moedas,
1
Pi = , i = 1, 2, ..., n , a probabilidade de se obter cara jogando a
2i + 1
moeda Mi e PTi a probabilidade de haver um nmero mpar de caras
jogando-se as moedas M1, M2, ..., Mi.
Queremos achar PT2007.
1
Para i = 1, temos que PT1 = P1 = , pois a probabilidade de haver 1
3
cara no lanamento de M1.
Para i = 2, temos que PT2 a probabilidade de M1 ser cara e M2 ser
coroa ou M1 ser coroa e M2 ser cara: PT2 = P1(1 P2) + (1 P1)P2.
1 1 1 1 4 2 6 2
PT2 = .(1 ) + (1 ). = + = = .
3 5 3 5 15 15 15 5
1 2 n
Sendo PT1 = e PT2 = , parece que PTn = . Vamos tentar
3 5 2 n +1
provar essa igualdade pelo princpio da induo finita.
n (n 1)
Tese: PTn = . Hiptese: PTn1 = .
2n + 1 2(n 1) + 1

(n 1) n 1 n 1
Temos PTn1 = = = . Porm, PT n a
2(n 1) + 1 2n 2 + 1 2n 1
probabilidade de haver um nmero mpar de caras em M1, M2, ..., Mn1
e Mn ser coroa ou haver um nmero par de caras em M1, M2, ..., Mn1
RPM OBMEP

e Mn ser cara. Portanto, PTn = PTn1(1 Pn) + (1 PTn1)Pn ou


n 1 1 n 1 1 n
PTn = ( )(1 ) + [1 ( )]( )= .
2n 1 2n + 1 2n 1 2n + 1 2n + 1

197
2007 2007
Portanto, PT2007 = = .
2(2007) + 1 4015

12.
Traamos a perpendicular reta CD, por A, que corta o segmento CD
no ponto E, pois ADCl agudo (mede 60o).

O BDE issceles, pois


DE = 2acos 60o = a = BD. Logo,
l
o BEC issceles, pois E BC
mede 15o e, portanto, BE = EC.
Alm disso, BE = AE, pois BEA
l e Bl
issceles, j que ABE AE
medem 30o. Logo, AE = EC e o
AEC issceles com E lAC me-
dindo 45 . Assim, = 75o.
o

Observaes
1. No site www.gogeometry.com possvel encontrar outros problemas desse
tipo.
2. Este problema tambm pode ser resolvido utilizando-se a lei dos senos ou
dos cossenos. Que tal tentar?

13.
Para que a equao x + 2 x 1 + x 2 x 1 = 2 tenha soluo
1
real necessrio que 2x 1 > 0, isto , x . Nesse caso, o primeiro
2
radicando sempre positivo e para ver que x 2 x 1 basta observar

que: ( x 1) 2 0 x 2 2 x 1 x 2 2 x 1 x = x 2 x 1.
Para x > 1/2, tem-se: x soluo da equao se e somente se

( x + 2 x 1 + x 2 x 1 ) 2 = 2 ( x 1) 2 = 1 x x 1 = 1 x
RPM OBMEP

1 x = 1 x 1 x 0 x 1.
Logo, o conjunto soluo da equao o intervalo S = [1/2, 1].

198
14.
a) No ABC, retngulo em B, M o ponto A
mdio de AC e MH perpendicular a BC.
Como BH = HC, o BMH e o CMH M
so congruentes e, portanto, BM = MC =
AM. Assim, todo tringulo retngulo pode
ser decomposto em dois tringulos issceles B H C
ABM e BMC.
b) Dado um PQR, qualquer, traamos uma altura interna que o
decompe em dois tringulos retngulos. Usando a), vemos que o
PQR pode ser decomposto em n = 4 tringulos issceles. Repetindo
o procedimento em um, dois, trs ou quatro desses tringulos,
decompe-se o PQR em sete, dez, treze ou dezesseis tringulos
issceles (que passaremos a chamar de partes). E assim por diante,
PQR pode ser decomposto em n = 4 + 3k partes, k .
Para decompor em n = 5 partes, consideremos dois casos:
i) PQR no equiltero.
Decompomos o tringulo em dois P
tringulos, um deles issceles, e
aplicamos a) no outro (dividido
em dois tringulos retngulos), Q R
obtendo 5 partes.
Repetindo o procedimento de a) em uma, duas, trs, quatro ou
cinco partes, decompe-se o PQR em oito, onze, catorze,
dezessete ou vinte partes. E assim, em n = 5 + 3k partes, k .
ii) PQR equiltero.
Uma altura separa o PQR em dois trin-
P
gulos retngulos. Num deles aplicamos a),
obtendo duas partes, e decompomos o
outro num tringulo issceles e outro
C
retngulo com vrtice comum no M
RPM OBMEP

circuncentro C do PQR. Aplicando a)


nesse tringulo retngulo, decompe-se o Q R
PQR em n = 5 partes. Como em i),

199
mostra-se que possvel decompor o PQR em n = 5 + 3k
partes, k .
Um PQR no equiltero pode ser
decomposto em trs tringulos, P
sendo dois issceles, e aplicando a)
no terceiro tringulo (dividido em
dois tringulos retngulos), decom-
Q R
pomos o PQR em n = 6 partes.
Se PQR equiltero, utilizando o P
circuncentro C, decompomos em trs partes,
e aplicando a) numa delas obtemos a
decomposio em 6 partes. Como nos casos C
anteriores, o PQR pode ser decomposto
em n = 6 + 3k partes, k . Q R

15.
A funo f deve satisfazer a igualdade 2f(x) + f(1 x) = x2008 para todo
x R. Escrevendo a igualdade para (1 x), obtemos
2f(1 x) + f( 1 (1 x)) = (1 x)2008. Ento,
2 f (1 x) + f ( x) = (1 x) 2008
2008
2 f ( x) + f (1 x) = x
ou, isolando f(1 x) na segunda equao e substituindo na primeira,
2[ x 2008 2 f ( x)] + f ( x) = (1 x) 2008 , o que leva a

2 x 2008 (1 x) 2008
f ( x) = .
3
fcil verificar que a ltima funo satisfaz a equao funcional dada,
logo a nica soluo.
16.
RPM OBMEP

Podemos fatorar 32008 1 da forma seguinte:


32008 1 = (38)251 1 = (38 1)(38250 + 38249 + 38248 + ... + 381 + 1).
Observe que a soma 38250 + 38249 + 38248 + ... + 381 + 1 tem 251

200
parcelas, todas mpares; logo, um nmero mpar, no sendo, portanto,
divisvel por 2. Por outro lado,
38 1 = (34 1)(34 + 1) = 80 82 = 25 5 41.
Ento, a maior potncia de 2 que divide 32008 1 25.
17.
a) Como cada pea na posio (i, j) substituda por duas, ocupando
as posies (i + 1, j) e (i, j + 1), temos que a soma dos pesos no
varia a cada jogada, pois 2ij = 2(i +1)j + 2i(j +1).
b) Para determinar a soma de todos os pesos do tabuleiro, observamos
1
que a soma da primeira coluna = 2 (PG infinita de razo
11 2
1/2 e termo inicial 1). Por outro lado, a soma da segunda coluna
metade da primeira, a da terceira, 1/4 da primeira, e assim por diante,
logo a soma de todos os pesos do tabuleiro
1 1 1 2
2 +1+ + + + ... = = 4.
2 4 8 11 2
c) A soma dos pesos das seis casas iniciais sombreadas
1 1 1 1 1 3
1+ + + + + = 2 + , enquanto a soma dos pesos das casas
2 2 4 4 4 4
3 1
no hachuradas 4 (2 + ) = 1 + .
4 4
Assim, impossvel mover as peas das casas hachuradas inicial-
mente para fora dessa regio, uma vez que a soma dos pesos das
3
casas iniciais 2 + ser mantida, em cada jogada, e maior do que
4
1
a soma dos pesos de todas as casas no hachuradas, 1 + .
4
Observao: o problema ainda no teria soluo caso as peas
RPM OBMEP

iniciais ocupassem apenas as casas de pesos 1, 1/2, 1/2, j que para


atingir o peso 2 da regio no hachurada necessrio utilizar infinitas
casas. Logo, o jogo no terminaria aps um nmero finito de
movimentos.

201
18.
n3 1 (n 1)(n 2 + n + 1)
Temos p = 3
+1
= 2
2 n 2008 n 2 n 2008 ( n + 1)( n n + 1)

n 1 n2 + n + 1
= . 2 .
2 n 2008 n + 1 2 n 2008 n n + 1

Por um lado, temos


n 1 1 2 3 4 2006 2007 1.2
(1) n + 1
= ...
3 4 5 6 2008

2009
=
2008 2
.2 009
.
2 n 2008

Por outro lado, se g(n) = n2 n + 1, temos que


g(n + 1) = (n + 1)2 (n + 1) + 1 = n2 + n + 1 e assim
n2 + n + 1 g (n + 1) g (3) g (4) g (2009)
(2) 2
= g ( n )
=
g ( 2)

g (3)
...
g (2008)
2 n 2008 n n + 1 2 n 2008

g (2009) 20092 2009 + 1 .


= =
g (2) 22 2 + 1
De (1) e (2), conclumos que
1.2 20092 2009 + 1 2 2009.2008 + 1
p= =
2008.2009 22 2 + 1 3 2008.2009
2 1
= (1 + ).
3 2008.2009

19.
Observe inicialmente que, sendo 7 um nmero primo que divide N,
ento necessariamente ele deve dividir algum dos fatores inteiros
(x + 6y), (2x + 5y) ou (3x + 4y). Mas ento observe as relaes abaixo:
3(2 x + 5 y ) + ( x + 6 y ) = 7( x + 3 y )

2(3 x + 4 y ) + ( x + 6 y ) = 7( x + 2 y ) .
RPM OBMEP

2(2 x + 5 y ) + (3 x + 4 y ) = 7( x + 2 y )

Se algum dos fatores for mltiplo de 7, elas implicam que os outros
tambm so, portanto N divisvel por 73 = 343.
202
20.
Suponhamos que exista um ngulo AOB l tal que AO = BO,
AC = CD = BD com os ngulos AOC l , COD
l e BODl congruentes. No
AOD, a ceviana OC mediana e bissetriz; logo, tambm a altura e
l reto. No COB, a ceviana OD mediana e bissetriz;
o ngulo ACO
l reto.
logo, tambm a altura e o ngulo ODC
Assim, o DOC tem dois ngulos retos, o que um absurdo.

RPM OBMEP

203

Você também pode gostar